You are on page 1of 177

Viet Nam Team Selection

Test Collection
From 1989 to 2010
Chapter I

Mục lục Chaper I

1 Đề thi chọn đội tuyển toán 3


1.1 Đề thi chọn đội tuyển toán năm học 1989 - 1990
(Ngày thi: 16, 17/5/1990) . . . . . . . . . . . . . . . . . . . 3
1.2 Đề thi chọn đội tuyển toán năm học 1990 - 1991
(Ngày thi 8, 9/5/1991) . . . . . . . . . . . . . . . . . . . . . 4
1.3 Đề thi chọn đội tuyển năm học 1991 - 1992
(Ngày thi 19, 20/05/1992) . . . . . . . . . . . . . . . . . . . 6
1.4 Đề thi chọn đội tuyển toán năm học 1992 - 1993
(Ngày 4, 5/05/1993) . . . . . . . . . . . . . . . . . . . . . . 7
1.5 Đề thi chọn đội tuyển toán năm học 1993 - 1994
(Ngày 18, 19/05/1994) . . . . . . . . . . . . . . . . . . . . . 8
1.6 Đề thi chọn đội tuyển toán năm học 1994 - 1995
(Ngày 5, 6/5/1995) . . . . . . . . . . . . . . . . . . . . . . . 9
1.7 Đề thi chọn đội tuyển toán năm học 1995 - 1996
(Ngày 17, 18/5/1996) . . . . . . . . . . . . . . . . . . . . . . 11
1.8 Đề thi chọn đội tuyển toán năm học 1996 - 1997
(Ngày 16, 17/5/1997) . . . . . . . . . . . . . . . . . . . . . . 12
1.9 Đề thi chọn đội tuyển toán năm học 1997 - 1998
(Ngày 13, 14/5/1998) . . . . . . . . . . . . . . . . . . . . . 13
1.10 Đề thi chọn đội tuyển năm học 2001 - 2002
(Ngày thi 7, 8/5/2002) . . . . . . . . . . . . . . . . . . . . . 14
1.11 Đề thi chọn đội tuyển toán năm học 2003 - 2004
(Ngày 7, 8/5/2004) . . . . . . . . . . . . . . . . . . . . . . . 15

2 Đáp án tuyển sinh 18


2.1 Đáp án chọn đội tuyển năm học 1991 - 1992 . . . . . . . . . 18
2.2 Đáp án chọn đội tuyển năm học 1992 - 1993 . . . . . . . . . 24
2.3 Đáp án chọn đội tuyển năm học 1993 - 1994 . . . . . . . . . 34
2.4 Đáp án chọn đội tuyển năm học 1994 - 1995 . . . . . . . . . 45
2.5 Đáp án chọn đội tuyển năm học 1995 - 1996 . . . . . . . . . 51
2.6 Đáp án chọn đội tuyển năm học 1996 - 1997 . . . . . . . . . 59

1
2 MỤC LỤC

2.7 Đáp án chọn đội tuyển năm học 1997 - 1998 . . . . . . . . . 66


2.8 Đáp án chọn đội tuyển năm học 2001 - 2002 . . . . . . . . . 76
2.9 Đáp án chọn đội tuyển năm học 2003 - 2004 . . . . . . . . . 81
Chương 1

Đề thi chọn đội tuyển toán

1.1 Đề thi chọn đội tuyển toán năm học 1989


- 1990
(Ngày thi: 16, 17/5/1990)
Bài 1: Trong mặt phẳng cho đa giác lồi M0 , M1 , . . . , M2n (n > 1) mà 2n + 1
đỉnh M0 , M1, . . . , M2n nằm (theo thứ tự ngược chiều quay của kim đồng
hồ) trên một đường tròn (C) bán kính R. Giả sử có điểm A bên trong đa
giác lồi đó sao cho các góc M\ \ \ \
0 AM1 , M1 AM2 , . . . , M2n−1 AM2n , M2n AM0 đều
360
bằng nhau, (và bằng 2n+1 độ). Giả sử A không trùng với tâm của (C) và
gọi B là điểm nằm trên đường tròn (O) sao cho đường thẳng AB vuông góc
với đường kính đi qua A.
Chứng minh:
2n + 1 AM0 + AM1 + · · · + AM2n
1 1 1 < AB < <R
AM0
+ AM1
+ ··· + AM2n
2n + 1

Bài 2: Cho bốn số thực dương a, b, A, B. Xét dãy số thực x1 , x2, x3, x4, . . .
xác định bởi:

x1 = a, x2 = b
p q
xn+1 = A 3 x2n + B 3 x2n−1 (n = 2, 3, 4, . . .)

Chứng minh rằng tồn tại giới hạn limn→∞ xn và hãy tính giới hạn ấy.
Bài 3: Chứng minh rằng không tồn tại hàm số f (x) xác định với mọi
số thực x và thoả mãn f (f (x)) = x2 − 2 với mọi x.
Bài 4: Xét tập hợp T gồm hữu hạn số nguyên dương thoả mãn hai điều
kiện:

3
4 Chương 1. Đề thi chọn đội tuyển toán

1. Với hai phần tử bất kỳ của T thì ước số chung lớn nhất và bội số
chung nhỏ nhất của chúng cũng là những phần tử của T .

2. Với mỗi phần tử x của T , có phần tử x0 của T sao cho x và x0 nguyên


tố cùng nhau và bội số chung nhỏ nhất của chúng là số lớn nhất của
T.

Với mỗi tập hợp T như thế, ký hiệu l(T ) là số phần tử của nó. Tìm số
l(T ) lớn nhất, biết rằng l(T ) nhỏ hơn 1990.
Bài 5: Cho tứ diện mà mỗi cặp cạnh đối diện đều có tích độ dài bằng l.
Gọi các góc giữa các cạnh đối diện đó là α, β, γ và gọi các bán kính của các
đường tròn ngoại tiếp các mặt của tứ diện là R1 , R2 , R3, R4 . Chứng minh:

l
sin2 α + sin2 β + sin2 γ > √
R1 R2 R3 R4

Bài 6: Có n em học sinh (n > 3) đứng thành một vòng tròn và luôn
quay mặt vào cô giáo ở tâm vòng tròn. Mỗi lần cô giáo thổi còi thì có hai
em nào đó đứng sát cạnh nhau đổi chỗ cho nhau, còn các em khác không
dời chỗ. Tìm số M bé nhất để sau M lần thổi còi, bằng các đổi chỗ như
nói ở trên một cách thích hợp, các học sinh đứng được thành vòng tròn sao
cho: Hai em bất kỳ lúc ban đầu đứng sát cạnh nhau thì lúc kết thúc cũng
đứng sát cạnh nhau, nhưng trong hai em đó, tạm gọi là A và B, nếu A lúc
ban đầu đứng bên tay trái của B thì lúc kết thúc A đứng bên tay phải của
B.

1.2 Đề thi chọn đội tuyển toán năm học 1990


- 1991
(Ngày thi 8, 9/5/1991)
Bài 1: Trong mặt phẳng xét tập hợp S gồm n điểm phân biệt (n > 3) thoả
mãn ba điều kiện sau:

1. Khoảng cách giữa hai điểm bất kỳ thuộc S đều không vượt quá 1 đơn
vị dài.

2. Mỗi điểm A thuộc S có đúng hai điểm "kề với nó", nghĩa là hai điểm
thuộc S có cùng khoảng cách bằng 1 đến điểm A.

3. Với hai điểm tuỳ ý A, B thuộc S gọi A0 và A00 là hai điểm kề với A,
gọi B 0 và B 00 là hai điểm kề với B thì A\ \
0 AA00 = B 0 BB 00.
1.2. Đề thi chọn đội tuyển toán năm học 1990 - 1991 (Ngày thi 8, 9/5/1991) 5

Hỏi có tồn tại tập hợp S như thế khi n = 1991 không và khi n = 2000
không? Vì sao?
Bài 2: Cho dãy số thực dương a1, a2, . . . , an với n lớn hơn 2 và a1 khác
an , là dãy không giảm (nghĩa là ak 6 ak+1 với k = 1, 2, . . . , n − 1) hoặc là
dãy không tăng (nghĩa là ak > ak+1 với k = 1, 2, . . . , n − 1), và cho các số
thực dương x, y thoả mãn xy > aa11−a
−a2
n
. Chứng minh rằng:

a1 ak
+ ··· + + ···+
a2 x + a3y ak+1 x + ak+2 y
an−2 an−1 an n
··· + + + >
an−1 x + an y an x + a1 y a1 x + a2 y x+y

Bài 3: Cho dãy số thực dương x1, x2, . . . , xn , . . . xác định bởi:

x1 = 1, x2 = 9, x3 = 9, x4 = 1

xn+4 = 4 xn xn+1 xn+2 xn+3 với n > 1

Chứng minh rằng dãy số trên có giới hạn. Tìm giới hạn đó.
Bài 4: Gọi T là hình tứ diện tuỳ ý thoả mãn hai điều kiện sau:
1. Mỗi cạnh có độ dài không vượt quá 1 đơn vị dài.
2. Mỗi mặt là một tam giác vuông.
Ký hiệu s(T ) là tổng bình phương diện tích bốn mặt của hình tứ diện
T . Tìm giá trị lớn nhất của s(T ).
Bài 5: Với mỗi số tự nhiên n, định nghĩa số f (n) như sau: f (1) = 1
và khi n > 1 thì f (n) = 1 + a1p1 + · · · + ak pk , trong đó n = p1 . . . pk là sự
phân tích thành thừa số nguyên tố của n (các số nguyên tố p1 , . . . , pk đôi
một khác nhau và a1, . . . , ak là số nguyên dương). Với mỗi số tự nhiên s,
đặt fs (n) = f (f (. . . (f (n)) . . .)), trong đó ở vế phải có đúng s lần chữ f .
Chứng minh rằng với số tự nhiên a cho trước, có số tự nhiên s0 để với
mọi số nguyên s > s0 thì tổng fs (a) + fs−1 (a) không phụ thuộc vào s.
Bài 6: Cho tập hợp X gồm 2n số thực đôi một khác nhau (n > 3). Xét
một tập hợp K gồm một số cặp số thực (x, y) với x, y thuộc X, x khác y,
mà K thoả mãn hai điều kiện sau:
1. Nếu cặp số (x, y) thuộc K thì cặp số (y, x) không thuộc K.
2. Mỗi số x thuộc X có mặt nhiều nhất trong 19 cặp số của K.
Chứng minh rằng ta có thể phân chia tập hợp X thành 5 tập hợp con
không rỗng và đôi một không giao nhau x1 , x2, x3, x4 , x5 sao cho với mỗi
i = 1, 2, 3, 4, 5 thì số cặp số (x, y) thuộc K mà x và y cùng thuộc Xi không
vượt quá 3n.
6 Chương 1. Đề thi chọn đội tuyển toán

1.3 Đề thi chọn đội tuyển năm học 1991 -


1992
(Ngày thi 19, 20/05/1992)
Bài 1: Cho hai số tự nhiên n và m (n > 1). Hãy tìm số nguyên dương k
nhỏ nhất có tính chất sau: Trong k số nguyên tuỳ ý a1, a2 , . . . , sk mà ai − aj
(i 6= j và i, j chạy từ 1 đến k) không chia hết cho n, luôn tồn tại hai số
ap, as (p 6= s) thoả mãn m + ap − as chia hết cho n.
Bài 2: Cho đa thức f (x) với hệ số thực và có bậc lớn hơn hoặc bằng 1.
Chứng minh rằng với mỗi số c > 0, tồn tại số nguyên dương n0 thoả mãn
điều kiện sau: Nếu đa thức P (x) với hệ số thực có bậc lớn hơn hoặc bằng
n0 , và có hệ số của số hạng bậc cao nhất bằng 1 thì các số nguyên x mà
|f (P (x))| 6 c không vượt quá bậc của P (x).
Bài 3: Cho tam giác ABC có BC = a, CA = b, AB = c (a 6= b 6= c).
Trong mặt phẳng ABC lấy các điểm A0, B 0, C 0 sao cho:

1. Các cặp điểm A và A0, B và B 0, C và C 0 hoặc đều ở cùng phía hoặc


đều ở khác phía theo thứ tự đối với các đường thẳng BC, CA, AB.

2. Các tam giác A0BC, B 0CA, C 0AB là các tam giác cân đồng dạng.

Hãy xác định các góc A \0 BC theo a, b, c để các độ dài AA0 , BB 0 , CC 0

không phải là ba độ dài của ba cạnh một tam giác.


(Tam giác được hiểu theo nghĩa thông thường: ba đỉnh của nó không
thẳng hàng).
Bài 4: Trong mặt phẳng cho một họ hữu hạn hình tròn thoả mãn: hai
hình tròn bất kỳ hoặc ở ngoài nhau hoặc tiếp xúc ngoài với nhau và mỗi
hình tròn không tiếp xúc với quá 6 hình tròn khác. Giả sử mỗi hình tròn
không tiếp xúc với 6 hình tròn khác đã được đặt ứng với một số thực nào
đó. Chứng minh rằng không có quá một cách đặt ứng với mỗi hình tròn còn
lại một số thực bằng trung bình cộng của 6 số ứng với 6 hình tròn tiếp xúc
nó.
Bài 5: Tìm tất cả các cặp số nguyên dương (x, y) thoả mãn phương
trình
x2 + y 2 − 5xy + 5 = 0
.
Bài 6: Trong một hội thảo khoa học tất cả các đại biểu tham dự biết
tổng cộng 2n ngôn ngữ n > 2. Mỗi người biết đúng 2 ngôn ngữ và bất cứ
hai người nào cũng biết chung nhiều nhất một ngôn ngữ. Biết rằng với một
số nguyên k thoả mãn 1 6 k 6 n − 1 đều có không quá k − 1 ngôn ngữ mà
mỗi ngôn ngữ này có không quá k người biết. Chứng minh rằng ta có thể
1.4. Đề thi chọn đội tuyển toán năm học 1992 - 1993 (Ngày 4, 5/05/1993) 7

chọn ra một nhóm 2n đại biểu biết tổng cộng 2n ngôn ngữ và mỗi ngôn ngữ
có đúng 2 đại biểu trong nhóm biết.

1.4 Đề thi chọn đội tuyển toán năm học 1992


- 1993
(Ngày 4, 5/05/1993)
Bài 1: Gọi hình chữ nhật kích thước 2 × 3 (hoặc 3 × 2) bị cắt bỏ một hình
vuông 1 × 1 ở một góc là hình chữ nhật khuyết đơn (xem hình 1). Gọi hình
chữ nhật kích thước 2 × 3 (hoặc 3 × 2) bị căt bỏ hai hình vuông 1 × 1 ở hai
góc đối diện là hình chữ nhật khuyết kép (xem hình 2). Người ta ghép một
số hình vuông 2 × 2, một số hình chữ nhật khuyết đơn và một số hình chữ
nhật khuyết kép với nhau sao cho không có hai hình nào chờm lên nhau,
để tạo thành một hình chữ nhật kích thước 1993 × 2000. Gọi s là tổng số
các hình vuông 2 × 2 và hình chữ nhật khuyết kép cần dùng trong mỗi cách
ghép hình nói trên. Tìm giá trị lớn nhất của s.

Bài 2: Cho dãy số {an } được xác định bởi:

1
a1 = 1 và an+1 = an + √ với n = 1, 2, 3, . . .
an


Hãy tìm tất cả các số thực α sao cho dãy {un } xác định bởi un = n
n
với
n = 1, 2, 3, . . . có giới hạn hữu hạn khác 0 khi n → +∞.
Bài 3: Xét các số thực x1, x2, x3 , x4 thoả mãn:

1
6 x21 + x22 + x23 + x24 6 1
2

Tìm giá trị lớn nhất và nhỏ nhất của biểu thức:

A = (x1 − 2x2 + x3 )2 + (x2 − 2x3 + x4)2 + (x2 − 2x1 )2 + (x3 − 2x4 )2


8 Chương 1. Đề thi chọn đội tuyển toán

Bài 4: Gọi H, I, O theo thứ tự là trực tâm, tâm đường tròn nội tiếp và
tâm đường tròn ngoại tiếp của một tam giác. Chứng minh rằng 2.IO > IH.
Hỏi dấu bằng xảy ra khi nào?
Bài 5: Cho số nguyên k > 1. Với mỗi số nguyên n > 1, đặt
1 1 1
f (n) = k.n(1 − )(1 − ) . . . (1 − )
p1 p2 pr

trong đó p1 , p2 , . . . , pr là tất cả các ước số nguyên tố phân biệt của n. Tìm


tất cả các giá trị k để dãy {xm} xác định bởi x0 = a và xm+1 = f (xm ), m =
0, 1, 2, 3, . . . là dãy bị chặn với mọi số nguyên a > 1.
Bài 6: Xét n điểm A1 , A2, . . . , An (n > 2) trong không gian, trong đó
không có 4 điểm nào đồng phẳng. Mỗi cặp điểm Ai , Aj (i 6= j) được nối với
nhau bởi một đoạn thẳng.
Tìm giá trị lớn nhất của n sao cho có thể tô tất cả các đoạn thẳng đó
bằng hai màu xanh, đỏ thoả mãn ba điều kiện sau:

1. Mỗi đoạn thẳng được tô bằng đúng một màu.

2. Với mỗi i = 1, 2, . . . , n số đoạn thẳng có một đầu mút là Ai mà được


tô màu xanh không vượt quá 4.

3. Với mỗi đoạn thẳng Ai , Aj được tô màu đỏ đều tìm thấy ít nhất một
điểm Ak (k khác i, j) mà các đoạn thẳng Ak Ai và Ak Aj đều được tô
màu xanh.

1.5 Đề thi chọn đội tuyển toán năm học 1993


- 1994
(Ngày 18, 19/05/1994)
Bài 1: Given a parallelogram ABCD. Let E be a point on the side BC and
F be a point on the side CD such that the triangles ABE and BCF have
the same are. The diagonal BD intersects AE at M and intersects AF at
N . Prove that.
a) There exists a triangle, three sides of which are equal to BM, MN, ND.
b) When E, F vary such that the length sides of MN decreases, the
radius of the circumcircle of the abovementioned triangle also decreases.
Bài 2: Consider the equation

x2 + y 2 + z 2 + t2 − Nxyzt − N = 0

where N is a given positive integer.


1.6. Đề thi chọn đội tuyển toán năm học 1994 - 1995 (Ngày 5, 6/5/1995) 9

a) Prove that for an infinite number of values of N , this equation has


positive integral solution (each such solution consists of four positive integers
x, y, x, t).
b) Let N = 4k (8m + 7) where k, m are non-negative integers. Prove that
the considered equation has no positive integral solution.
Bài 3: Let be given a polynomial P (x) of degree 4, having 4 positive
roots. Prove that the equation
1 − 4x 1 − 4x 0
2
P (x) + (1 − )P (x) − P 00(x) = 0
x x2
has also 4 positive roots.
Bài 4: Given an equilateral triangle ABC and a point M in the plan
(ABC). Let A0, B 0, C 0 be respectively the symmetric through M of A, B, C.
a) Prove that there exists s unique point P equidistant from A and B 0,
from B and C 0 and from C and A0.
b) Let D be the midpoint of the side AB. When M varies (M does not
coincide with D), prove that the circumcircle of triangle MNP (N is the
intersection of the lines DM and AP ) passes through a fixed point.
Bài 5: Determine all function f : R → R satisfying
√ √ √
f ( 2x) + f ((4 + 3 2)x) = af ((2 + 2)x)
for all x.
Bài 6: Calculate
1
T =
n1 !n2 ! . . . n1994!(n2 + 2n3 + 3n4 + · · · 1993n1994 )!
where the sum is taken over all 1994-upple of natural numbers (n1, n2 , . . . , n1994)
satisfying
n1 + 2n2 + 3n3 + · · · + 1994n1994 = 1994

1.6 Đề thi chọn đội tuyển toán năm học 1994


- 1995
(Ngày 5, 6/5/1995)
Bài 1. Cho tam giác ABC với BC = a, CA = b, AB = c. Lấy sáu điểm
phân biệt A1 , A2, B1, B2 , C1, C2 không trùng với A, B, C sao cho các điểm
A1, A2 nằm trên đường thẳng BC; các điểm B1 , B2 nằm trên đường thẳng
CA; các điểm C1, C2 nằm trên đường thẳng AB. Gọi α, β, γ là các số thực
xác định bởi
−−−→ α −−
→ −−−→ β −→ −−−→ γ −→
A1A2 = BC, B1B2 = CA, C1 C2 = AB.
a b c
10 Chương 1. Đề thi chọn đội tuyển toán

Xét các đường tròn ngoại tiếp các tam giác AB1 C1,AB2C2 , BC1A1, BC2A2 ,
CA1B1, CA2B2 và gọi dA , dB , dC theo thứ tự là các trục đẳng phương của
cặp đường tròn đi qua A, cặp đường tròn đi qua B, cặp đường tròn đi qua
C. Chứng minh rằng dA , dB , dC đồng quy khi và chỉ khi

aα + bβ + cγ 6= 0.

Bài 2. Tìm tất cả các số nguyên k sao cho có vô số giá trị nguyên n ≥ 3
để đa thức

Pn (x) = xn+1 + kxn − 870x2 + 1945x + 1995

có thể phân tích được thành tích của hai đa thức với hệ số nguyên có bậc
lớn hơn hay bằng 1.
Bài 3. Tìm tất cả các số nguyên a, b, n lớn hơn 1 thoả mãn điều kiện

(a3 + b3)n = 4(ab)1995.

Bài 4. Trong không gian cho n điểm (n ≥ 2) mà không có bốn điểm nào
đồng phẳng và cho 12 (n2 − 3n + 4) đoạn thẳng mà tất cả các đầu mút của
chúng nằm trong số n điẻm đã cho. Biết rằng có ít nhất một đoạn thẳng
mà sau khi bỏ nó đi (giữ nguyên các đầu mút) thì sẽ tồn tại hai điểm phân
biệt mà không phải là hai đầu mút của một đường gấp khúc nào.
Hãy tìm số k lớn nhất sau cho có k đoạn thẳng tạo thành đường gấp
khúc khép kín mà mỗi đỉnh của nó là mút của đúng hai đoạn thẳng thuộc
đường gấp khúc đó.
Bài 5. Với mỗi số nguyên không âm n đặt f (n) là số nguyên không âm
lớn nhất sao cho 2f (n) là một ước số của n + 1. Cặp số nguyên không âm
(n, p) được gọi là đẹp nếu 2f (n) > p. Hãy tìm tất cả các bộ ba số nguyên
không âm (n, p, q) sao cho các cặp số (n, p), (p, q), và (n + p + q, n) đều là
các cặp số đẹp.
Bài 6. Cho hàm số thực
2x3 − 3
f (x) = .
3(x2 − 1)

1. Chứng minh rằng tồn tại hàm số g(x) liên tục trên R và có đồng thời
các tính chất sau

f (g(x)) = x, ∀x ∈ R; g(x) > x ∀x ∈ R.

2. Chứng minh rằng tồn tại số thực a > 1 để dãy {an }, n = 0, 1, 2, ...,
được xác định bởi a0 = a, an+1 = f (an ) ∀n ∈ N là dãy tuần hoàn với
chu kỳ dương nhỏ nhất bằng 1995.
1.7. Đề thi chọn đội tuyển toán năm học 1995 - 1996 (Ngày 17, 18/5/1996) 11

1.7 Đề thi chọn đội tuyển toán năm học 1995


- 1996
(Ngày 17, 18/5/1996)
Bài 1. Trong mặt phẳng cho 3n điểm (n > 1) mà không có ba điểm nào
thẳng hàng và khoảng cách giữa hai điểm bất kỳ không vượt quá 1. Chứng
minh rằng có thể dựng được n tam giác đôi một rời nhau và thoả mãn đồng
thời các điều kiện sau

1. Mỗi điểm trong 3n điểm đã cho là đỉnh của đúng một tam giác;

2. Tổng diện tích của n tam giác nhỏ hơn 12 .

Hai tam giác được gọi là rời nhau nếu chúng không có điểm nào
chung nằm bên trong cũng như nằm trên cạnh tam giác.

Bài 2. Với mỗi số nguyên dương n, gọi f (n) là số nguyên lớn nhất để
số
[ n−1 ] 
X 2
2i + 1 i
3 chia hết cho 2f (n) .
i=0
n

Tìm tất cả các số nguyên dương n mà f (n) = 1996.


Bài 3. Xét các số thực a, b, c. Tìm giá trị nhỏ nhất của biểu thức

f (a, b, c) = (a + b)4 + (b + c)4 + (c + a)4 − 47 (a4 + b4 + c4 ).

Bài 4. Cho ba điểm A, B, C không thẳng hàng. Với mỗi điểm M của
mặt phẳng (ABC) gọi M1 là điểm đối xứng của M qua đường thẳng AB,
gọi M2 là điểm đối xứng của M1 qua đường thẳng BC và gọi M 0 là điểm
đối xứng của M2 qua đường thẳng CA. Hãy xác định tất cả các điểm M
của mặt phẳng (ABC) mà khoảng cách MM 0 bé nhất. Gọi khoảng cách
đó là d. Chứng minh rằng với mỗi điểm M của mặt phẳng (ABC) khi ta
thực hiện liên tiếp ba phép đối xứng qua ba đường thẳng chứa ba cạnh của
tam giác ABC theo thứ tự khác (so với thứ tự trên) để được điểm M 00 thì
khoảng cách bé nhất của MM 00 cũng bằng d.
Bài 5. Người ta muốn mời một số em học sinh tới dự một buổi gặp mặt,
mà trong số đó mỗi em chưa quen với ít nhất là 56 em khác, và với mỗi
cặp hai em chưa quen nhau thì đều có ít nhất một em quen với cả hai em
đó. Hỏi số học sinh được mời dự buổi gặp mặt nói trên có thể là 65 em hay
không?
12 Chương 1. Đề thi chọn đội tuyển toán

Bài 6. Hãy tìm tất cả các số thực a sao cho dãy số {xn }, n = 0, 1, 2, ...,
xác định bởi
√ a
x0 = 1996, xn+1 = với n = 0, 1, 2, , ...
1 + x2n
có giới hạn hữu hạn khi n → ∞.

1.8 Đề thi chọn đội tuyển toán năm học 1996


- 1997
(Ngày 16, 17/5/1997)
Bài 1. Cho tứ diện ABCD với BC = a, CA = b, AB = c, DA = a1 , DB =
b1, DC = c1 . Chứng minh rằng có điểm P duy nhất thoả mãn
P A2 +a21 +b2 +c2 = P B 2 +b21 +c2 +a2 = P C 2 +c21 +a2 +b2 = P D2 +a21 +b21 +c21
và với điểm P đó ta luôn có P A2 + P B 2 + P C 2 + P D2 ≥ 4R2 , trong đó R
là bán kính mặt cầu ngoại tiếp tứ diện ABCD. Tìm điều kiện cần và đủ
với độ dài các cạnh của tứ diện để bất đẳng thức trên trở thành đẳng thức.
Bài 2. Ở một nước có 25 thành phố. Hãy xác định số k bé nhất sao cho
có thể thiết lập các đường bay (dùng cho cả đi lẫn về) giữa các thành phố
để hai điều kiện sau được đồng thời thoả mãn
1. Từ mỗi thành phố có đường bay trực tiếp đến đúng k thành phố khác;
2. Nếu giữa hai thành phố không có đường bay trực tiếp thì tồn tại ít
nhất một thành phố có đường bay trực tiếp đến hai thành phố đó.
Bài 3. Hãy tìm số thực α lớn nhất sao cho tồn tại vô hạn số tự nhiên
(an ), n = 1, 2, 3, ..., thoả mãn đồng thời các điều kiện sau
1. an > 1997n với mọi n ∈ N∗ ;
2. với mỗi n ≥ 2 đều có un ≥ aαn , trong đó un là ước số chung lớn nhất
của họ tất cả các số ai + ak mà i + k = n.
Bài 4. Cho hàm số f : N → Z thoả mãn các điều kiện f (0) = 2, f(1) =
503 và f (n + 2) = 503f (n + 1) − 1996f (n) với mọi n ∈ N.
Với mỗi số k ∈ N∗ lấy số nguyên s1, s2 , ..., sk sao cho si ≥ k với mọi
i = 1, 2, ..., k. Với mỗi số si (i = 1, 2, ..., k) lấy một ước nguyên tố p(si ) nào
đó của f (2si ). Chứng minh rằng với số nguyên dương t ≤ k, ta có
X
k
. .
p(si )..2t khi và chỉ khi k ..2t .
i=1
1.9. Đề thi chọn đội tuyển toán năm học 1997 - 1998 (Ngày 13, 14/5/1998) 13

Bài 5. Hãy xác định tất cả các cặp số thực a, b sao cho với mọi n ∈ N∗
và với mọi nghiệm thực xn của phương trình
4n2 x = log2 (2n2 x + 1)
ta luôn có
axn + bxn ≥ 2 + 3xn .
Bài 6. Cho các số nguyên dương n, k, p với k ≥ 2 và k(p + 1) ≤ n. Cho
n điểm phân biệt cùng nằm trên một đường tròn. Tô tất cả n điểm đó bởi
hai màu xanh, đỏ (mỗi điểm tô bởi một màu) sao cho có đúng k điểm được
tô bởi màu xanh và trên mỗi cung tròn mà hai đầu mút là hai điểm màu
xanh liên tiếp (tính theo chiều quay của kim đồng hồ) đều có ít nhất p điểm
được tô bởi màu đỏ.
Hỏi có tất cả bao nhiêu cách tô màu khác nhau?
(Hai cách tô màu được gọi là khác nhau nếu có ít nhất một điểm được
tô bởi hai màu khác nhau trong hai cách đó).

1.9 Đề thi chọn đội tuyển toán năm học 1997


- 1998
(Ngày 13, 14/5/1998)
Bài 1. Cho hàm số f (x) xác định trên R sao cho với mọi số thực dương c
tồn tại đa thức hệ số thực Pc (x) thoả mãn
|f (x) − Pc (x)| ≤ cx1998 với mọi x ∈ R.
Chứng minh rằng f (x) là một đa thức với hệ số thực.
Bài 2. Trong mặt phẳng cho đường tròn (C) bán kính R chứa và tiếp
xúc với đường tròn (C 0) bán kính R2 . Xét họ H các đường trong bên trong
(C), bên ngoài (C 0), tiếp xúc với (C) và (C 0). Với mỗi số nguyên n ≥ 3 và
các số dương p1 , pn , chứng minh rằng hệ thức
(p1 − pn )2 = (n − 1)2 (2(p1 + pn ) − (n − 1)2 − 8)
là điều kiện cần và đủ để có n đường tròn phân biệt (C1 ), (C2 ), ..., (Cn) của
họ H mà (Ci ) tiếp xúc ngoài với (Ci−1 ) và (Ci+1 ) (i = 2, 3, ..., n − 1), ở đó
(C1 ) có bán kính pR1 , (Cn ) có bánh kính pRn .
Bài 3. Cho các số nguyên dương m > 3. Giả sử p1 , p2 , ..., pk là tất cả
các số nguyên tố không vượt quá m. Chứng minh rằng
k 
X 1 1
+ > ln(ln m).
i=1
pi p2i
14 Chương 1. Đề thi chọn đội tuyển toán

Bài 4. Tìm tất cả các đa thức P (x) hệ số nguyên với hệ số bậc cao nhất
bằng 1, có tính chât: Tồn tại vô số các số vô tỉ α để P (α) đều là số nguyên
dương.
Bài 5. Giả sử d là ước dương của 5 + 19981998 . Chứng minh rằng d có
thể biểu diễn dưới dạng d = 2x2 + 2xy + 3y 2 , ở đó x, y là các số nguyên khi
và chỉ khi d chia cho 20 có dư 3 hoặc 7.
Bài 6. Trong một cuộc hội thảo có n, n ≥ 10 người tham dự. Biết rằng
 
1. Mỗi người quen với ít nhất n+2 3
người tham dự.

2. Hai người bất kỳ A và B nếu không quen nhau thì quen nhau gián
tiếp, nghĩa là có k (k ≥ 1) người A1, A2, ..., Ak sao cho A quen A1 , Ai
quen Ai+1, (i = 1, 2, ..., k − 1) và Ak quen B.

3. Không thể xếp n người thành một hàng ngang sao cho hai người cạnh
nhau bất kỳ đều quen nhau.

Chứng minh rằng có thể chia n người thành hai nhóm: nhóm thứ nhất xếp
được quanh một bàn tròn sao cho hai người cạnh nhau bất kỳ đều quen
nhau, còn nhóm thứ hai gồm người đôi một không quen nhau.

1.10 Đề thi chọn đội tuyển năm học 2001 -


2002
(Ngày thi 7, 8/5/2002)
[ là góc nhọn và đường trung
Bài 1. Tìm tất cả các tam giác ABC có BCA
[
trực của đoạn thẳng BC cắt các tia Ax và Ay, là các tia chia góc BAC
[ = xAy
thành ba phần bằng nhau (BAx d = yAC)
[ tại các điểm M và N thoả
mãn:
AB = NP = 2HM
trong đó: H là hình chiếu vuông góc của A trên BC và M là trung điểm
của đoạn thẳng BC.
Bài 2. Người ta ghi lên bảng số nguyên dương N0 . Hai người A và B
chơi trò chơi trò chơi sau: Người A xoá số N0 rồi ghi lên bảng số N1 ∈
{N0 − 1; [N0 /3]}. Tiếp theo người B xoá số N1 rồi ghi lên bảng số N2 ∈
{N1 − 1; [N1 /3]}. Đến lượt mình người A lại thực hiện phép toán trên đối
với N2 ;...Trò chơi cứ tiếp tục cho đến khi trên bảng xuất hiện số 0. Người
ghi số 0 đầu tiên được coi là thắng cuộc, người còn lại bị coi là thua cuộc.
Hỏi ai, người A hay người B, là người có cách chơi để chắc chắn thắng nếu:
1) N0 = 120
1.11. Đề thi chọn đội tuyển toán năm học 2003 - 2004 (Ngày 7, 8/5/2004) 15

2) N0 = (32002 − 1)/2
3) N0 = (32002 + 1)/2 √
Bài 3. Cho số nguyên dương m có một ước nguyên tố lớn hơn 2m + 1.
Hãy tìm số nguyên dương M nhỏ nhất sao cho tồn tại một tập hợp gồm
hữu hạn số nguyên dương đôi một khác nhau thoả mãn đồng thời các điều
kiện sau:
i) m và M tương ứng là số nhỏ nhất và số lớn nhất trong T .
ii) Tích tất cả các số thuộc T là một số chính phương.
Bài 4. Cho số nguyên dương n ≥ 2 và cho bảng ô vuông kích thước
n × 2n (bảng gồm n hàng và 2n cột). Người ta đánh dấu một cách ngẫu
nhiên n2 ô vuông con của bảng. Chứng minh rằng với mỗi số nguyên k mà
1 < k ≤ [n/2] + 1, luôn tồn tại k hàng sao cho bảng ô vuông kích thước
k × 2n, được tạo nên từ k hàng đó, có không ít hơn

k!(n − 2k + 2)
(n − k + 1)(n − k + 2) . . . (n − 1)

cột chỉ gồm các ô được đánh dấu.


([a] ký hiệu số nguyên lớn nhất không vượt quá a).
Bài 5. Hãy tìm tất cả các đa thức P (x) với hệ số nguyên sao cho đa
thức
Q(x) = (x2 + 6x + 10)[P (x)]2 − 1
là bình phương của một đa thức với hệ số nguyên.
Bài 6. Chứng minh rằng tồn tại số nguyên m ≥ 2002 và m số nguyên
dương đôi một khác nhau a1, a2, . . . , am sao cho số

Y
m X
m
a2i −4 a2i
i=1 i=1

là số chính phương.

1.11 Đề thi chọn đội tuyển toán năm học


2003 - 2004
(Ngày 7, 8/5/2004)
Bài 1. Xét tập hợp S gồm 2004 số nguyên dương phân biệt a1, a2, . . . , a2004,
có tính chất: Nếu với mỗi i = 1, 1, . . . , 2004, ký hiệu f (ai ) là số các số thực
thuộc S nguyên tố cùng nhau với ai thì d(ai ) < 2003 và f (ai ) = f (aj ) với
mọi i, j ∈ {1, 2, . . . , 2004}.
16 Chương 1. Đề thi chọn đội tuyển toán

Hãy tìm số nguyên dương k nhỏ nhất sao cho trong mỗi k tập con của
một tập S tuỳ ý có tính chất nêu trên đều tồn tại hai số phân biệt mà ước
số chung lớn nhất của chúng khác 1.
(k - tập con là tập con có k phần tử).
Bài 2. Hãy xác định tất cả các số thực α mà ứng với mỗi α, có một và
chỉ một hàm số f xác định trên tập hợp R, lấy giá trị trong R và thoả mãn
hệ thức.
f (x2 + y + f (y)) = (f (x))2 + αy
với mọi x, y thuộc R.
(R ký hiệu tập hợp các số thực).
Bài 3. Trong mặt phẳng, cho hai đường tròn (O1) và (O2 ) cắt nhau tại
hai điểm A và B. Các tiếp tuyến tại A và B của đường tròn (O1 ) cắt nhau
tại điểm K. Xét một điểm M (không trùng với A và B) nằm trên đường
tròn (O1 ). Gọi P là giao điểm thứ hai của đường thẳng MA và đường tròn
(O2 ). Gọi C là giao điểm thứ hai của đường thẳng MK và đường tròn (O1 ).
Gọi Q là giao điểm thứ hai của đường thẳng CA và đường tròn (O2 ). Chứng
minh rằng:
1) Trung điểm của đoạn thẳng P Q nằm trên đường thẳng MC.
2) Đường thẳng P Q luôn đi qua một điểm cố định khi điểm M di động
trên đường tròn (O1 ).
((O) ký hiệu đường tròn tâm O).
Bài 4. Cho dãy số (xn ), n = 1, 2, 3, . . . xác định bởi
p
x1 = 603, x2 = 102 và xn+2 = xn+1 +xn +2 xn+1 xn − 2 với mọi n ≥ 1

Chứng minh rằng:


1) Tất cả các số hạng của dãy số đã cho đều là các số nguyên dương.
2) Tồn tại vô số số nguyên dương n sao cho biểu diễn thập phân của xn
có bốn chữ số tận cùng là 2003.
3) Không tồn tại số nguyên dương n mà biểu diễn thập phân của xn có
bốn chữ số tận cùng là 2004.
Bài 5. Xét lục giác lồi ABCDEF . Gọi A1, B1, C1 , D1 , E1 , F1 lần lượt là
trung điểm của các cạnh AB, BC, CD, DE, EF, F A. Ký hiệu p và p1 tương
ứng là chu vi của lục giác ABCDEF và của lục giác A1B1 C1D1 E1 F1 . Giả
sử lục giác A1B1 C1D1 E1F1 có tất cả các góc trong bằng nhau. Chứng minh
rằng: √
2 3
p≥ p1
3
Hỏi dấu đẳng thức xảy ra khi và chi khi nào?
Bài 6. Cho S là một tập hợp gồm một số số nguyên dương mà số nhỏ
nhất và số lớn nhất trong S là hai số nguyên tố cùng nhau.
1.11. Đề thi chọn đội tuyển toán năm học 2003 - 2004 (Ngày 7, 8/5/2004) 17

Với mỗi số tự nhiên n, ký hiệu Sn là tập hợp gồm tất cả các số tự nhiên
mà mỗi số đều là tổng của nhiều nhất n số (không nhất thiết đôi một khác
nhau) thuộc tập S. Quy ước 0 là tổng của 0 số thuộc S. Gọi a là số lớn
nhất trong S. Chứng minh rằng tồn tại số nguyên dương k và số nguyên b
sao cho
|Sn | = an + b
với mọi n > k.
(|X| ký hiệu số phần tử của tập hợp X)
Chương 2

Đáp án tuyển sinh

2.1 Đáp án chọn đội tuyển năm học 1991 -


1992
Bài 1. Trong trường hợp m chia hết cho n (kể cả khi m = 0 (nếu coi 0 là
số tự nhiên, chia hết cho n)), rõ ràng không có số nguyên k > 1 thoả mãn
đề bài mà k ≤ n. Vậy k = n + 1.
Sau đây xét m > 0, m không chia hết cho n, (n > 1).
với l ∈ Z. Xét

ϕ(l) : Zn = Z/nZ → Zn
x 7→ lm + x = lm + x

thì nó xác định tác động của nhóm cộng Z lên Zn . Nhóm dừng tại x gồm
các l ∈ Z mà lm = 0 tức là lm (một bội của m) là bội của n, vậy nhóm dừng
đó goòm các bội của BSCBN(m,n)
m
= m.n
d m
. 1 = nd = n0 , trong đó d = (m, n).
Từ đó mỗi quỹ đạo α của tác động nói trên có n0 phần tử, cụ thể là dãy
xα thuộc α thì α = {ϕ(l)(xα) | l = 01, ..., n0 − 1} và Zn là hợp rời rạc của
n
n0
= d quỹ đạo như thế.
Chú ý: do m không chia hết cho n nên n0 > 1. Vậy số N = d[ n20 ] + 1 > 1
và rõ ràng N ≤ n. Hãy chứng minh N bằng số k cần tìm.
1) a1, a2, ..., aN là N phần tử phân biệt của Zn thì do có đúng d quỹ đạo
rời nhau nên có hơn [ n20 ] phần tử ai đó nằm trong một quỹ đạo α nào đó và
do α có n0 phần tử, có ap, as thuộc α mà ϕ(1)(ap ) = as , tức m + ap = as
hay m + ap − as = 0.
2) Khi d = 1 và n0 = 2 hay 3 thì N = 2 rõ ràng có tính chất bé nhất
cần tìm, tức N = k.
Trong các trường hợp khác thì N > 2 và lấy trong mỗi quỹ đạo α phần tử
xα thì tập hợp {ϕ(2l)(xα) | l = 0, 1, ..., [ n20 ], α chạy qua tập các quĩu đạo}

18
2.1. Đáp án chọn đội tuyển năm học 1991 - 1992 19

gồm N − 1 phần tử phân biệt của Zn mà không có hai phần tử khác nhau
nào có hiệu bằng m. Vậy N có tính chất bé nhất đang xét.
Kết luận: m chia hết cho n (kể cả m = 0): k = n + 1. Còn các trường
hợp khác: đặt d = (m, n), n0 = nd thì k = d[ n20 ] + 1.
Bài 2. Do f (x) là đa thức có bậc không nhỏ hơn 1 nên |f (x)| → +∞
khi |x| → +∞, vậy có x0 > 0 để |f (x)| > c với mọi x mà |x| > x0 . Kí hiệu
n0 là số nguyên dương bé nhất thoả mãn 2nn00! > x0. Hãy chứng minh n0 là
số cần tìm.
Giả sử p(x) là đa thức có deg P = k ≥ n0 và có hệ số của số hạng bậc
k bằng 1. Với k + 1 số nguyên phân biệt tuỳ ý b1 < b2 < · · · < bk+1 , theo
công thức nội suy Lagrange, ta có
X
k+1 Y (x − bj )
P (x) = P (bi ) .
i=1 j6=i
(bi − bj )

Tính chất hệ số của số hạng bậc cao nhất của P (x) bằng 1 cho

X
k+1 Y 1
1= P (bi ).
i=1
bi − bj
j6=i

X
k+1
1
≤ max |P (bi )|.
1≤i≤k+1
i=1
(bi − b1) · · · (bi − bi−1 )(bi+1 − bi ) · · · (bk+1 − bi )
X
k+1
1
≤ max |P (bi )| (do bj − bl ≥ j − l ∀j > l)
1≤i≤k+1
i=1
(i − 1)!(k + 1 − i)!
X 2k  
k
1 j
≤ max |P (bi )| Ck = max |P (bi )|
1≤i≤k+1
j=0
k! k! 1≤i≤k+1

Từ đó
k! n0 !
max |P (bi )| ≥ k
≥ n > x0
1≤i≤k+1 2 2 0
.
Vậy có i ∈ {1, 2, ..., k + 1} để |f (P (bi ))| > c, tức là số các số nguyên x
mà |f (P (x))| ≤ c không vượt quá k = deg P .
Bài 3. Coi các tam giác cân A0BC, B 0CA, C 0AB có các đỉnh cân theo
thứ tự là A0, B 0, C 0, (đỉnh cân đối diện với đáy).
Coi tam giác ABC xác định hướng thuận trong mặt phẳng và đặt θ =
(AC 0, AB) = (BA0, BC) = (CB 0, CA) (góc định hướng), ở đây − π2 < θ < π2 ,
θ 6= 0, và A0, B 0, C 0 theo thứ tự thuộc các trung trực của BC, CA, AB.
−−→ −→ −−→ −→ − →− → −−→
− −→ −−→
− −
−→
1) AA0 = AB + BA0 = AB + f (BC), BB 0 = BC + CB 0 = BC +
→ −→ −−→0 −→ −−→0 −→ −
− → −→ →

f (CA), CC = OA + AC + OA + f (AB), trong đó f là tích, phép vị tự
20 Chương 2. Đáp án tuyển sinh

1
−−→ −−→ −−→
(véctơ) hệ số 2 cos θ
với phép quay (véctơ) góc −θ. Vậy AA0 + BB 0 + CC 0 =
−→ − −→ −→ − →− −→ −→ −→ →
− −−→ −−→ −−→
AB + BC + CA + f (BC + CA + AB) = 0 . Chú ý rằng AA0, BB 0, CC 0
là những véctơ khác véctơ không (vì a, b, c khác nhau đôi một) nên suy ra
−−→
luôn có tam giác có cạnh dài AA0, BB 0, CC 0 trừ khi và chỉ khi AA0 song
−−→
song với BB 0.

−−→ −−→
2) Với hai véctơ CM, ON trong mặt phẳng đã xác định hương, kí hiệu

 −−→ − → −−→ − →
0 nếu OM = 0 hay ON = 0
−−→ −−→  −−→ −−→ −−→ →− −−→ − →
OM × ON = |OM ||ON | sin(OM, ON ) nếu OM 6 0 , ON 6= 0 hay

 −−→ −→
ON = 0
−−→
Lấy hệ toạ độ Đềcac vuông góc định hương thuận trong mặt phẳng, OM (x, y),
−−→ 0 0
ON (x , y ) thì do sin(OM, ON ) = sin((Ox, ON ) − (Ox, OM )) tính được
−−→ −−→ −−→ −−→ −−→ −−→
OM × ON = xy 0 − x0y.Từ đó dễ thấy (OM + OM 0 ) × CN = OM × ON +
−−→0 −−→ −−→ −−→ −−→ −−→ −−→ −−→ −−→
OM × CN , OM × (ON + ON 0 ) = OM × ON + OM × CN 0.
−−→ −−→
Trở lại bài toán: Dễ thấy từ định nghĩa AA0 song song với BB 0 khi và
−−→ −−→
chỉ khi AA0 × BB 0 = 0.
Ta có
−−→ −−→ −→ −−→ −−→ −−→ −→ − −→ −→ −−→
AA0 × BB 0 = (AB + BB 0) × (BC + CB 0) = AB × BC + AB × CB 0+
−−→ − −→ −−→ −−→
+ BA0 × BC + BA0 × CB 0
−→ − −

Tính từng số hạng của tổng này: AB× BC = 2S, S là diện tích của tam giác
−→ b b trong đó A b = (AB, AC), để ý rằng
ABC; AB × CB 0 = c 2 cos θ
sin(θ − A),
(AB, CB 0) = (AB, AC) + (AC, CA) + (CA, CB 0) = A b + π − θ; −
−→ − −→
BA0 × BC =
a
−−→ −−→ a b b trong đó C b = (CA, AB), để ý
2 cos θ
a sin θ; BA0 × CB 0 = 2 cos θ 2 cos θ
sin C,
rằng (BA0, CB 0) = (BA0, BC) + (BC, CA) + (CA, CB 0) = θ + π − C b−θ =
π − C. b
2.1. Đáp án chọn đội tuyển năm học 1991 - 1992 21

Vậy
−−→ −−→ 2
bc b + a sin θ + ab sin C b
AA0 × BB 0 = 2S + sin(θ − A)
2 cos θ 2 cos θ 4 cos2 θ
1 1 3
= S tan2 θ + (a2 + b2 + c2 ) tan θ + S.
2 4 2
Từ đó θ cần tìm là nghiệm của phương trình
1
S tan2 θ + (a2 + b2 + c2 ) tan θ + 3S = 0.
2
Biệt thức của tam thức bậc hai đối với tan θ ở vế trái là
1
∆ = (a2 + b2 + c2 ) − 12S 2
4
1
= {(a2 − b2)2 + (b2 − c2 )2 + (c2 − a2)2 } > 0.
2
(Để ý công thức Heron: 16S 2 = 2(a2b2 + b2 c2 + c2 a2) − (a4 + b4 + c4). Vậy
phương trình trên có hai nghiệm tan θ1 và tan θ2 phân biệt, âm

1 √ p
tan θ1, tan θ2 = {−(a2+b2+c2 )± 2 (a2 − b2)2 + (b2 − c2 )2 + (c2 − a2 )2},
4S
p
trong đó S = 14 (a + b + c)(a + b − c)(a − b + c)(−a + b + c).
Coi các tam giác cân A0BC, B 0CA, C 0AB có đỉnh cân theo thứ tự là
−−→ −−→ −−→ − →
B, C, A thì chúng minh theo đúng cách trên ta vẫn có AA0 + BB 0 + CC 0 = 0 ,

− −−→ −−→
ở đây f là phép quay (véctơ) góc −θ (ở đây 0 < θ < π), còn AA0 × BB 0 =
2 2 2
4S − 2S cos θ + a +b2 +c sin θ nên θ cần tìm phải là nghiệm của phương trình
lượng giác
α sin θ + β cos θ = γ
2 2 2
trong đó, α = a +b2 +c , β = −2S, γ = −4S. Gọi ϕ là góc mà sin ϕ =
√ β , cos ϕ = √ α thì sin(θ + ϕ) = √ γ2 2 = − √ 2 8S2 2 2 2 , cho
2
α +β2 2 2
α +β α +β 16S +(a +b +c )
hai nghiệm θ1, θ2 phân biệt.
Nếu coi các tam giác cân A0BC, B 0CA, C 0AB có đỉnh cân theo thự tự
là C, A, B thì cũng chứng minh theo cách tương tự, trong kết quả vừa rồi
thay θ bởi π − 2θ, ở đây (− π2 < θ < π2 , θ 6= 0).
Bài 4. Kí hiệu A là tập hợp các hình tròn của họ tiếp xúc 6 hình tròn
khác, B là tập các hình tròn còn lại.
Với mỗi hình tròn C0 của họ, kí hiệu L(C0 ) là tập giữa C0 và các hình
tròn C của họ mà có dãy C1 , C2 , ..., Cm hình tròn của họ (m ≥ 1) để Ci tiếp
xúc với Ci+1 (i = 0, 1, ..., n − 1), C = Cm .
22 Chương 2. Đáp án tuyển sinh

1) Trong mỗi L(C0 ) phải có C ∈ B vì nếu mọi C ∈ L(C0 ) đều thuộc A


thì xét hình tròn có bán kính bé nhất trong L(C0 ) suy ra mọi hình tròn của
(C0) phải có bán kính bằng bán kính đó (*) và khi đó lưới các tâm các hình
tròn của L(C0 ) không thể bị chặn, mâu thuẫn với họ hữu hạn hình tròn.
(*) suy ra từ: một hình tròn bán kính R không thể tiếp xúc ngoài
với sáu hình tròn ngoài nhau hay tiếp xúc ngoài nhau mà sáu bán kính
không nhỏ hơn R và có bán kính lớn hơn R. Thực vậy, nếu có hình vẽ bên:
O1 O2 = R1 + R2 + l, (R ≤ R1 , R1 ≤ R2 , 0 ≤ l) thì góc ϕ = (OO1 , OO2 )
phải giữa 60◦ và 180◦ do

(R + R1 )2 + (R + R2 )2 − (R1 + R2 + l)2 1
cos ϕ = ≤ .
2(R + R1 )(R + R2 ) 2
2) Nếu có hai cách đặt f, g thỏa mãn đề bài thì f − g lấy giá trị 0 trên
B và nó thỏa mãn điều kiện trung bình cộng của đề bài. Cần chứng minh
f − g lấy giá trị 0 trên A. Giả sử có Ce ∈ A mà (f − g)(C) e 6= 0; kí
hiệu C0 là hình tròn của họ sao cho (f − g)(C0) = min (f − g)(C) nếu
mọi C
e < 0 hay (f − g)(C0 ) = max (f − g)(C) nếu (f − g)(C)
)f − g)(C) e > 0,
mọi C
thì C0 ∈ A và theo 1), trong L(C0 ) có dãy C1, ..., Cm (m > 1), Ci tiếp
xúc Ci+1 (i = 0, 1, ..., m − 1), Ci ∈ A (i = 0, 1, ..., m − 1), Cm = C ∈ B.
Do tính chất trung bình cộng của các số lơn hơn hoặc bằng a (trong các
số nhỏ hơn hoặc bằng b) chỉ bằng a (theo thứ tự b) khi tất cả các số đó
bằng a (theo thứ tự b) và do tính chất min, max nói trên , với C0 suy ra
(f − g)(C0 ) = (f − g)(C1 ) = · · · = (f − g)(Cm−1 ) = (f − g)(C) = 0 vì
C ∈ B), do đó có mâu thuẫn.
Chú ý: sau khi xét trường hợp (f − g)(C) e < 0 có thể đưa trường hợp
e > 0 vè trường hợp nó âm nhờ xét g − f thay cho f − g.
(f − g)(C)
Bài 5. Dễ thấy rằng: Nếu (x, y) là nghiệm thì (y, x) là nghiệm. Nếu
không có nghiệm nguyên (x, x).
2) Xét ánh xạ f : R2 → R2 , f ((x, y)) = (x0 = 5x − y, y 0 = x)
a) dễ thấy rằng x02 − 5x0y 0 + y 02 = x2 − 5xy + y 2.
b) Kí hiệu S là tập các nghiệm nguyên dương thì dễ thấy rằng f (S) = S
2.1. Đáp án chọn đội tuyển năm học 1991 - 1992 23

(để ý rằng phương trình có thể viết y(5x − y) = x2 + 5).


c) Hãy chứng minh rằng với (x, y) ∈ S mà 1 < x < y thì f ((x, y)) =
(x , y ) có tính chất 1 ≤ x/ < x(= y/). Thực vậy, khi 1 < x < y = (x0, y 0 )
0 0

có tính chất 1 < x < y mà x, y nguyên thì (y − x)2 + 5 = 3xy ≥ 3.2.3 nên
(y − x)2 ≥ 13, vậy y − x ≥ 4. Khi đó nếu như 4x ≥ y thì 5xy = x2 + y 2 + 5 ≤
x2 + 4xy + 5, do đó x2 + 5 ≥ xy hay x(y − x) ≤ 5, mà y − x ≥ 4 nên x = 1,
trái với giả thiết x > 1. Vậy 4x < y, do đó x0 = 5x − y < x.
d) dễ thấy tập các nghiệm nguyên dạng (1, y) là S0 {(1, 3), (1, 2)}. vậy
từ c) và d) suy ra rằng nếu kí hiệu S1 = {(x, y) ∈ S | 1 < x < y}
thì f (S1 ) ⊂ S1 ∪ S0 và với mọi (x, y) ∈ S1 , có số nguyên dương k để
f k ((x, y)) ∈ S0.
3) Đổi vai trò x với y trong 2) suy ra ánh xạ g : R2 → R2 : g((x, y)) =
(x0 = y, y 0 = 5y − x) có tính chất g(S) ⊂ S và kí hiệu Se1 = {(x, y) ∈ S |
1 < y < x}, Se0 = {(2, 1), (3, 1)} thì g(Se1 ) ⊂ Se1 ∪ Se0, Se0 ⊂ S, và với mọi
(x, y) ∈ Se1 , có số nguyên dương k để g k ((x, y)) ∈ Se0.
4) Dễ thấy rằng g = f −1 , do đó f |s , g |s là song ánh và để ý rằng
f (S0 ) = S˜0, suy ra S = {f k ((1, 3)), f k ((1, 2)) | k ∈ Z}. Chú ý rằng với mọi
k ∈ Z, f k là song ánh và khi k 6= 1 thì f k không có điểm bất động do nếu
(x, y) ∈ S, f ((x, y)) = (x0 , y 0) thì x0 −y 0 = 5x−y −x = 4x−y = x−y +3x >
x − y, vậy các phần tử viết trên đây của S là đôi một khác nhau.
Có thể tính được (với mọi k ∈ Z)

(
f k ((1, 3)) = α1 (λk+1
1 − λk+1
2 + 3(λk2 − λk1 ), λk1 − λk2 + 3(λk−1
2 − λk−1
1 ))
f ((1, 2)) = α (λ1 − λ2 + 2(λ2 − λ1 ), λ1 − λ2 + 2(λ2 − λk−1
k 1 k+1 k+1 k k k k k−1
1 ))

trong đó λ1 = 12 (5 + α), λ2 = 12 (5 − α), α = 21. Vài nghiệm

f f f f f f
· · · 7→ (14, 67) 7→ (3, 14) 7→ (1, 3) 7→ (2, 1) 7→ (9, 2) 7→ (43, 9) 7→ · · ·
f f f f f f
· · · 7→ (9, 43) 7→ (2, 9) 7→ (1, 2) 7→ (3, 1) 7→ (14, 3) 7→ (67, 14) 7→ · · ·
Bài 6. Lập đồ thị G: đỉnh biểu diễn cho “ngôn ngữ", cạnh nối hai đỉnh
biểu diễn “người biết hai ngôn ngữ đó". Vậy G là đồ thị 2n đỉnh. Điều kiện
“hai người biết chung nhiều nhất một ngôn ngữ" nói rằng G là đồ thị đơn.
Điều kiện còn lại cho biết: với mỗi k nguyên 1 ≤ k ≤ n − 1 có không quá
k − 1 đỉnh, mỗi đỉnh có bậc nhỏ hơn hoặc bằng k (*).
Theo đề bài, cần chứng minh: từ tất cả các cạnh của G có thể ... 2n
cạnh thuộc 2n đỉnh và mỗi đỉnh luôn thuộc đúng hai cạnh ... 2n cạnh đó.
Để chứng minh điều này ta sẽ chứng minh:
Trong G tồn tại một đường đi khép kín có độ dài 2n và đi qua tất cả
các định của G (một đường đi như thế ta sẽ gọi là chu trình H. Ta chứng
24 Chương 2. Đáp án tuyển sinh

minh điều này bằng phản chứng. Giả sử trong G không có chu trình H.
Khi đó tập các đỉnh không kề nhau của G là không rỗng và hữu hạn. Bằng
cách thêm dần hai cạnh nối hai đỉnh không kề nhau ta sẽ xây dựng đồ thị
2n đỉnh G e thoả mãn 1) (*), 2) trong G e không có chu trình H 3) Khi thêm
cạnh nối hai đỉnh bất kì không kề nhau của G e ta sẽ nhận được đồ thị có
chu trình H.
Xét G e với v là đỉnh của G e kí hiệu f (v) là bậc của v.
a) Từ 2) và 3) suy ra giữa hai đỉnh bất kì không kề nhau của G̃ đều tồn
tại một đường đi nhận hai đỉnh ấy làm hai đầu mút, đi qua tất cả các đỉnh
của Ge và có độ dài 2n − 1.
b) Nếu hai đỉnh v và v 0 của G e có f (v) ≥ n, f (v 0 ) ≥ n thì v và v 0 phải kề
nhau.
Thật vậy, giả sử v và v 0 không kề nhau thì có đường đi v1, v2, ..., v2n
(v1 ≡ v, v2n ≡ v 0 đi qua tất cả các đỉnh của G e và có độ dài 2n − 1. Giả
sử f (v) = s ≥ n. Kí hiệu vi1 , vi2 , ..., vis (2 = i1 < i2 < · · · < is < 2n) là
các đỉnh kề với v1 ≡ v. Khi đó với mỗi j = 1, 2, ..., s các đỉnh v(ij )−1 không
kề với v2n ≡ v 0 vì nếu ngược lại thì chu trình H trong G e là v1v2...v(i )−1 v2n
j

v2n−1 ...vij mâu thuẫn với 2). Từ đó suy ra f (v 0) ≤ 2n − (s − 1) ≤ n − 1 (do


s ≥ n), mâu thuẫn với f (v 0) ≥ n. Vậy v, v 0 phải kề nhau.
c) Từ b) suy ra tập v gồm các đỉnh v của G e mà f (v) ≤ n − 1 là không
rỗng, vậy có maxv∈V f (v) = m ≤ n − 1. Lấy v1 mà f (v1) = m. Điều kiện (*)
với k = n − 1 nói rằng có ít nhất 2n − (n − 1) + 1 = n + 2 đỉnh có bậc ≥ n,
do với k = n − 1 nói rằng có ít nhất một trong các đỉnh này, chẳng hạn v2n ,
không kề với v1 . Suy ra có đường đi v1, v2, ...v2n đi qua tất cả các đỉnh của G e
và có độ dài 2n − 1. Kí hiệu vi1 , vi2 , ...vim (2 = i1 < i2 < · · · < im < 2n là các
đỉnh kề với v1 thì lập luận như ở b) chứng tỏ với mọi j = 1 → n ta có v(ij )−1
không kề với v2n (chú ý rằng điều kiện (*) với k = 1) chứng tỏ mọi đỉnh
của Ge có bậc ≥ 2. Áp dụng điều kiện (*) với k = m (2 ≤ m ≤ n − 1) suy
ra {v(i1 )−1 , v(i2)−1 , ..., v(im)−1 } phải có ít nhất một đỉnh vq có f (vq ) ≥ m + 1
. Từ định nghĩa của m suy ra f (vq ) ≥ n. Như vậy, vq , v2n có f (vq ) ≥ n,
f (v2n ) ≥ n mà không kề nhau, mâu thuẫn với b). Mâu thuẫn này cho ta
điều phải chứng minh.

2.2 Đáp án chọn đội tuyển năm học 1992 -


1993
Bài 1. Trường hợp tổng quát khi đặt 1993 = 2m + 1 (dòng) và 2000 = 2n
(cột) với m = 996 và n = 1000.
Gọi s là tổng số các hình vuông 2 × 2 và hình chữ nhật khuyết kép, gọi y
2.2. Đáp án chọn đội tuyển năm học 1992 - 1993 25

là số các hình chữ nhật khuyết đơn. Ta có đẳng thức về diện tích các hình:
4s + 5y = 2n(2m + 1) (1)

Đánh dấu (×) vào các ô vuông có toạ độ (2r, 2t) với 1 6 r 6 m và
1 6 t 6 n ta được m.n dấu (×). Dễ thấy trên hình:
+) Hình vuông 2 × 2 hoặc hình chữ nhật khuyết kép chứa đúng một dấu
(×).
+) Hình chữ nhật khuyết đơn chứa một hoặc hai dấu (×). Từ đó đặt
1 6 v 6 2, v nguyên, ta có bất đẳng thức sau về số dấu (×) trên hình
m.n = s.1 + y.v > s + y (2)
Từ (1) và (2) ta có 5m.n > 5(s + y) = 4s + 5y + s = 2n(2m + 1) + s hay
s 6 5m.n − 2n(2m + 1) = m.n − 2n = n(m − 2).
Áp dụng với m = 996 và n = 1000 có kết quả s 6 994000. Sự tồn tại
của cách ghép với s = 994000 cho trên hình vẽ.
26 Chương 2. Đáp án tuyển sinh

Bài 2. 1) Ta có
 1 3 √ 1
a3n+1 = an + √ = a3n + 3an an + 3 + √
an an an
√ 9 p 3
> a3n + 3an an + = ( a3n + )2
q 4 2
p 3 p 3n
⇒ a3n+1 > a3n + ⇒ a3n > ∀n > 1 (1)
2 2

2) Mặt khác
√ 1 2 1 √ 1 2
an+1 = an + − 2 < an +
2an 4an 2an

√ √ 1
⇒ an+1 < an +
2an
q p 3 3 1 1
⇒ a3n+1 < a3n + + . √ + 3
2 4 an an 8an
3n 3 X 1 1X 1
n−1 n−1
p
3
⇒ an < 1 + + √ + , ∀n > 1
2 4 k=0 ak ak 8 k=0 a3k

Từ (1) suy ra với n > 1 thì

X
n−1
1 X
n
1 1X 1
n
<1+ <1+ < K = const
a3k a3k 9 k3
k=0 k=1 k=1

v
u n
X
n−1
1 X
n
1 √ uX 1 √
√ < 1+ √ 6 1+ nt 3
< 1+K 0 n với K 0 = const
ak ak ak ak ak
k=0 k=1 k=1

Do vậy
p 3n √
a3n < + K1 + K2 . n ∀n > 1
2
ở đây K1 , K2 = const
3) Từ kết quả của 1) và 2) ta được:

3n 3 3n √
< an2 < + K1 + K2 . n ∀n > 1
2 2
3
an2 3
⇒ ∃ lim = (2)
n→∞ n 2
2.2. Đáp án chọn đội tuyển năm học 1992 - 1993 27

Từ (1) ta có limn→∞ an = +∞. Do đó, kết hợp với (2) suy ra

aαn 3
lim = +∞ với α >
n→∞ n 2
aα 3
lim n = 0 với α <
n→∞ n 2

Vậy
aαn 3
lim = L 6= 0 ⇔ α =
n→∞ n 2
.
Bài 3. 1) Tìm max A. Viết lại A dưới dạng

A = 5(x21 + x24) + 6(x22 + x23) − 8(x1 x2 + x2 x3 + x3 x4) + 2x1 x3 + 2x2 x4

Với mọi α > 0 ta có


16 2
−8x1x2 6 αx21 + x dấu ” = ” ⇔ αx1 = −4x2 (1)
α 2
−8x2 x3 6 4(x22 + x23) dấu ” = ” ⇔ x2 = −x3 (2)
16
−8x3x4 6 αx24 + x23 dấu ” = ” ⇔ αx4 = −4x3 (3)
α
α 2 4 2
2x1 x3 6 x1 + x3 dấu ” = ” ⇔ αx1 = 4x3 (4)
4 α
4 2 α 2
2x2 x4 6 x2 + x4 dấu ” = ” ⇔ αx4 = 4x2 (5)
α 4

Do vậy
 5α  2  20  2
A6 5+ (x1 + x24) + 10 + (x2 + x23)
4 α
Chọn α > 0 thoả mãn
5α 20 √
5+ = 10 + ⇔ α = 2(1 + 5
4 α

. Khi đó
5α 2 5α 5 √
A 6 (5 + )(x1 + x22 + x23 + x24) 6 5 + = (3 + 5)
4 4 2
√ P
Ta có A = 52 (3 + 5) ⇔ có đồng thời (1), (2), (3), (4), (5) và 4i=1 x2i = 1
s √ √
1 1 1+ 5 1+ 5
⇔ x1 = ± 1 − √ , x2 = − x1, x3 = x1, x4 = −x1
2 5 2 2
28 Chương 2. Đáp án tuyển sinh

Vậy
5 √
max A = (3 + 5)
2
2) Tìm min A: Lấy 1 < α < 2. Viết lại A dưới dạng

A = (x1 − αx2 + x3)2 + (x2 − αx3 + x4)2 + −2(4 − α)x1x2 − 4(2 − α)x2x3 −
− 2(4 − α)x3 x4 + 4(x21 + x24 ) + (5 − α2 )(x22 + x23 )

Do α < 2 nên ∀β > 0 ta có


 x2 
−2(4 − α)x1 x2 > −(4 − α) βx21 + 2 ” = ” ⇔ βx1 = x2 (6)
β
 x2 
−2(4 − α)x3 x4 > −(4 − α) βx24 + 3 ” = ” ⇔ βx4 = x3 (7)
β
−4(2 − α)x2x3 > −2(2 − α)(x22 + x23 ) ” = ” ⇔ x2 = x3 (8)

Suy ra

A > −2(4 − α)x1 x2 − 4(2 − α)x2x3 − 2(4 − α)x3 x4 +


+ 4(x21 + x24 ) + (5 − α2 )(x22 + x23 )
4−α
> [4 − (4 − α)β](x21 + x24 ) + [1 + 2α − − α2 ](x22 + x23)
β

Chọn β sao cho

4−α
4 − (4 − α)β = 1 + 2α − − α2 (9)
β

Khi đó
1
A > [4 − (4 − α)β](x21 + x22 + x23 + x24) > [4 − (4 − α)β]
2

Dấu ” = ” ở bất đẳng thức cuối cùng đạt được khi và chỉ khi có đồng thời


 βx1 = x2



 βx4 = x3


x = x
2 3
(I)

 x1 − αx2 + x3 = 0



 x2 − αx3 + x4 = 0


 2 1
x1 + x22 + x23 + x24 = 2
2.2. Đáp án chọn đội tuyển năm học 1992 - 1993 29

Dễ thấy, các giá trị x1, x2, x3 , x4 thoả mãn hệ trên là các giá trị có cùng dấu.
Bởi vậy, để đơn giản ta sẽ chỉ xét xi = 0, i = 1, 4. Với điều kiện đó ta có
 

 x = x , x = x 
 x3 = x2 , x 4 = x1


2 3 1 4 

 x1 = β1 x2  x1 = (α − 1)x2
(I) ⇔ ⇔

 x 1 = (α − 1)x 2 
 α − 1 = β1 (10)

 

2x2 + 2x2 = 1  x1 = √ 1
1 2 2 2 1+β 2

Bây giờ ta sẽ xác định β nhờ vào (9) và (10). Thế (10) vào (9) ta được
1  1 11 
4+ −3 β =2− 2 + −3
β β β β

2 1+ 5
⇔β − β − 1 = 0 ⇔ β = (do β > 0)
2
Từ đó √
1 1+ 5 1
x1 = p √ , x2 = .p √
10 + 2 5 2 10 + 2 5
Vậy √
7−3 5
min A =
4
đạt được khi

1 1+ 5
x1 = x4 = p √ , x 2 = x3 = p √
10 + 2 5 2 10 + 2 5

Bài 4. (Hình học phẳng)

Từ hai đẳng thức quen thuộc (với a = BC, b = CA, c = AB)


−−→ −→ − −→ −→
OH = OA + OB + OC (1)
30 Chương 2. Đáp án tuyển sinh



→ −→ −→ − →
aIA + bIB + cIC = 0 (2)
suy ra

→ −→ −
−→ −→
(a + b + c)OI = aOA + bOB + cOC (3)

−→ −→ −−→ −→
(a + b + c)IH = (b + c)OA + (c + a)OB + (a + b)OC (4)
−−→ −→ −→ −→ −→ − −→
Để ý rằng 2OB.OC = 2R2 − a2, 2OC .OA = 2R2 − b2, 2OA.OB = 2R2 − c2
với R là bán kính đường tròn ngoại tiếp tam giác ABC, từ (3) và (4) suy
ra
−→
(a + b + c)2OI 2 = (a + b + c)2 R2 − abc(a + b + c) (5)
−→2
(a + b + c)IH = 4(a + b + c)2 R2 − (a + b + c)(a3 + b3 + c3 + abc) (6)
abc
Gọi r là bán kính đường tròn nội tiếp 4ABC thì a+b+c = 2Rr nên từ (5)
và (6) có:
abc
OI 2 = R2 − = R(R − 2r)
a+b+c

(a3 + b3 + c3 − 3abc) + 4abc
IH 2 = 4R2 −
a+b+c
= 4R(R − 2r) − (a2 + b2 + c2 − bc − ca − ab)
1
= 4OI 2 − [(b − c)2 + (c − a)2 + (a − b)2 ]
2
Từ đó IH 6 2IO và dấu bằng xảy ra khi và chỉ khi a = b = c tức là 4ABC
là tam giác đều.
Bài 5. Ký hiệu p(n) là ước nguyên tố lớn nhất của n. Ký hiệu α(n) là
số mũ của p(n) trong phân tích nguyên tố của n.
1) Trước hết, chứng minh nhận xét sau:
Nhận xét: Nếu p(x0 ) > k thì tồn tại ít nhất một chỉ số m1 > 0 sao cho
p(xm1 ) < p(x0 ).
Chứng minh: Giả sử x0 = pα1 1 pα2 2 · · · pαr r với p1 < p2 < · · · < pr là tất cả
các ước nguyên tố của x0 . Khi đó.

x1 = f (x0 ) = kpα1 1 −1 pα2 2 −1 · · · pαr r −1 (p1 − 1)(p2 − 1) . . . (pr − 1) (1)

Vì pr > k > 1 nên từ (1) suy ra:


+) Nếu α(x0) = αr > 1 thì p(x1 ) = p(x0) = pr và α(x1 ) = α(x0) − 1
+) Nếu α(x0 ) = αr = 1 thì p(x1 ) < p(x0 ) = pr Từ đó suy ra p(xαr ) <
p(x0 ). Nhận xét được chứng minh.
2.2. Đáp án chọn đội tuyển năm học 1992 - 1993 31

2) Từ nhận xét trên suy ra nếu p(x0) > k thì sẽ tồn tại chỉ số M sao
cho p(xM ) 6 k. Như thế, ∀a > 1 luôn ∃M > 0 sao cho p(xM ) 6 k. Do đó
+) Với k = 2 thì p(xM ) = 2 suy ra xM có dạng xM = 2α , α ∈ N∗ hay
xM +1 = f (xM ) = 2α . Suy ra xm = xM , ∀m > M, vậy dãy {xM }∞
0 là dãy bị
chặn ∀a > 1.
+) Với k = 3 thì p(xM ) = 2 hoặc p(xM ) = 3. Do vậy, xM sẽ có một
trong các dạng sau:

∗) xM = 2α .3β với α, β > 1


β
∗) xM = 3 với β > 1
∗) xM = 2α với α > 1

- Xét xM = 2α .3β . Khi đó xM +1 = f (xM ) = 2α .3β ⇒ xm = xM , ∀m > M


- Xét xM = 3β . Khi đó xM +1 = f (xM ) = 2.3β ⇒ xm = xM +1 , ∀m >
M + 1 (theo kết quả của bước trên).
- Xét xM = 2α . Khi đó
(
3 nếu α = 1
xM +1 = f (xM ) =
3.2α−1 nếu α > 1

Từ đó, theo kết quả của hai bước trên, ta sẽ có xm = xM +2, ∀m > M + 2
(trường hợp α = 1) hoặc xm = xM +1, ∀m > M + 1 (trường hợp α > 1).
Tóm lại, với k = 3 thì dãy {xm }∞
0 là dãy bị chặn ∀a > 1.
Xét k > 3. Với chú ý rằng: ∀m ∈ N, ∀k > 1 nếu p(xm ) 6 k thì p(xm+1 ) 6
k (dễ thấy từ cách xác định dãy {xm }∞0 ), ta thấy p(xm ) 6 k, ∀m > M. Do
đó, ∀m > M ta có:
1 1
xm+1 = f (xm ) = kxm (1 − ) · · · (1 − )
p1 pr
1 1 1
> kxm (1 − )(1 − ) · · · (1 − ) (do pr 6 k và k > 3)
2 3 k
= xm . (2)

Mà dãy {xm }∞ 0 là dãy các số nguyên dương, nên từ (2) suy ra dãy {xm }0

là dãy không bị chặn ∀a > 1.


Vậy dãy {xm }∞ 0 là dãy bị chặn ∀a > 1 khi và chỉ khi k = 2 hoặc k = 3.
Bài 6. Xét n điểm A1, A2, . . . , An mà có thể tô màu tất cả các đoạn
Ai Aj thoả mãn đề bài. Xét grafG có tập đỉnh V = {A1, A2, . . . , An } và tập
cạnh là các đoạn được tô màu xanh. Dễ thấy G đơn, vô hướng, n đỉnh và
thoả mãn:
a) d(Ai ) 6 4, ∀i = 1, n (d(Ai ) ký hiệu bậc của đỉnh Ai).
32 Chương 2. Đáp án tuyển sinh

b) Với bất cứ hai đỉnh Ai , Aj nào cũng đều tồn tại một xích đơn nối
chúng và có độ dài nhỏ hơn hoặc bằng 2.
Vấn đề đặt ra ở bài đã ra tương đương với tìm số đỉnh n lớn nhất của
grafG đơn, vô hướng và thoả mãn a) và b).
Xét G đơn, vô hướng, n đỉnh và thoả mãn a) và b). Xét một đỉnh Ai
bất kỳ của G. Khi đó mỗi đỉnh trong số n − 1 đỉnh còn lại phải hoặc kề với
Ai hoặc kề với ít nhất một đỉnh kề với Ai (theo b)). Kết hợp với a) suy ra
n 6 1 + 4 + 3 × 4 = 17.
1) Xét n=17: Khi đó dễ thấy, phải có

d(Ai ) = 4 ∀i = 1, 17 (*)
4×17
và do đó G có tất cả 2
= 34 cạnh.

Hình 1
Xét đỉnh Ai bất kỳ của G. Từ (*) suy ra Ai kề với đúng 4 đỉnh khác,
giả sử là Ai1 , Ai2 , Ai3 , Ai4 . Qui ước gọi tất cả các đỉnh còn lại của G là các
đỉnh rìa, và gọi tất cả các cạnh có cả hai đầu mút là hai đỉnh rìa là các cạnh
−→
rìa. Từ b) và (*) suy ra mỗi đỉnh Aij , (j = 1, 4) (xem H.1). Từ đó dễ thấy
không có hai đỉnh nào của G cùng với Ai lập thành nhóm ba đỉnh đôi một
kề nhau, nên trong G không có ba đỉnh nào đôi một kề nhau (vì Ai lấy ra
xét là đỉnh bất kỳ). Vậy mỗi cạnh rìa đều có hai đầu mút là hai đỉnh rìa
không cùng kề với một đỉnh Aij suy ra mỗi cạnh rìa cho ta một chu trình
đơn độ dài 5 và đi qua Ai . Mà số cạnh rìa có tất cả 34 − 16 = 18, nên có
tất cả 18 chu trình đơn độ dài 5 và đi qua Ai. Vì Ai là đỉnh bất kỳ nên từ
đó suy ra số chu trình đơn độ dài 5 trong G có tất cả là: 18×17 5
/ Z, vô lý.

Vậy n 6= 17.
2) Xét n=16. Khi đó dễ thấy, phải có

d(Ai ) = 4 ∀i = 1, 16 (1)

và do đó G có tất cả 16×4
2
= 32 cạnh. Xét một đỉnh Ai bất kỳ của G. Theo
(1), Ai kề với đúng 4 đỉnh khác, giả sử là Ai1 , Ai2 , Ai3 , Ai4 . Tiếp tục, bằng
2.2. Đáp án chọn đội tuyển năm học 1992 - 1993 33

phương pháp lập luận như ở 1), ta sẽ được: mỗi đỉnh Aij , (j = 1, 4) đều kề
với đúng ba đỉnh rìa và có đúng một đỉnh rìa, tạm gọi là Ak , kề với đúng
hai đỉnh Aij (xem H.2). Từ đó, do Ai là bất kỳ nên suy ra: Trong G không
có ba đỉnh nào đôi một kề nhau, suy ra mỗi cạnh rìa không liên thuộc Ak
cho ta đúng một chu trình đơn độ dài 5 và đi qua Ai, còn mỗi cạnh rìa liên
thuộc Ak cho ta đúng hai chu trình đơn độ dài 5 và đi qua Ai . Mà số cạnh
rìa có tất cả là 32 − 16 = 16 và trong số này có đúng hai cạnh liên thuộc
Ak (do d(Ak ) = 4), nên suy ra có tất cả 14 × 1 + 2 × 2 = 18 chu trình đơn
độ dài 5 đi qua Ai . Vì Ai bất kỳ nên suy ra số chu trình đơn độ dài 5 trong
G có tất cả là 18×16
5
/ Z, vô lý. Vậy n 6= 16.

Hình 2
3) Xét n = 15. Ta có G được mô tả ở (H.3) thoả mãn mọi yêu cầu đặt
ra.

Hình 3
Vậy nmax = 15.
Bình luận:
1) Việc xây dựng G có 15 đỉnh xuất phát từ graf quen thuộc (graf
Petersơn) (H.4) và bởi vậy G còn có thể mô tả như sau (H.5).
34 Chương 2. Đáp án tuyển sinh

2) Có thể xét trường hợp n = 16 bằng cách khác dễ lập luận chặt chẽ
hơn. Tuy nhiên việc xét như đã trình bày ở trên đảm bảo cho lời giải nhất
quán về phương pháp.

Hình 4

Hình 5
3) Lời giải trên chỉ dùng cho người chấm thi.

2.3 Đáp án chọn đội tuyển năm học 1993 -


1994
Bài 1. a) Đặt BE = x, AB = CD = a, CF = y, AD = BC = b. Từ
SABE = SBCF có

1 1
.AB.BE sin α = .BC.CF sin(1800 − α)
2 2
⇔ AB.BE = BC.CF
⇔ a.x = b.y (1)

b) Kẻ DB//AE cắt tia BC tại P thì CP = BE = x. Kẻ BQ//AF cắt tia


2.3. Đáp án chọn đội tuyển năm học 1993 - 1994 35

DC tại Q thì CQ = F Q − CF = a − y. Theo định lý Talet, ta có:


BM BE x
= = (2)
BD BP b+x
DN DF a−y
= = (3)
DB DQ 2a − y

Từ đó tính

MN 2 − BM 2 − DN 2 = (BD − BM − DN )2 − BM 2 − DN 2
= BD2 − 2BD.BM − 2BD.DN + 2BM.DN

hay
MN 2 − BM 2 − DN 2 BM DN BM DN
= 1 − 2 − 2 + 2 . (4)
BD2 BD BD BD BD
Thay (2),(3) vào (4) ta được
MN 2 − BM 2 − DN 2 2x 2(a − y) x a−y
= 1 − − + 2 =
BD2 b+x 2a − y b + x 2a − y
1
= [(b + x)(2a − y) − 2x(2a − y)−
(b + x)(2a − y)
− 2(a − y)(b + x) + 2x(a − y)]
(2a − y)(b − x) − 2(a − y)b yb + yx − 2ax
= = (5)
(b + x)(2a − y) (b + x)(2a − y)

Thay (1) vào tử số của (5) được

yb + yx − 2ax = ax + yx − 2ax = yx − ax = −x(a − y)

Vậy
MN 2 − BM 2 − DN 2  x  a − y  BM DN
2
= − =− .
BD b + x 2a − y BD BD
hay
MN 2 = BM 2 + DN 2 − BM.DN (6)
36 Chương 2. Đáp án tuyển sinh

Nếu dựng 4M 0 N 0 S sao cho SM 0 = BM, SN 0 = DN và góc đối diện với


M 0 N 0 bằng 600 thì

M 0 N 02 = SM 02 + SN 02 − SM 0 .SN 0 (7)

So sánh (6) và (7) ta có MN = M 0 N 0 , nghĩa là tồn tại 4AM 0 N 0 có các


cạnh tương ứng bằng MN, BM, DN . Gọi R là bán kính đường tròn ngoại
tiếp 4SM 0 N 0. Theo định lý hàm số sin ta có

MN = M 0 N 0 = 2R sin 600 = 3R

nên MN 0 và R cùng giảm.


Bài 1. a) Xét biểu thức

BM 2 + DN 2 − MN 2
ϕ=
2BM.DN
chỉ cần chứng minh −1 6 ϕ 6 1 (ϕ đó sẽ là cosin của góc α đối diện cạnh
của tam giác mà ta cần chứng minh sự tồn tại). Kẻ MM1 , NN1 song song
DA (M1 , N1 thuộc AB) thì do Talet

BM12 + AN22 − M1 N12


ϕ=
2BM1 .AN1

Với mỗi điểm P trong hình bình hành ABCD, kẻ P P1 song song DA (P1 ∈
AB), kẻ P P2 song song AB (P2 ∈ AD), gọi x(P ) = AP
AB
1
, y(P ) = AP
AD
2
.

Giả thiết cho


BE FC
= =t 0<t<1
AD AB

P thuộc đường chéo BD ⇔ x(P ) + y(P ) = 1 (*)


P thuộc AE ⇔ y(P ) = tx(P ) (**)
1
P thuộc AF ⇔ y(P ) = x(P ) (***)
1−t
2.3. Đáp án chọn đội tuyển năm học 1993 - 1994 37

Từ (*), (**) suy ra

AM1 1
x(M1) = =
AB 1+t

Từ (*), (***) suy ra

AN1 1−t
x(N1 ) = =
AB 2−t

Từ đó
M1 B t AN1 1−t
= 1 − x(M1) = , = x(N1 ) =
AB 1+t AB 2−t

M1 N1 t2 − t + 1
= x(M1) − x(N1 ) =
AB (1 + t)(2 − t)
Vậy
 2  2  2
t 1−t t2 −t+1
1+t
+ 2−t
− (1+t)(2−t)
ϕ= t(1−t)
2 (1+t)(2−t)

dễ tính ϕ = 12
Vậy cos α = 12 .
b) Bán kính R đường tròn ngoại tiếp tam giác vừa xây dựng xác định
bởi
MN 1
R= = √ MN
2 sin α 3
nên khi MN càng nhỏ thì R càng nhỏ.
Bài 2. a)

x2 + y 2 + z 2 + t2 − Nxyzt − N = 0 (1)
⇔ t(t − Nxyzt) = N − (x2 + y 2 + z 2 ) (2)

Với ba số nguyên dương bất kỳ a, b, c và N = a2 + b2 + c2 thì dễ thấy phương


trình (2) có nghiệm

x0 = a, y0 = b, z0 = c, t0 = Nabc = (a2 + b2 + c2)abc (*)

Chú ý rằng khi hoán vị bốn số a, b, c, Nabc ta lại được nghiệm (x1, y1 , z1, t1)
của phương trình (1).
b) Giả sử phương trình (1) có nghiệm nguyên dương, chọn (x0 , y0, z0, t0 )
là nghiệm nguyên dương của (1) sao cho tổng x0 + y0 + z0 + t0 là số nguyên
38 Chương 2. Đáp án tuyển sinh

dương nhỏ nhất. Không làm mất tính chất tổng quát, giả định rằng x0 6
y0 6 z0 6 t0.
Ta sẽ chứng minh rằng với N > 7 thì nghiệm nguyên dương của phương
trình (1) với x0 6 y0 6 z0 6 t0 nếu có phải có dạng (*) như trên.
Theo giả thiết t0 là nghiệm của phương trình bậc hai

t2 − Nx0 y0z0 t + x20 + y02 + z02 − N = 0 (3)

Phương trình (3) có nghiệm thứ hai t1 thoả mãn:

t1 + t0 = Nx0y0 z0 (4)
t1.t0 = x20 + y02 + z02 − N (5)

Từ (4) suy ra t1 ∈ Z. Lại theo giả thiết

N (1 + x0y0 z0t1) = t21 + x20 + y02 + z02 > 0

nên
1
t1 > −
x0 y0z0
Vì t1 ∈ Z nên t1 > 0.
Giả sử t1 > 0 khi đó (x0, y0 , z0, t1) là nghiệm nguyên dương của (1). Do
cách chọn (x0, y0 , z0, t0) thì x0 + y0 + z0 + t1 > x0 + y0 + z0 + t0 nên t1 > t0 .
Từ đó theo (5) ta có

t20 6 t1 t0 = x20 + y02 + z02 − N < x20 + y02 + z02 6 3z02

Ta có

N (1+x0y0 z02 ) 6 N (1+x0 y0z0t0 ) = x20 +y02 +z02 +t20 6 z02 +z02 +z02 +3z02 = 6z02

Từ đó, do N > 7 nên N (1 + x0 y0z02 ) 6 6z02 < N z02 suy ra 1 + x0 y0z02 < z02.
Điều vô lý này chứng tỏ t1 > 0 là sai, suy ra t1 = 0. Từ (4), (5) suy ra

N = x20 + y02 + z02 và t0 = Nx0 y0z0

là nghiệm (*) của phương trình (1).


Với N = 4k (8m + 7) > 7, áp dụng kết quả trên thì N = x2 + y 2 + z 2. Do
đó nếu chứng minh được phương trình x2 + y 2 + z 2 = 4k (8m + 7) không có
nghiệm nguyên dương thì phương trình (1) cũng không có nghiệm nguyên
dương.
+) Khi k = 0 ta có x2 + y 2 + z 2 = 8m + 7 hay x2 + y 2 + z 2 ≡ 7 (mod 8).
Trong ba số x, y, z phải có một số lẻ hoặc cả ba số lẻ. Nếu số a lẻ thì a2 ≡ 1
(mod 8), do đó x2 + y 2 + z 2 6= 7 (mod 8).
2.3. Đáp án chọn đội tuyển năm học 1993 - 1994 39

+) Khi k > 0 ta có
x2 + y 2 + z 2 = 4k (8m + 7) (*)
hay x2 +y 2 +z 2 ≡ 0 (mod 4). Trong ba số x, y, z phải có một số chẵn hoặc ba
số chẵn. Nếu có một số chẵn, còn hai số a, b lẻ thì a2 +b2 ≡ 2 (mod 4)), suy ra
x2 +y 2 +z 2 6= 7 (mod 8). Nếu x, y, z đều chẵn, đặt x = 2x1, y = 2y1 , z = 2z1
thì (*) tương đương với x2 + y 2 + z 2 = 4k−1 (8m + 7). Sau k lần biến đổi
như thế ta có x2 + y 2 + z 2 = 8m + 7, nhưng phương trình này vô nghiệm
nguyên dương như khi xét k = 0
Bài 3.a)
1 − 4x  1 − 4x  0
P (x) + 1 − P (x) − P 00(x) = 0 (1)
x2 x2
1 − 4x
⇔ (P (x) − P 0 (x)) + (P 0 (x) − P 00(x)) = 0 (2)
x2
Đặt Q(x) = P (x) − P 0 (x) thì Q0(x) = P 0(x) − P 00(x) nên
1 − 4x
(2) ⇔ 2
Q(x) + Q0(x) = 0 (3)
x
b) Ta chứng minh nếu đa thức bậc bốn P (x) có bốn nghiệm dương thì
đa thức bậc bốn Q(x) = P (x) − P 0 (x) cũng có bốn nghiệm dương. Không
mất tính chất tổng quát, giả định rằng hệ số bậc cao nhất của P (x) là 1.Đặt
P (x) = x4 − ax3 + bx2 − cx + d = (x − x1)(x − x2)(x − x3)(x − x4)
với x1, x2 , x3, x4 là các nghiệm dương. Từ đó theo Định lý Viet thì a, b, c, d >
0.
P 0 (x) = (x − x2 )(x − x3 )(x − x4 ) + (x − x1 )(x − x3 )(x − x4 )+
+(x − x1 )(x − x2 )(x − x4 ) + (x − x1 )(x − x2 )(x − x3 )

Q(x) = P (x) − P 0 (x) = x4 + a1x3 + b1x2 + c1 x + c + d


Vì Q(x1)Q(x2) < 0, Q(x2)Q(x3) < 0, Q(x3)Q(x4) < 0 nên Q(x) có ba
nghiệm dương là y1, y2 , y3. Gọi nghiệm thứ tư là y4 thì y1 y2y3 y4 = c + d > 0
nên y4 > 0. Vậy Q(x) có bốn nghiệm dương.
c) Đặt R(t) = t4 Q( 1t ). Vì Q(x) có bốn nghiệm dương thì R(t) cũng có
bốn nghiệm dương, do đó lại áp dụng kết quả trên, đa thức R(t) − R0 (t)
cũng có bốn nghiệm dương.
1 h 1 t4 1 i
R(t) − R0 (t) = t4Q( ) − 4t3 Q( ) − 2 Q0( )
t t t t
1 1
= (t4 − 4t3 )Q( ) + t2 Q0( )
t t
40 Chương 2. Đáp án tuyển sinh

Hay phương trình sau có bốn nghiệm dương

1 1
(t4 − 4t)Q( ) + Q0 ( ) = 0 (4)
t t
1
Đặt x = t
thì phương trình (4) trở thành phương trình (3), nên phương
trình (3)
1 − 4x
Q(x) + Q0(x) = 0
x2
cũng có bốn nghiệm dương.
Bài 3. Đặt P1 (x) = e−x P (x). Vì đa thức P (x) có bốn nghiệm dương
nên phương trình P1 (x) = 0 có bốn nghiệm dương. Suy ra, phương trình

P10 (x) = 0 (1)

có ba nghiệm dương. Ta có:

(1) ⇔ e−x [P (x) − P 0 (x)] = 0 ⇔ P (x) − P 0(x) = 0

Như vậy, đa thức Q(x) = P (x) − P 0 (x) có ba nghiệm dương, giả sử là


x1, x2 , x3. Tuy nhiên, do degQ(x) = 4 (vì degP (x) = 4) nên Q(x) còn có
nghiệm thực thứ tư x4 .
Vì đa thức bậc bốn P (x) có bốn nghiệm dương nên không mất tổng
quát, có thể coi P (x) có dạng

P (x) = ax4 − bx3 + cx2 − dx + e, với a, b, c, d, e > 0

Từ đó
Q(x) = ax4 + · · · + (−d − 2c)x + (e + d)
Suy ra (theo Định lý Viet)

e+d
x1 x2 x3 x4 = >0 ⇒ x4 > 0
4

Vậy Q(x) có bốn nghiệm dương.


Xét đa thức (biến t):
1
R(t) = t4 Q( )
t
. Dễ thấy degR(t) = 4, R(t) có bốn nghiệm dương. Do đó, theo kết quả
phần trên ta có phương trình

R(t) − R0 (t) = 0 (2)


2.3. Đáp án chọn đội tuyển năm học 1993 - 1994 41

có bốn nghiệm dương. Ta có


1 1 1 1
(2) ⇔ t4[P ( ) − P 0 ( )] − {t4[P ( ) − P 0 ( )]}0 = 0
t t t t
1 1 1 1 1 1 1 1
⇔ t4[P ( ) − P 0 ( )] − {4t3[P ( ) − P 0 ( )] + t4[− 2 P 0 ( ) + 2 P 00 ( )]}
t t t t t t t t
1 1 1
⇔ (t2 − 4t)P ( ) + (1 + 4t − t2)P 0 ( ) − P 00 ( ) = 0 (3)
t t t
1
Đặt t
= x. Từ phương trình (3) ta có phương trình (ẩn x):
1 − 4x  1 − 4x  0
P (x) + 1 − P (x) − P 00(x) = 0 (4)
x2 x2
Do (3) có bốn nghiệm dương nên phương trình (4) có bốn nghiệm dương.
(Đpcm).
Bài 4. Lấy tâm O của tam giác đều ABC làm gốc của mặt phẳng phức
2iπ 4iπ
C, coi A, B, C có toạ vị a, b, c thì b = ae 3 , c = ae 3 . Nếu gọi M có toạ vị
z0 thì A0 , B 0, C 0 có toạ vị a0 = 2z0 − a, b0 = 2z0 − b, c0 = 2z0 − c.

Gọi P là điểm mà B 0AP là "tam giác" đều định hướng thuận (có thể

suy biến thành 1 điểm nếu B 0 ≡ A) tức P có toạ vị p0 mà p−b0 = e 3 (a−b0).
−2iπ iπ −iπ −2iπ iπ
Vậy với chú ý b = ce 3 , a + b + c = 0, và 1 − e 3 = e 3 , e 3 − e 3 =

e 3 (eiπ − 1) = 0 ta có
iπ iπ
p = b0 + e 3 (a − b0) = 2z0 + b + e 3 (a − 2z0 + b)
−2iπ iπ
= 2z0 + ce 3 − e 3 (c + 2z0)
iπ −2iπ iπ
= 2z0 (1 − e 3 ) + c(e 3 −e3 )
−iπ
= 2z0 e 3

Vì kết quả không phụ thuộc a, b, c nên bằng cách hoán vị vòng quanh
A, B, C được C 0BP, A0CP cũng là tam giác đều định hướng thuận. Vậy P
cách đều B 0 và A, C 0 và B, A0 và C.
42 Chương 2. Đáp án tuyển sinh

−→ − →
Nếu còn có Q 6= P cách đều các cặp điểm đó thì P Q 6= 0 trực giao với
−−→0 −−→0 −−→0
AB , BC , CA khi đó ba véctơ này cùng phương tức là O cùng các điểm có
toạ vị b0 − a = 2z0 + c, c0 − b = 2z0 + a, a0 − c = 2z0 + b phải thẳng hàng,
2iπ 4iπ
khi đó O và các điểm có toạ vị a − b = a(1 − e 3 ), a − c = a(1 − e 3 ) phải
thẳng hàng là điều vô lý.
−iπ
b) Xét biểu thức đồng dạng f xác định bởi z0 7→ f (z0 ) = p = 2z0 e 3
(tích quay góc − π3 với vị trí tỷ số 2 cùng tâm O). Trung điểm D của AB có
−iπ −iπ 4iπ −iπ
toạ vị a+b2
nên f (D) có toạ vị (a + b)e 3 = −ce 3 = −ae 3 e 3 = a, vậy
f (D) = A. Vậy có biến đổi đồng dạng f, f(0) = 0, f(D) = A, f (M) = P , từ
đó f biến đường thẳng DM(M 6= D) thành đường thẳng AP . Hai đường
thẳng này phải cắt nhau tại một điểm N mà có một véctơ chỉ phương đường
thẳng DM tạo với một véctơ chỉ phương đường thẳng AP tạo thành góc
− π3 , tức góc định hướng giữa hai đường thẳng đó là − π3 hay 2π 3
. Khi M ≡ O
thì P ≡ O, MNP không tạo thành tam giác; Khi M 6= O, góc định hướng
giữa hai đường thẳng OM, OP cũng là − π3 hay 2π 3
, nên tứ giác MNP O nội
tiếp: đường tròn ngoại tiếp MNP qua tâm O của ABC. (Điều vừa chứng
minh đúng cho mọi đồng dạng thuận)
0
Bài 4. Qua phép đối xứng tâm SM = R180 M thì

G → G0
4ABC → A0 B 0C 0

Qua phép quay


0
R120
G0 : 4A0 B 0C 0 → 4B 0C 0A0
trong đó G, G0 là trọng tâm 4ABC, 4A0B 0C 0
0 0
R180 R120
G0
0 0 0
4ABC M
−−−→ 4A B C −−−→ 4B 0C 0 A0

Tích hai phép quay có thể là phép tịnh tiến khi tổng của hai góc quay
là bội của 1800 , có thể là phép quay trong trường hợp còn lại. Ở đây tổng
2.3. Đáp án chọn đội tuyển năm học 1993 - 1994 43

hai góc quay là 1800 + 1200 = 3000 ≡ −600 (mod 3600 ) nên tích là phép
0 0 0
1800
quay tâm P xác định R120
G0 .RM = R−60
P . Qua phép quay R−60
P thì

4ABC → 4B 0C 0A0

G → G0
nên tồn tại duy nhất điểm P cách đều A và B 0,cách đều B và C 0, cách đều
C và A0 .
Từ đó P G = P G0 và (P G, P G0 ) = −600 . Mặt khác SM : G → G0 nên M
là trung điểm của GG0 . Vậy 4P GG0 là tam giác đều và GM⊥MP, GM =
MP
2
.
1
(GA,GD)
b) Xét phép đồng dạng S là phép vị tự quay S = RG .HG2 thì

S = 4GP A → 4GMD

trong đó (GA, GD) = 600 . Vì đường thẳng AP biến thành đường thẳng
DM nên góc MNP\ = 600 hoặc MNP \ = 1200 tuỳ theo góc MNP
\ là nhọn
\
hoặc tù. Vì P \ = 600 là góc nhọn thì G và N đều
GM = 600 nên nếu MNP
0
nhìn đoạn P M dưới một góc 60 suy ra G, N, M, P cùng thuộc một đường
\ = 1200 là góc tù thì MNP
tròn, nếu MNP \ + MGP \ = 1200 + 600 = 1800
nên tứ giác GMNP là tứ giác nội tiếp trong một đường tròn. Đường tròn
này luôn đi qua điểm G là trọng tâm 4ABC, nên G là điểm cố định.
Cách khác giải câu a: Dựng tam giác đều AB 0P ngược hướng với tam
giác đều ABC. Ta chứng minh rằng các 4BC 0P, 4CA0 P cũng là tam giác
đều và cùng hướng với 4AB 0P . Thật vậy
−−→0 −−0→ −→ −−→0 −→ −→ −
−→
BC = B C = CA + AB = f (BA) + f (AB) = f (BP )

trong đó f là phép quay véctơ của mặt phẳng, góc +600 . Chứng minh tương
−−→ −→
tự ta cũng có CA0 = f (CP ). Bởi vậy 4BC 0P và 4CA0P là các tam giác
đều, cùng hướng với 4AB 0P , do đó trung trực của AB 0, BC 0, CA0 đồng quy
ở tâm quay P , góc quay bằng 600 , biến 4ABC thành 4B 0C 0A0 .
Bài 5.
√ √ √
f ( 2x) + f ((4 + 3 2)x) = 2f ((2 + 2)x) ∀x (1)

a) Với x = 0 thì ta có 2f (0) √


= 2f (0), vậy f (0) = a là hằng số tuỳ ý.
b) Với x > 0 ta đặt (2 + 2)x = t hay x = 2+t√2 . Lúc đó (1) trở thành

 √2   4 + 3√ 2 
f √ t +f √ t = 2f (t) ∀t > 0 (2)
2+ 2 2+ 2
44 Chương 2. Đáp án tuyển sinh

2

Ta chú ý rằng √ = √1 = 2 − 1 và
2+ 2 2+1
√ √ √ √
2 4+3 2 4 2+6 4+3 2 1
√ . √ = √ =1⇒ √ =√ .
2+ 2 2+ 2 6+4 2 2+ 2 2−1

Ta đặt t = ( 2 − 1)u thì (2) trở thành
√  √  √ 
f ( 2 − 1)u+1 + f ( 2 − 1)u−1 = 2f ( 2 − 1)u ∀u (3)
√ 
u
Lại đặt f ( 2 − 1) = g(u) thì (3) trở thành

g(u + 1) + g(u − 1) = 2g(u) ∀u (4)


⇔g(u + 1) − g(u) = g(u) − g(u − 1) ∀u

Đặt g(u + 1) − g(u) = h(u) thì h(u + 1) = h(u) ∀u. Bằng quy nạp dễ thấy

g(u + n) = nh(u) + g(u)

Vậy
(
h(u) + k(u) với 0 6 u < 1
g(u) = ∀n ∈ Z (5)
nh(u) + k(u − n) với n 6 u < n + 1

trong đó, k(u), h(u) là các hàm tuỳ ý, h(u) tuần hoàn chu kỳ 1. Thay lại
theo biến số x > 0 ta được
 
f (x) = g log√2−1 x với x > 0

trong đó, g(u) được xác định theo


√ (5). √
c) Với x < 0 ta đặt −(2 + 2)x = t = ( 2 − 1)u ta có
 
f (x) = g log√2−1 |x| với x < 0

Tóm lại (
a tuỳ ý  khi x = 0
f (x) =
g log√2−1 |x| khi x 6= 0

còn g(u) được xác định theo (5).


Bài 6. Xét tập A gồm tất cả có bộ thứ tự (a1, a2, . . . , a1994, . . . , a1993+1994)
thoả mãn đồng thời các điều kiện sau:
1) ai ∈ {0, 1} ∀i = 1, 1993 + 1994
2) Số 1 có mặt đúng 1994 lần trong mỗi bộ.
2.4. Đáp án chọn đội tuyển năm học 1994 - 1995 45

Xét phân hoạch


[
A= A(n1 ,n2 ,...,n1994 )

ở đây
+) Hợp lấy theo tất cả các bộ có thứ tự các số tự nhiên (n1 , n2 , . . . , n1994)
thoả mãn n1 + 2n2 + · · · + 1994n1994 = 1994.
+) A(n1 ,n2 ,...,n1994 ) là tập gồm tất cả các bộ có thứ tự (a1, a2, . . . , a1993+1994) ∈
A và thoả mãn điều kiện là trong mỗi bộ có đúng nk nhóm k ∀k = 1, 1994.
(Nhóm k được định nghĩa là nhóm gồm đúng k số 1 đứng liên tiếp trong bộ,
nói khác đi là nhóm có 1 trong các dạng sau (1| .{z . . 1} 0; 0 |1 .{z
. . 1} 0; 0 |1 .{z
. . 1})).
ksố1 ksố1 ksố1

1993
CardA = C1993+1994

1994!
CardA(n1 ,n2 ,...,n1994 ) =
n1 !n2! . . . n1994!(1994 − n1 − · · · − n1994)!
1994!
=
n1 !n2! . . . n1994!(n2 + 2n3 + · · · + 1993n1994 )!


X
CardA = CardA(n1 ,n2 ,...,n1994 ) .

Nên suy ra:


1
T = C 1993
1994! 1993+1994
.

2.4 Đáp án chọn đội tuyển năm học 1994 -


1995
Bài 1. Cho tam giác ABC với mỗi điểm M, gọi khoảng cách đại số từ M
đến đường thẳng BC là ± (khoảng cách thông thường từ M đến BC), lấy
dấu + hay − tuỳ theo M cùng phía hay khác phía với A đối với BC (tất
nhiên M thuộc BC thì khoảng cách đó bằng không). Tương tự cho khoảng
cách đại số từ M đến CA, AB.
1) Xét các đường tròn (AB1C1), (AB2 C2) như trong đề bài. Hãy chứng
minh trục đẳng phương của cặp đường tròn đó là quỹ tích các điểm M mà
các khoảng cách đại số từ M đến AB và đến CA tỉ lệ với γ vàβ.
46 Chương 2. Đáp án tuyển sinh

−→ −→
Thực vậy, lấy hệ toạ độ vuông góc Oxy mà O ≡ A, B ∈ Ox+ , (AB, AC) =
−→ −→
ϕ, 0 < ϕ < 180◦ . Khi đó ABc
= (1, 0), CA
b
= (− cos ϕ, sin ϕ). Gọi


 B1 = (b1 cot ϕ, b1),

B = (b cot ϕ, b ),
2 2 2
, c1 , c2 6= 0, c1 6= c2 , b1 , b2 6= 0, b1 6= b2

 C1 = (c1 , 0),


C2 (c2 , 0)

−−−→ −→
thì B1 B2 = βb CA hay ((b2 − b1 ) cot ϕ, b2 − b1) = β(− cos ϕ, − sin ϕ) suy
ra b2 − b1 = −β sin ϕ.
−−−→ −→
Ta cũng có C1 C2 = γc AB tương đương với (c2 − c1, 0) = γ(1, 0) hay
c2 − c1 = γ.
Đường tròn (AB1C1 ) đia qua A, C1 nên x2 + y 2 − c1 x − λ1 y = 0, nó đi
qua B1 nên λ1 = b1 −c1sin sin ϕ cos ϕ
2ϕ , đường tròn (AB2 C2): x2 + y 2 − c2 x − λ2y = 0,
λ2 = b2 −c2 sin
β
ϕ cos ϕ
.
Trục đẳng phương hai đường tròn đó là

(c2 − c1 )x + (λ2 − λ1 )y = 0
β + γ cos ϕ
⇔ γx − y=0
sin ϕ
hay
y x sin ϕ − y cos ϕ
=
γ β
Để ý rằng y là khoảng cách đại số từ M(x, y) đến AB còn x sin ϕ − y cos ϕ
là khoảng cách đại số từ M(x, y) đến CA, ta suy ra điều phải chứng minh.
2) Với mỗi điểm M, kí hiệu X, Y, Z là khoảng cách đại số từ M đến
BC, CA, AB thì dễ thấy aX + bY + cZ = 2S, (S là diện tích tam giác
ABC) và ngược lại (X, Y, Z) mà aX + bY + cZ = 2S xác định một điểm
M duy nhất có các khoảng cách đại số nói trên là X, Y, Z.
Theo phần 1), phương trình dA là Yβ = Zγ , của dB là Xα
= Zγ , của dC
là Zα = Yβ . Điểm chung của dA , dB , dC (nếu có) là điểm M(X, Y, Z) mà
2.4. Đáp án chọn đội tuyển năm học 1994 - 1995 47

(X, Y, Z) là nghiệm của hệ


(
aX + bY + cZ = 2S
X
α
= Yβ = Zγ

hay
X Y Z 2S
= = =
α β γ aα + bβ + cγ
hệ đó có nghiệm (và chỉ có một nghiệm) khi và chỉ khi aα + bβ + cγ 6= 0.
Chú ý: Nếu (AB1 C1), (AB2 C2) cắt nhau tại A0 6= A, có thể chứng minh
phần 1) (trong trường hợp này) nhờ phép đồng dạng thuận tâm A0, biến
B1 thành C1, biến B2 thành C2 và để hoàn thiện 1) còn cần xét (AB1C1 ),
(AB2C2 ) tiếp xúc nhau.
Bài 2. Gọi A là tập các giá trị n (n ≥ 3) để đa thức Pn (x) khả quy. Với
n ∈ A ta có
Pn (x) = f (x).g(x) (*)
trong đó

f (x) = am xm + · · · + a1x + a0
g(x) = bs xs + · · · + b1 x + b0
Pn (x) = xn+1 + kxn − 870x2 + 1945x + 1995
m ≥ 1, s ≥ 1, m + s = n + 1

Ta sẽ chứng minh m = 1 hoặc s = 1.


Giả sử có m ≥ 2 và s ≥ 2 suy ra m < n và s < n. Vì a0 b0 = 1995 chia
hết cho 5 và không chia hết cho 25 nên chỉ có một trong hai số a0, b0 là chia
.
hết cho 5, chẳng hạn a0 ..5 và b0 không chia hết cho 5. Trong dãy a0, a1, ..., am
gọi r là chỉ số nhỏ nhất để ar không chia hết cho 5, (1 ≤ r ≤ m < n, chú ý
tồn tại r vì am bằng ±1). Từ (??) suy ra cr = ar b0 + ar−1 b1 + · · · + a0br . Do
cách chọn r thì a0, a1, ..., ar−1 đều chia hết cho 5, cr là hệ số của xr trong
.
Pn (x) cũng chia hết cho 5, suy ra ar b0..5, điều này mâu thuẫn với ar , b0 đều
không chia hết cho 5. Vậy giả sử m ≥ 2 và s ≥ 2 là sai, suy ra hoặc m = 1
hoặc s = 1, lúc đó Pn (x) có nghiệm nguyên với ∀n ∈ A. Xét các trường hợp
sau
a) Nếu |xn | ≥ 2 ∀n ∈ A. Khi đó từ

xn (x + k) = 870x2 − 1945x − 1995

ta có
|870x2 − 1945x − 1995|
|xn + k| =
|xn |n
48 Chương 2. Đáp án tuyển sinh

Vì A vô hạn nên với n ∈ A đủ lớn thì |xn + k| < 1 suy ra xn + k = 0 suy ra


. .
870x2n − 1945xn − 1945 = 0 hay 174x2n − 389xn − 399 = 0. Vì 399..3 và 197..3
.
nên xn ..3. Đặt xn = 3y, ta có 522y 2 − 389y − 133 = 0 suy ra k = −3.
b) Nếu |xn | < 2, ∀n ∈ A thì xn chỉ có thể là +1, −1.
Với xn = 1 thì Pn (1) = 0 suy ra k = −3071.
Với xn = −1 thì Pn (−1) = 0.
Từ đó với n chẵn thì k = 821, còn với n lẻ thì k = −819.
Thử lại, thấy nếu k = −3, k = −3071 thì Pn (x) khả quy ∀n ≥ 3. Nếu
k = 821 thì Pn (x) khả quy với ∀n chẵn. Nếu k = −819 thì Pn (x) khả quy
với ∀n lẻ.
Bài 3. Có a3 + b3 ≥ 2(ab)3/2 suy ra (a3 + b3)n ≥ 2n (ab)3n/2 ≥ 4(ab)3n/2
vì n ≥ 2. Vì vậy, từ

(a3 + b3 )n = 4(ab)1995
ta được 3n ≤ 3990. Đặt (a, b) = d ta có a = da1 , b = db1 và (a1, b1 ) = 1.
Khi đó, từ (??) có d3n (a31 + b31 )n = 4d2990(a1 b1)1995 hay

(a31 + b31)n = 4d3990−3n (a1b1 )1995.


. .
Suy ra (a31 + b31 )n ..(a1b1)1995 suy ra (a31 + b31)n ..(a1b1 )n (do n < 1995 vì 3n ≤
.
3990) suy ra a3 + b3..a1b1 . Do vậy
1 1
 
a3 ...b a ...b
1 1 1 1

b3 ...a b ...a
1 1 1 1

(do (a1, b1) = 1) nên a1 = b1 = 1, lại do (a1, b1) = 1) suy ra a = b = d.


Khi đó từ (??) có 2n−2 = d3990−3n . Vì d > 1 nên suy ra d có dạng 2k
với k ≥ 1, và do đó n − 2 = k(3990 − 3n) hay n = 3990k+2 3k+1
, và do đó
.
n = 1330 − 1328 . Vì n ∈ N∗ nên 1328..3k + 1. Do 1328 = 24 .83 và
3k+1

( (
i 1 (mod 3) nếu i chẵn i 1 (mod 3) nếu i chẵn
2 ≡ , 2 .83 ≡
2 (mod 3) nếu i lẻ 2 (mod 3) nếu i lẻ

nên suy ra 3k + 1 ∈ {22 , 24 , 2 × 83, 23 × 83}.


Với 3k + 1 = 4 có k = 1 suy ra a = b = 2 và n = 998.
Với 3k + 1 = 16 có k = 5 suy ra a = b = 25 và n = 1247.
Với 3k + 1 = 166 có k = 55 suy ra a = b = 255 và n = 1322.
Với 3k + 1 = 664 có k = 221 suy ra a = b = 2221 và n = 1328.
2.4. Đáp án chọn đội tuyển năm học 1994 - 1995 49

Bài 4. Xét graph G có tập đỉnh là tập gồm n điểm đã cho và tập cạnh
là tập gồm 12 (n2 − 3n + 4) đoạn thẳng đã cho. Từ giả thiết của bài toán ta
thấy trong G tồn tại một cạnh mà sau khi bỏ nó đi thì được G0 không liên
thông. Giả sử a và b là hai đỉnh không liên thông với nhau trong G0 .
Gọi Va và Vb lần lượt là tập gồm tất cả các đỉnh của G0 mà liên thông
với a và b. Giả sử |Va | = n1 và |Vb | = n2 .
Dễ thấy, G0 có 12 (n2 − 3n + 2) cạnh; n1 ≥ 1, n2 ≥ 1, n1 + n2 ≤ n và

1 2 1 1 1
(n − 3n + 2) ≤ n1 (n1 − 1) + n2 (n2 − 1) + (n − n1 − n2)(n − n1 − n2 − 1)
2 2 2 2
hay (n1 − 1)(1 − n2 ) + (n − n1 − n2 )(1 − n1 − n2 ) ≥ 0. Do đó
(
(n1 − 1)(1 − n2 ) = 0
(n − n1 − n2 )(n1 + n2 − 1) = 0

Vậy n1 = n − 1, n2 = 1 hoặc n2 = n − 1 và n1 = 1.
Từ đó suy ra G0 có một đỉnh cô lập và (n − 1) đỉnh mà bậc của mỗi đỉnh
bằng n − 2. Do đó G có một đỉnh bậc 1, (n − 2) đỉnh mà bậc của mỗi đỉnh
bằng n − 2 và một đỉnh có bậc bằng n − 1. Bởi thế chu trình đơn có độ dài
lớn nhất trong G là chu trình đơn độ dài n − 1 nếu n ≥ 4, 0 nếu n = 2hoặc
n = 3.
Vậy (
n − 1 nếu n ≥ 4
kmax =
0 nếu n = 2, n = 3

Bài 5. Giả sử n + 1 = 2f (n) (1 + 2α), p + 1 = 2f (p)(1 + 2β) với α, β ≥ 0.


Cặp số (n, p) là cặp số đẹp khi và chỉ khi 2f (n) > p hay

2f (n) ≥ p + 1 (1)

Từ đó ta có n + 1 = 2f (n) (1 + 2α) ≥ p + 1 suy ra

n≥p (2)

Từ (1) ta có 2f (n) ≥ p + 1 = 2f (p)(1 + 2β) suy ra 2f (n) ≥ 2f (p), thành thử


f (n) ≥ f (p). Từ đó
.
(n + 1)..2f (p) (3)
Ta cần tìm bộ ba số (n, p, q) sao cho ba cặp số (n, p) (p, q) và (n+p+q, n)
đều là các cặp số đẹp.
Giả sử n + p + q + 1 = 2f (n+p+q) (1 + 2γ).
50 Chương 2. Đáp án tuyển sinh

Theo (2) vì (n, p) và (p, q) là cặp số đẹp nên n + p + q + 1 ≤ 3n + 1. Vì


(n + p + q, n) là cặp số đẹp nên 2f (n+p+q) ≥ n + 1 theo (1). Kết hợp các điều
kiện trên có

2f (n+p+q) (1 +2γ) = n + p + q +1 ≤ 3n +1 ≤ 3(2f (n+p+q) − 1) +1 < 3.2f (n+p+q)

suy ra 1 + 2γ < 3, và do đó 1 + 2γ = 1 hay

n + p + q + 1 = 2f (n+p+q) (4)

Mặt khác, 2f (n+p+q) ≥ n + 1 = 2f (n) (1 + 2α) suy ra

f (n + p + q) ≥ f (n) (5)

Từ 2f (n+p+q) = n + p + q + 1 = (n + 1) + (p + 1) + (q − 1) theo (3) và (5)


. . .
ta có (n + 1)..2f (p) và (p + 1)..2f (p) suy ra (q − 1)..2f (p), nhưng từ cặp số đẹp
(p, q) có 2f (p) > q nên chỉ xảy ra hai trường hợp hoặc q = 0 và f (p) = 0,
hoặc q = 1 và f (p) > 0.
Xét q = 0 và f (p) = 0, từ (4) có n + p + q + 1 = n + p + 1 = 2f (n+p+q) ,
đồng thời n + p + 1 = (n + 1) + p = 2f (n) (1 + 2α) + p.
.
Từ (5) và 2f (n) (1 + 2α) + p suy ra p..2f (n) mà 2f (n) > p nên p = 0. Từ
cặp số đẹp (n + p + q, n) = (n, n) suy ra n + 1 ≥ 2f (n) ≥ n + 1 suy ra
n − 1 = 2f (n) = 2m . Tử lại, ta thấy bộ ba số (n, p, q) = (2m − 1, 0, 0) thỏa
mãn với m ∈ Z và m ≥ 0.
Xét q = 1 và f (p) > 0. Từ (4) ta có 2f (n+p+q) = n + p + q + 1 = (n + 1) +
(p+1) = 2f (n) (1+2α)+2f (p) (1+2β). Chú ý rằng f (n+p+q) ≥ f (n) ≥ f (p)
.
suy ra 2f (p)..2f (n) nên f (p) = f (m).
Từ 2f (p) = 2f (n) ≥ p + 1 = 2f (p)(1 + 2β) suy ra 1 + 2β = 1 suy ra
p+1 = 2f (p) . Ta có 2f (n+p+q) = n+p+q +1 = (n+1)(+(p+a) = n+1+2f (n) ,
suy ra n + 1 = 2f (n+p+q) − 2f (n) = 2k − 2m . Thử lại ta thấy, bộ ba số
(n, p, q) = (2k − 2m − 1, 2m − 1, 1) thỏa mãn với k, m ∈ Z và k > m ≥ 1.
Bài 6. 1) Ta tính đạo hàm

2x(x3 − 3x + 3)
f 0 (x) = .
3(x2 − 1)2

Dễ dàng chứng minh được x3 − 3x + 3 > 0 ∀x > 1. Từ đó suy ra f 0 (x) >


0 ∀x > 1 và do đó hàm f (x) đồng biến trên (1, +∞). Hơn nữa lại có
lim+ f (x) = −∞ và lim f (x) = +∞. Nếu suy ra f (x) với tập xác định
x→1 x→+∞
(1, +∞) sẽ có tập giá trị là (−∞, +∞).
Từ các kết quả ở trên, theo định lí về hàm ngược, ta suy ra tồn tại hàm
g(x) liên tục trên R, có tập giá trị là (1, +∞), và f (g(x)) = x, ∀x ∈ R.
2.5. Đáp án chọn đội tuyển năm học 1995 - 1996 51

tiếp theo, ta sẽ chứng minh g(x) > x, ∀x ∈ R. Thật vậy, với x ≤ 1 thì
3 −3x+3
g(x) > 1 ≥ x. Với x > 1 thì do x − f (x) = x3(x 2 −1) nên x > f (x) ∀x > 1,

hay g(x) > x, (do tính đồng biến của f (x) trên (1, +∞).
2) Kí hiệu gn (x) = g(g(...g(x))...). Ta sẽ tìm a dưới dạng a = gn (x0 ) với
x0 ∈ R. Khi đó
a0 = gn (x0)
a1 = f (x0 ) = gn−1 (x0 ) > 1
a2 = f (a1 ) = gn−2 (x0) > 1
·········
an−1 = g(x0) > 1
an = x0

Với x0 6= ±1 thì an+1 f (x0 ). Do g(x) > x, ∀x nên a0 > a1 > a2 > · · · an . Suy
ra nếu chọn x0 sao cho x0 = ±1 và f (x0 ) = gn (x0) thì dãy {an } sẽ là dãy
tuần hoàn với chu kỳ dương nhỏ nhất bằng n + 1.
Bây giờ ta sẽ chứng minh tồn tại x0 thỏa mãn các điều kiện nói trên. Thật
vậy, xét hàm h(x) = f (x) − gn (x) trên (−1, 0]. Ta có h(0) = f (0) − gn (0) =
1 − gn (0) < 0, và limx→−1+ h(x) = +∞ (do gn (−1) là số xác định). Hơn
nữa, do h(x) liên tục trên (−1, 0) của phương trình h(x) = 0, ta có a > 1
và dãy {an } là dãy tuần hoàn với chu kỳ dương bé nhất bằng 1995.

2.5 Đáp án chọn đội tuyển năm học 1995 -


1996
Bài 1. Gọi 3n điểm đã cho là A1, A2 , ..., A3n. Hiển nhiên trong mặt phẳng
chứa 3n điểm đó, ta có thể dựng được đường thẳng ∆ sao cho Ai ∈ / ∆,
i = 1, 3n, A1, A2 , ..., A3n nằm về cùng một phía của ∆; và ∆ không song
song với Ai Aj (∀inot = j ∈ {1, 2, ..., 3n}).
Kí hiệu dAi là khoảng cách từ điểm Ai đến ∆. Khi đó dAi 6= dAj (∀i 6=
j ∈ {1, 2, ..., 3n}). Không mất tính tổng quát, giả sử
dA1 < dA2 < · · · < dA3n (1)
Qua mỗi điểm A3i+1, i = 0, ..., n − 1, kẻ đường thẳng ∆i k ∆ dễ dàng suy ra
n tam giác A3j+1 A3j+2A3j+3 , i = 0, ..., n − 1 đôi một rời nhau và mỗi điểm
Ai , i = 1, 3n) là đỉnh có đúng một tam giác trong số n tam giác đó.
Bây giờ ta sẽ chứng minh tổng S diện tích của n tam giác nói trên thoả
mãn S < 12 . Thật vậy, xét 4A3i+1A3i+2 A3i+3, i ∈ {0, 1, ..., n − 1}) và gọi Si
là diện tích của nó. Dễ thấy có thể dựng được hai đường thẳng a, b cùng
vuông góc với ∆ và sao cho
52 Chương 2. Đáp án tuyển sinh

1) a đi qua đúng một trong ba điểm A3i+1 , A3i+2, A3i+3 còn b đi qua ít
nhất một trong hai điểm còn lại.
2) cả ba điểm A3i+1, A3i+2 , A3i+3 cùng nằm trong dải phẳng (kể cả hai
biên) bị giới hạn bởi a và b.
Thế thì nếu gọi {A} = a ∩ ∆i , {B} = a ∩ ∆i+1 , {C} = b ∩ ∆i+2 , {D} =
b ∩ ∆i ta sẽ có hình chữ nhật ABCD
chứa toàn bộ ∆A3i+1A3i+2A3i+3 . Từ đó Si < 12 SABCD = 12 AD.CD < 12 di
với di là khoảng cách giữa hai đường thẳng ∆i và ∆i+1 . Từ đó suy ra

X
n−1 X
n−1
1
S= Si < 2
di ≤ 12 A1 A3n ≤ 12 .
i=0 i=0

(vì A1 A3n ≤ 1). Bài toán được chứng minh.


P[ n2 ] 2i+1
 i
Bài 2. Từ an = i=0 n
3 suy ra công thức tổng quát
√ √ n
(1 + 3)n − (1 − 3)
an = √ .
2 3

Xét n chẵn và n lẻ.


1) Với n = 2k, ta có an = 2k uk với
√ √
(2 + 3)k − (2 − 3)k
uk = √ (1)
2 3

Dãy {uk } thoả mãn


uk+2 = 4uk+1 (2)
với u1 = 1, u2 = 4
.
Gọi g(k) là số l ∈ N lớn nhất để uk ..2l .
Từ an = 2k un suy ra
f (2k ) = k + g(h) (3)
Từ (2) thấy k lẻ, suy ra uk lẻ. Vậy k lẻ thì

g(k) = 0 (4)

nghĩa là
f (2k) = k nếu k lẻ (40 )
Xét k = 2m chẵn thì
√ √ m
u2m = um .dm với dm = (2 + 3)m + (2 − 3) (5)
2.5. Đáp án chọn đội tuyển năm học 1995 - 1996 53

Dãy {dm } thoả mãn

dm+2 = 4dm+1 − dm với d1 = 4, d2 = 14 (6)

Từ (6) và vì dm chẵn với mọi m nên suy ra dm+2 ≡ −dm (mod 8).

Nếu m lẻ, thì dm ≡ 4 (mod 8)


(7)
Nếu m chẵn, thì dm ≡ 6 (mod 8)

Từ (7) và u2m = um .dm suy ra


(
g(m) + 2 nếu m lẻ
g(2m) = (8)
g(m) + 1 nếu m chẵn

Với k = 2s h, h lẻ thì từ (8) và g(k) = 0 suy ra

g(2s h) = g(2s−1 h) + 1 = · · · = g(2h) + s − 1 = g(h) + 2 + s − 1 = s + 1 (9)

Từ (3), (9), và do (40 ta có


(
k với k lẻ
f (2k) = (10)
k+s+1 với k = 2s h, s ≥ 1, k lẻ

2) Với n = 2k + 1. Ta thấy dãy {an } thoả mãn

an+2 = 2an+1 + 2an , với a1 = 1, a2 = 2. (11)

Ta chứng minh
f (2k + 1) = k (12)
bằng quy nạp với k = 0, 1 dễ thấy đúng. Giả sử đúng với k. Từ (10) có
f (2k) ≥ k và theo giả thiết quy nạp f (2k + 1) = k nên

a2(k+1)+1 = 2a2(k+1) + 2a2k+1 = 2.2k+1 .n + 2.2k N = 2k+1 (2M + N )

với N lẻ theo quy nạp. Vậy f (2(k + 1) + 1) = k + 1 đúng với k + 1.


3) Tìm mọi n để f (n) = 1996
Nếu 2k + 1, theo f (2k + 1) = k thì f (n) = f (2k + 1) = k = 1996 suy ra
n = 3993.
Nếu n = 2s+1 h với h lẻ. Với s = 0 suy ra f (2k) = k lẻ không thoả mãn
với s ≥ 1 suy ra f (2s+1 h) = 2s h + s + 1 = 1996 suy ra 2s h + s = 1995 suy
ra h lẻ và 1 ≤ s ≤ 9. Thử thấy s = 1, n = 3998 và s = 3, n = 3984 với
s = 5, 7, 9 không có nghiệm. Đáp số n = 3984, n = 3998, n = 3993.
54 Chương 2. Đáp án tuyển sinh

Bài 3. Đặt

f (a, b, c) = (a + b + c)4 + a4 + b4 + c4 − 12abc(a + b + c) − 47 (a4 + b4 + c4 )


= (a + b + c)4 + 37 (a4 + b4 + c4 ) − 12abc(a + b + c)

Vì f (a, b, c) = f (−a, −b, −c) nên chỉ cần xét f (a, b, c) tại a, b, c mà a+b+c ≥
0. Khi đó chỉ có thể xảy ra một trong ba trường hợp sau
Nếu a, b, c ≥ 0. Lúc này ta có a + b + c ≥ 3(abc)1/3 suy ra (a + b + c)4 ≥
27abc(a + b + c) suy ra f (a, b, c) ≥ 15abc(a + b + c) ≥ 0 và f (a, b, c) = 0 khi
và chỉ khi a = b = c = 0.
Nếu có đúng một trong ba số a, b, c là số âm. Khi đó có f (a, b, c) > 0.
Nếu có đúng hai trong ba số a, b, c là số âm. Không giảm tính tổng quát,
coi a, b, < 0 từ a + b + c ≥ 0 suy ra c ≥ −a(+b) > 0. Đặt a = −α, b = −β
với α, β > 0, thế thì c ≥ α + β và

f (a, b, c) = f (−α, −β, c)


= {c − (α + β)}4 + 37 (c4 + α4 + β 4) − 12αβc{c − (α + β)}.

Vì α, β > 0 nên α4 + β 4 ≥ 2( α+β


2
)4 và 0 < α < β ≤ ( α+β
2
)2 . Do đó ta có
đánh giá

α+β 4 α+β 2
f (a, b, c) ≥ {c − (α + β)}4 + 37 {c4 + 2( ) } − 12c( ) {c − (α + β)}
2 2
= (c − 2x)4 + 37 (c4 + 2x4 ) − 12cx2 (c − 2x)

trong đó x = α+β
2
, x > 0.
Đặt c = tx, từ c ≥ 2x suy ra t ≥ 2. Vậy ta tiếp tục đánh giá f (a, b, c)

f (a, b, c) ≥ x4{(t − 2)4 + 37 (t4 + 2) − 12t(t − 2)}


= x4{(t − 2)4 + (t4 + 2) − 12t(t − 2) − 47 (t4 + 2)}
= x4{2(t − 1)4 + 16 − 47 (t4 + 2)} = 2x4 {(t − 1)4 − 27 t4 + 52
7
}

Xét g(t) = (t−1)4 − 27 t4 + 52 trên [2, +∞). Ta có g 0 (t) = 4{(t−1)3 − 27 t3}. Dễ


7 q
1 2
thấy rằng với t ≥ 2 thì g 0 (t) > 0 hay t > 1−t 0
với t 0 = 7
, g 0(t) = 0 khi và
1 1 1
chỉ khi t = 1−t0
, g 0 (t) < 0 khi và chỉ khi 2 ≤ t ≤ 1−t0
. Do đó g(t) ≥ g( 1−t 0
)
với t ≥ 2.
Ta có r 
1 2 3 2 1 4 2  1 4 52
g( )= 7
−7 +7.
1 − t0 7 1 − t0 1 − t0
2.5. Đáp án chọn đội tuyển năm học 1995 - 1996 55
q
3 2
Do t0 = 7
nên

    
1  2 1 4 2 1
g = 7
26 − (1 − t0( ) = 7
26 −
1 − t0 1 − t0 (1 − t0 )3
 
 1 
2
= 7 26 − q .
 (1 − 3 27 )3 

1

Ta chứng minh rằng 26 > 3 2 4
. Thật vậy, bất đẳng thức này tương
(1− 7
)
đương với mỗi bất đẳng thức sau

rr
2 33 1
1− − >0
7 26
r
2 1 3 1
1− − −3 >0
7 26 91
41 1
> √
182 3
91
68921 > 66428.

1
Vậy nên g(t) ≥ g( 1−t 0
) > 0 ∀t ≥ 2. Tóm lại f (a, b, c) ≥ 0 ∀a, b, c ∈ R và
f (0, 0, 0) = 0 nên mina,b,c∈R f (a, b, c) = 0

Bài 4. Chú ý rằng phép đối xứng qua đường phân giác AB của góc định
−−→ −−→ −−→ −−→
hương (AH, AH 0) biến AH thành AH 0, còn phép đối xứng qua đường phân
−−→ −−→ −−→ −−→ −−→
giác ngoài AC của góc (AH, AH 0) thì biến AH thành AH 00 = −AH 0 .

Hình 1
1) Giả sử SAB (M) = M1 , SBC (M1 ) = M2 , SCA (M2 ) = M 0 . Đặt f =
SCA .SBC .SAB . Gọi H, K, I lần lượt là hình chiếu của A, B, C xuống cạnh
đối diện. Gọi đường thẳng IK là ∆. Ta sẽ chỉ ra f : ∆ → ∆ và bảo tồn
hướng của ∆.
56 Chương 2. Đáp án tuyển sinh

a) Khi 4ABC không vuông thì H, K, I phân biệt, do đó AB, BC, CA


là phân giác ngoài của 4HKI. Khi có một góc tù, chẳng hạn ∠A > 90◦ thì
BC là phân giác ngoài, còn CA, AB là phân giác trong 4HKI. The chú ý
trên, dễ thấy f : 4 → 4 và bảo tồn hướng của ∆.

Hình 2
b b b ◦
b) Khi C = H = K = 90 (C ≡ K) thì ∆ là đường thẳng CI.
c) Khi Bb=H b = Ib = 90◦ (B ≡ I) thì ∆ là đường thẳng BK.
d) Khi Ab=K b = Ib = 90◦ (A ≡ I ≡ K) thì ∆ là đường thẳng đối xứng
của đường thẳng AH qua AB (hay qua AC) (Hình 1)
2) f | ∆ : ∆ → ∆ bảo tồn khoảng cách và hướng nên f | ∆ là phép
tịnh tiến theo véctơ − →v nào đó (−→v k ∆) (xem hình 2: M, N ∈ ∆, M 0 =
− −−→ −− →
f (M), N 0 = f (N ) thì MM 0 = NN 0 = − →
v ) và hình 1: H 0 ∈ ∆, H 00 = f (H 0 )
−−0−→00 −
thì H H = → v ).
Với M tuỳ ý, gọi J là hình chiếu vuông góc của M xuống ∆ thì f (J ) là
−→ →
hình chiếu vuông góc của f (M) xuống ∆ và J J 0 = − v . Do f đảo hướng nên
M và f (M) = M 0 nằm khác phía đối với ∆ nên MM 0 ≥ JJ 0 = |− →
v |, và nếu
0 0 0
M ∈ / ∆ thì MM > JJ . Vậy tập hợp các điểm M để MM đạt giác trị bé
nhất là đường thẳng ∆.
3) a) Khi lấy liên tiếp phép đối xứng qua các trục là hoán vị vòng quanh
ba cạnh 4ABC được

SAB ◦ SCA ◦ SBC = SAB ◦ f ◦ SAB


SBC ◦ SAB ◦ SCA = SCA ◦ f ◦ SCA

đều có dạng Sx ◦ f ◦ Sx = g (Sx là phép đối xứng qua trục x).


Đặt ∆0 = Sx (∆) thì với M0 ∈ ∆0 khoảng cách

ρ(M0 , g(M0 )) = ρ(M0 , Sx ◦ f ◦ Sx−1 (M0 ))


= ρ(Sx−1 (M0 ), f(Sx−1 (M0)) = |−

v | ≤ ρ(M, f (M))
= ρ(M, Sx gSx (M)) = ρ(Sx (M), g(Sx (M)))

Vậy khoảng cách ngắn nhất đối với g = Sx ◦ f ◦ Sx cũng là |→



v |.
2.5. Đáp án chọn đội tuyển năm học 1995 - 1996 57

b) Khi lấy liên tiếp phép đối xứng qua các trục theo thứ tự sau:
SAB .SBC , SCA = f −1
thì khoảng cách bé nhất của MM 0 cũng như đối với f đã xét trên. Lấy hoán
vị vòng quanh các trục đối xứng ở trên ta được
SCA ◦ SAB ◦ SBC , SBC ◦ SCA ◦ SAB
đều có dạng Sx ◦ f −1 ◦ Sx = g 0 nên theo a) khoảng cách ngắn nhất đối với
g 0 cũng là −

v.
Chú ý: Có thể chứng minh 1), 2): f là phép đối xứng trượt, trục ∆ đi
qua trung điểm Mf (M) (M tuỳ ý) và xác định được ∆ (một cách hình
học) đến kết quả như trên, véctơ trượt là − →
v.
Có thể xét cụ thể các trường hợp tam giác ABC tương tự trên đây, tính
được khoảng cách bé nhất nói trên là |−→v | = a cos A+b cos B +c cos C, chẳng
hạn khi 4ABC có ba góc nhọn thì |− v | kà chu vi tam giác HIK nên |−
→ →v | có


tính chất đối xứng đối với a, b, c và ∠A, ∠B, ∠C : | v | = IH + HK + KI.

Bài 5. Giả sử số học sinh được mời là 65 em. Ta đặt tương ứng mỗi
em với một điểm trên mặt phẳng và hai em được đặt tương ứng với hai
điểm khác nhau. Với mỗi cặp, hai em chưa quen nhau ta nối hai điểm tương
ứng với hai em đó bởi một đoạn thẳng. Khi đó ta được một Graph đơn, vô
hương, có 65 đỉnh, bậc mỗi đỉnh không nhỏ hơn 56 và với hai đỉnh kề nhau
bất kỳ luôn tồn tại ít nhất một điểm không kề với cả hai đỉnh ấy, có 65
đỉnh và thoả mãn

1) Bậc của mỗi đỉnh không lớn hơn 8


2) Với hai đỉnh không kề nhau, tồn tại ít nhất một đỉnh kề với cả hai
đỉnh ấy.
Xét G: xét đỉnh A bất kỳ của G và gọi A1 , A2, ..., Ak (k ≤ 8) là tất cả
các đỉnh kề với A. Nếu k ≤ 7 thì sẽ có tối đa 72 = 49 đỉnh mà mỗi đỉnh
kề với ít nhất một trong các đỉnh A1, A2, ..., Ak và không kề với A. Suy ra
số đỉnh của G không vượt quá 49 + 7 + 9 = 57 < 65 trái với giả thiết. Vậy
phải có k = 8, suy ra mỗi đỉnh của G có bậc bằng 8. Từ đây, kết hợp với
2), ta được
58 Chương 2. Đáp án tuyển sinh

i) Ai, Aj không kề nhau ∀i 6= j ∈ {1, 2, ..., 8}


ii) Mỗi đỉnh Ai , (i = 1, 8), ngoài A ra sẽ kề với đúng bảy đỉnh khác và
nếu kí hiệu Ait t = 1, 7) là bảy đỉnh ấy thì {Ai1 , ..., Ai7 } ∩ {Aj1 , ..., Aj7 } = ∅
(∀i ∈ / j ∈ {1, 2, ..., 8}).
Từ đó suy ra trong G không có chu trình đơn độ dài 3 cũng như chu
trình đơn đội dài 4. Do vậy Ait và Ais không kề nhau (∀i = 1, 8), ∀t 6= s ∈
{1, 2, ..., 7}, và do đó nếu Ait kề Ajs (i 6= j) thì Ait không kề Ajm m 6= s.
Từ đó suy ra có tất cả 14 38 = 14.7.8 chu trình đơn độ dài 6 đi qua A.
Vì A là đỉnh bất kỳ của G nên số chu trình đơn độ dài 6 trong G là
14.7.8.65
6
= 49.8.65
3
/ Z. Điều vô lý. Vây không tồn tại G và do đó không tồn

tại G thoả mãn đề bài.

Bài 6. Ta xét bài toán tổng quát với x0 > 0 cho trước bất kỳ.
1) Với a = 0 hiển nhiên xi = 0, ∀i > 0 dãy hội tụ.
a
2) Với a > 0 kí hiệu f (x) = 1+x 2 và g(x) = f (f (x)). Từ đó suy ra f (x)

là hàm giảm trên (0, +∞) nên g(x) là hàm tăng trên (0, ∞).
Xét dãy {x2n } (chỉ số chẵn). Đặt un = x2n thì un+1 = g(un ). Do g(x)
tăng nên {un } là dãy đơn điệu (tăng hoặc giảm). Mặt khác, 0 < un < a,
nên luôn luôn tồn tại lim un = limn→∞ x2n = l. Lấy giới hạn khi n → ∞ có

g(l) = l (1)

a) Ta chứng minh 0 < a ≤ 2 thì dãy {xn } có giới hạn. Thật vậy, giả sử
f (l) = v hay
a = v + vl2 (2)
Ta có f (f (l)) = f (v) tương đương với f (v) = g(l) = l hay

a = v + lv 2 (3)
Trừ (2) cho (3) được (v − l)(vl − 1) = 0. Nếu v 6= l thì vl = 1, thay vào
(2) được v + l = a suy ra v.l là nghiệm của phương trình x2 − ax + 1 = 0.
Vì a ≤ 2 nên a = 2 suy ra v = l = 1 trái điều giả sử. Vậy, v = l, tức là
f (l) = l. Từ đó limx→∞ x2n+1 = limx→∞ x2n = f (l) = l = lim x2n .
Vậy limn→∞ xn tồn tại.
b) Ta chứng minh với a > 2 thì dãy {xn } hội tụ khi và chỉ khi a = x32 +x1 .
a
Thật vậy, giả sử a = x30 + x0 suy ra x0 = 1+x 3
2 = x1 suy ra a = x1 + x1 suy
0
ra x2 = x1 ,.v..v... Vậy {xn } là dãy hằng, suy ra tồn tại limn→∞ xn = x0.
Ngược lại, giả sử tồn tại limn→∞ xn = k. Qua giới hạn ta được f (k) = k
hay a = k 3 + k, suy ra k là nghiệm duy nhất của x3 + x − a. Ta có

(1 + x2 )2 (a − x) − a2x −(x2 + x − a)(x2 − ax + 1)


g(x) − x = = .
a2 + (1 + x2 )2 a2 + (1 + x2)2
2.6. Đáp án chọn đội tuyển năm học 1996 - 1997 59

Vì a > 2 nên phương trình x2 − ax + 1 = 0 có hai nghiệm phân biệt, α, β.


Giả sử α < β, ta thấy α < k < β. Thật, vậy vì a > 2 suy ra k > 1 do đó
a − ak 2 < 0. Thay a = k 3 + k vào ta được k 3 + k − ak 2 < 0 hay α < k < β.
Dấu của g(x) − x là dấu của −(x − k)(x − α)(x − β). Ta có bảng xét
dấu của g(x) − x như sau

x α k β
g(x) − x + 0 − 0 + 0 −

Xét dãy un = x2n như trên thì dãy {un } là đơn điệu. Vì limn→∞ un = k nên
∃n0 để α < un0 < β. Nếu α < un0 < k thì un0 +1 = g(un0 ) < un0 suy ra
{un } là dãy giảm và un0 < k, do đó nó không hội tụ về k. Tương tự nếu
k < un0 < β thì {un } là dãy tăng với un0 > k do đó nó không hội tụ về k.
Vậy chỉ có thể un0 = k. Chú ý rằng nếu ∃i: xi = k thì suy ra xi−1 = k. Thật
vậy x0 = k = 1+xa2 suy ra k(1 + x2i−1 ) = a = k 2 + k suy ra x2i−1 = k 2 suy
i−1

ra xi−1 = k. Thành thử un0 = k, do đó un = k hay x0 = k. Vì a = k 3 + k


nên a = x30 + x0. Điều phải chứng minh.
3) Với a < 0. Đặt yn = −xn. Khi đó

−a |a|
yn+1 − xn+1 = 2
= .
1 + xn 1 + yn2

Vậy {xn } hội tụ khi và chỉ khi {yn } hội tụ. Vì yn > 0, ∀n ≥ 1 nên theo
trên với |a| ≤ 2 thì {yn } hội tụ. Nếu |a| > 2 thì {yn } hội tụ khi và chỉ khi
|a| = y13 + y1 . Mà từ (2.5) ta có |a| = y1 (1 + y02) = y1(1 + x20 ) suy ra x0 = y1 ,
thành ra |a| = x30 + x0.
3
Kết
√ luận: Dãy {x√ n } hội tụ khi và chỉ khi |a| ≤ 2 hoặc |a| = x0 + x0 =
1997 1996 khi x0 = 1996.

2.6 Đáp án chọn đội tuyển năm học 1996 -


1997
Bài 1. Gọi O là tâm mặt cầu ngoại tiếp ABCD. Điều kiện xác định điểm
P là

P A2 +a21 +b2 +c2 = P B 2 +b21 +c2 +a2 = P C 2 +c21 +a2 +b2 = P D2 +a2 +b21 +c21 .
60 Chương 2. Đáp án tuyển sinh

Tính

P A2 + a21 + b2 + c2 = P A2 + DA2 + BA2 + cA2


−→ −−→ −→ −→
= P A2 + DA2 + BA2 + CA2
−→ −→ −−→ −→ −−→ −→ −→ −→
= (P O + CA)2 + (DO + OA)2 + (BO + OA)2 + (CO + OA)2
−→ −→ −→−→ −→ −−→ − −
→ −→ −→
= P O2 + 4OA2 + 2P O OA − 2OA2(OD + OB + OC + OA)+
−→ −−→ −
−→ −→
2OA2 + DO 2 + BO 2 + OC 2
−→ −→ − −→
= P O2 + 9R2 + 2P O.OA.OE
−→ − −→ −→
= P O2 + 9R2 − 2OA(OE + OP ).


−→ −→ −→ −→ −−→
trong đó đặt OE = OA + OA + OC + OD (điểm E xác định). Tính tương
tự đối với P B 2 + b21 + c2 + a2, P C 2 + c21 + a2 + b2, P D2 + a21 + b22 + c22. thì
ta được

−→ −
−→ −→ −
−→ −
−→ − −→ −→ −
−→ −→ −−→ −
−→ −→
OA(OE + OP ) = OB(OE + OD) = OC(OE + OP ) = OD(OE + OP )

−→ −
−→ −→ −
−→− −→ −→ −→ − −→ −−→ −→
Ta có OA(OE + OP ) = OB(OE + OP hay (OA − OB)(OE + OP ) = 0
−→ − −→ −→
hay BA(OE 2 + OP ) = 0. Từ đó ta có
− −→ −→ −→

(OE + OP ).BA = 0

−→ −→ −→
(OE + OP ).CA = 0

− −→ −→ − −→
(OE + OP ).DA = 0


−→ −→ − → −→ − −

hay là OE 2 + OP = 0 hay P O = OE. Vậy điểm P xác định duy nhất.
−→ −→ −→ −→ −→ −→ −→ −→
Từ P A2 = (P O 2 +OA)2 = P O 2 +OA2 +2P O.OA = P O2 +R2 +2P O.OA
2.6. Đáp án chọn đội tuyển năm học 1996 - 1997 61

thì
−→ −→ − −→ −→ −−→
P A2 + P B 2 + P C 2 + P D2 = 4P O2 + 4R2 + 2P O(OA + OB + OC + OD)
−→ − −

= 4P O2 + 4R2 + 2P O.OE
= 4P O2 + 4R2 + 2P O2
= 6P O2 + 4R2 ≥ 4R2 .

Dấu đẳng thức xảy ra khi và chỉ khi P O2 = 0 hay P ≡ O. Khi đó ta có

a21 + b2 + c2 = b21 + c2 + a2 = c21 + a2 + b2 = a21 + b21 + c21

hay a1 = a, b1 = b, c1 = c, tức là tứ diện ABCD gần đều.

Bài 2. 1) Giả sử k là số sao cho có thể thiết lập được hệ thông đường
bay thoả mãn các điều kiện của đề bài. Khi đó, tổng số đường bay trực tiếp
giữa hai thành phố sẽ là 25×k
2
. Suy ra k ≡ 0 (mod 2).
Xét một thành phố A bất kỳ. Theo giả thiết, từ A có đường bay trực
tiếp đên k thành phố khác, gọi là A1, A2, ..., Ak . Mỗi thành phố Ai , i = 1, k,
lại có đường bay trực tiếp đến k − 1 thành phố khác, (không kể A). Hơn
nữa, ta lại có: Nếu từ B đến A không có đường bay trực tiếp thì từ B phải
có đường bay trực tiếp đến ít nhất một thành phố Ai . Từ những lập luận
trên suy ra, số thành phố chỉ có thể tối đa là 1 + k + k(k − 1) = k 2 + 1. Như
vậy 25 ≤ k 2 + 1. Kết hợp với k ≡ 0 (mod 2), suy ra k ≥ 6.

2) Với k = 6 ta sẽ chỉ ra cách thiết lập hệ thống đường bay thoả mãn
các điều kiện của đề bài. Chia 25 thành phố thành năm nhóm, mỗi nhóm
gồm năm thành phố. Các thành phố của nhóm thứ i, i = 1, 5, ta kí hiệu bởi
(i) (i) (i) (i) (i)
A1 , A2 , A3 , A4 , A5 . Với các thành phố trong cùng nhóm i, ta thiết lập
(i) (i) (i) (i) (i) (i) (i) (i) (i) (i)
các đường bay A1 A2 , A2 A3 , A3 A4 , A4 A5 , A5 A1 . Giữa các thành
phố thuộc hai nhóm i, j bất kỳ, i 6= j ∈ {1, 2, 3, 4, 5}, xây dựng các đường
(i) (j) (i) (j) (i) (j) (i) (j) (i) (j)
bay sau A1 A1 , A2 A4 , A3 A2 , A4 A5 , A5 A3 .
62 Chương 2. Đáp án tuyển sinh

Bằng cách xây dựng các đường bay như trên, ta có: Từ thành phố A
bất kỳ sẽ có đường bay trực tiếp đến đúng hai thành phố, trong cùng nhóm
với A và có đường bay trực tiếp đến đúng bốn thành phố khác nhóm với
A. Do vậy từ mỗi thành phố sẽ có đường bay trực tiếp đến đúng sáu thành
phố khác.
Hơn nữa, với A, B là hai thành phố bất kỳ mà giữa chúng không có
đường bay trực tiếp ta thấy:
- Nếu A, B thuộc cùng nhóm thì dễ thấy luôn tồn tại một thành phố
trong nhóm đó mà từ C có đường bay trực tiếp đến cả A và B.
- Nếu A, B không cùng nhóm thì qua hình vẽ trên dễ dàng kiểm tra
được sự tồn tại của thành phố C mà từ C có đường bay trực tiếp đến cả A
và B.
3) Vậy kmin = 6.

Bài 3. Giả sử α là số thực sao cho tồn tại dãy vô hạn các số tự nhiên
{an } thoả mãn các điều kiện của đề bài. Dễ thấy α = 0 là một số thực như
vậy. Do đó, để tìm số α lớn nhất, dưới đây ta chỉ xét α > 0.
Từ giả thiết về dãy {an } suy ra ai + aj ≥ aαi+j , ∀i, j ∈ N∗ . Dẫn tới
2a2i ≥ aα2i+1 ∀i ∈ N (1)
Nếu α 6= 1 thì (1) tương đương với mỗi bất đẳng thức sau
1 1
a 1−α a2i ≥ (2 1−α .a2i+1 )α ∀i ∈ N
1 1
a 1−α a2i ≤ (2 1−α .a2i )1/α ∀i ∈ N
hay
1 1
a 1−α a2n ≤ (2 1−α .a2n−1 )1/α ∀i ∈ N∗
1 1 n 1 n
Suy ra 2 1−α .a2n ≤ (2 1−α a1)1/αn, ∀n ∈ N∗. Mà a2n > 19972 nên 2 1−α .19972 <
1
(2 1−α .a1)1/αn , ∀n ∈ N∗ , hay
1 1 1
ln 2 1−α + 2n ln 1997 < n
ln(2 1−α .a1) ∀n ∈ N∗.
α
hay
1 1 1 1
ln 1997 < ln(2 1−α .a1) − ln 2 1−α ∀n ∈ N∗ (2)
2n .αn 2n

Từ (2) suy ra α ≤ 12 , vì nếu α > 12 thì, cho n → ∞, từ (2) ta được


ln 1997 ≤ 0, vô lý.
n−1
Nếu α = 1 thì (1) cho ta a2i+1 ≤ 2a2i , ∀i ∈ N, hay a2n ≤ 2a2 . Suy ra
n n
a2n ≤ 2n a1, ∀n ∈ N∗ . Mà a2n > 19972 nên 19972 < 2n .a1, ∀n ∈ N∗, hay
2n ln 1997 < n ln 2 + ln a1, ∀n ∈ N∗. Bất đẳng thức này tương đương với
n ln a1
ln 1997 < n
. ln 2 + n ∀n ∈ N∗ (3)
2 2
2.6. Đáp án chọn đội tuyển năm học 1996 - 1997 63

Cho n → ∞, từ (3) ta được ln 1997 ≤ 0 (vô lý). Tóm lại, ta phải có α ≤ 12 .


2) Với α = 12 ta sẽ xây dựng dãy {an } thoả mãn các điều kiện của đề
bài. Xét dãy {an } được xác định bởi a1 = 2 × 1997, a2 = 4 × 19972 − 2 và
an+1 = 2 × 1997an − an−1 ∀n ≥ 2. Dựa vào phương trình đặc trưng và các
n n ∗
√ an = p + q và n ∈ N ,
số hạng ban đầu a1, a√2 của {an } dễ dàng tìm được
2 2
trong đó p = 1997 + 1997 − 1 và q = 1997 − 1997 − 1. Suy ra an ∈ N,
n ∈ N∗ và
 p + q n
n n
an = p + q ≥ 2 = 2 × 1997n > 1997n ∀n ∈ N∗ (4)
2
Tiếp theo, ta sẽ chứng minh

un = a[ n ] + a[ n+1 ], ∀n ≥ 2 (5)
2 2

Vì a[ n ] + a[ n+1 ] = n nên (5) tương đương với


2 2

 .   
a[ n ]−t + a[ n+1 ]+t .. a[ n ] + a[ n+1 ] ∀n ≥ 2, t ∈ {0, 1, ..., n2 − 1} (6)
2 2 2 2

Do vậy, để chứng minh (5), ta sẽ chứng minh (6).


Với t = 0 thì (6) hiển nhiên đúng.
Với t = 1, ta có: nếu n = 2k thì
.
a[ n ]−1 + a[ n+1 ]+1 = ak−1 + ak+1 = 2 × 1997ak ..(ak + ak )
2 2

hay  . 
a[ n ]−1 + a[ n+1 ]+1 .. a[ n ] + a[ n+1 ]
2 2 2 2

Nếu n = 2k + 1 thì

a[ n ]−1 + a[ n+1 ]+1 = ak−1 + ak+2 = 2 × 1997ak − ak+1 + 2 × 1997ak+1 − ak


2 2

.
= (2 × 1997 − 1)(ak+1 + ak )..(ak+1 + ak )

Từ đây suy ra (6) đúng.  


Giả sử ta đã có (6) đến t mà 1 ≤ t ≤ n2 − 1. Ta sẽ chứng minh (6)
cũng đúng với t := t + 1. Thật vậy, ta có

a[ n ]−(t+1) + a[ n+1 ]+(t+1) = 2 × 1997a[ n ]−t − a[ n ]−t+1 +


2 2 2 2

+ 2 × 1997a[ n+1 ]+t − a[ n+1 ]+t−1


2 2
   
= 2 × 1997 a[ n ]−t + a[ n+1 ]+t − a[ n ]−(t−1) + a[ n+1 ]+(t−1)
2 2 2 2
64 Chương 2. Đáp án tuyển sinh

Theo giả thiết quy nạp thì


   . 
.
a[ n ]−t + a[ n+1 ]+t − a[ n ]−(t−1) + a[ n+1 ]+(t−1) . a[ n ] + a[ n+1 ] .
2 2 2 2 2 2

Theo nguyên lý quy nạp, (6) cũng đồng thời là (5), được chứng minh. Từ
đó, ta có
n n n+1 n+1
un = p[ 2 ] + q [ 2 ] + p[ 2 ] + q [ 2 ]
≥ 2(pn/2 + q n/2) > 2(pn + q n )1/2 = 2a1/2 1/2
n > an .

Từ đây và (4) cho thấy dãy {an } thoả mãn các điều kiện của đề bài ứng với
α = 12 .
3) Vậy αmax = 12 .

Bài 4. Tìm công thức tổng quát của f (n).


Từ
f (n + 2) = 503f (n + 1) − 1996f (n) (*)
có phương trình đặc trưng x2 − 503x + 1996 = 0. Phương trình này có hai
nghiệm nguyên là x= 4 và x2 = 499, từ đó f (n) = 4n + 499n .
Ta sẽ chứng minh rằng ước nguyên tố p = p(s) của f (2s ) có dạng
p(s) = 21+s u + 1 với u ∈ N.
Vì (4, 499) = 1 nên nếu
s s
p(s) | f (2s ) = 42 + 4992 (**)

với s ≥ 1 thì p(s) lẻ, suy ra (4, p(s)) = 1 và (499, p(s)) = 1.


s s
Từ (**) có thể viết 42 + 4992 = tp(s).
Giả sử p(s) − 1 = 2m v với v lẻ, m ≤ s thì có thể áp dụng định lý Fermat
s−m s p−1 s p−1
p(s) = p | (4002 )p−1 − 1 = (4992 ) 2m − 1 = (pt − 42 ) 2m − 1
n s−m o
2s p−1 p−1
2 p−1 p−1
= p.k + (−4 ) 2 m
− 1 = p.k + (−1) 2 m
(4 ) − 1 + (−1) 2m − 1

s−m p−1 ..
mà p | (42 )p−1 − 1 nên (−1) 2m = 1 suy ra v = p−1 2m
.2, trái với điều giả
1+s
sử. Vậy m > s suy ra p(s) = 2 .u + 1.
Đặt
X
k X
k X
k
1+si
Sk = p(si ) = (2 ui + 1) = 21+si ui + k.
i=1 i=1 i=1

Với s = min{si | i = 1, 2, ..., k} thì Sk = 21+s m + k, (s ≥ k). Từ đó


. . .
Sk ..2t tương đương với (21+s , +k)..2t hay k ..2t (vì t ≤ k ≤ s < 1 + s, suy ra
2r | 21+s ).
2.6. Đáp án chọn đội tuyển năm học 1996 - 1997 65

Bài 5. 1) Điều kiện để phương trình

4n2 x = log2 (2n2 x + 1) (1)

có nghĩa là x > − 2n1 2 . Với điều kiện đó ta có (1) tương đương với
2x
24n = 2n2 x + 1 (2)

Đặt 4n2 x + 1 = t, từ phương trình (2) ta có phương trình (ẩn t)

2t = t + 1 (3)

Với 0 < t < 1 ta có 2t < 2t + (1 − t) = t + 1.


Với t < 0, t > 1 ta có 2t > 2t + (1 − t) = t + 1.
Với t = 0, t = 1 ta có 2t = t + 1.
Như vậy, phương trình (3) có tất cả hai nghiệm là t = 0 và t = 1. Dẫn tới
phương trình (2), và cũng là (1) cũng có hai nghiệm xn = 0 và xn = − 4n1 2 .
2) Với xn = 0 ta có axn + bxn = a0 + b0 = 2 = 2 + 3xn ∀a, b > 0. Xét
xn = − 4n1 2 . Khi đó các bất đẳng thức sau là tương đương

axn + bxn ≥ 2 + 3xn ∀n ∈ N∗ .


1 − 12 1 3 1
(a 4n + b− 4n2 ) ≥ 1 + (− 2 ) ∀n ∈ N∗
2 2 4n
!−4n2  −4n2
3 1 
1 1
a− 4n2 + b− 4n2
≤ 1+ − 2 .
2 2 4n

Cho
√ n→ +∞, từ bất đẳng thức trên ta được ab ≤ e3/2. Ngược lại, với
3/2
ab ≤ e , ta có
1 − 12 1 √ 1 3 1 3 1
(a 4n + b− 4n2 ) ≥ ( ab)− 4n2 ≥ e 2 (− 4n2 ≥ 1 + (− 2 )
2 2 4n
hay axn + bxn ≥ 2 + 3xn .
3) Vậy, tất cả các cặp số thực dương a, b cần tìm là a, b > 0 sao cho
ab ≤ e3.

Bài 6. Trước hết, ta chứng minh khẳng định sau:


Khẳng định K. Cho n điểm phân biệt cùng nằm trên một đường thẳng.
Tô n điểm đói bởi hai màu xanh, đỏ sao cho có đúng k điểm được tô bởi
màu xanh, và giữa hai điểm màu xanh liên tiếp (tính từ trái qua phải) có ít
nhất p điểm được tô bởi màu đỏ (tính từ trái qua phải) có ít nhất p điểm
được tô bởi màu đỏ và ở bên phải điểm màu xanh cuối cùng có ít nhất p
điểm được tô bởi màu đỏ và ở bên phải điểm màu xanh cuối cùng có ít nhất

k
p điểm được tô bởi màu đó. Khi đó số cách tô màu khác nhau là n−kp
66 Chương 2. Đáp án tuyển sinh

Chứng minh. Lần lượt từ trái trái qua phải, gọi các điểm là 1, 2, ..., n. Đặt
tương ứng mỗi cách tô màu với bộ k số nguyên dương (i1 < i2 < · · · < ik ),
trong đó i1 , i2, ..., ik là các điểm được tô màu xanh. Dễ thấy, tương ứng nói
trên xác lập một song ánh từ tập gồm tất cả các cách tô màu tới tập

T = {(i1 < i2 < · · · < ik ) | ij ∈ {1, 2, ..., n − p} ∀i = 1, k;


ij+1 − ij > p ∀i = 1, k − 1

Xét ánh xạ

f : T → T 0 = {(j1 < j2 < · · · < jk ) | {1, 2, ..., n − kp} ∀t = 1, k}


(i1 < i2 < · · · < ik ) ∈ T 7→ (i1, i2 − p, ..., ik − (k − 1)p) ∈ T 0

Dễ chứng minh được f là song ánh từ T đến T 0. Từ đó, ta có điều phải


chứng minh.
2) Trở lại bài toán. Lần lượt, theo chiều kim đồng hồ, gọi các điểm là
A1, A2, ..., An. Gọi X là tập gồm tất cả các cách tô màu khác nhau. Xét
phân hoạch
X = X 0 ∪ X 00
trong đó X = {x ∈ X | trong x có điểm màu xanh thuộc{Ai, ..., Ap}},
X 00 = X\X 0 . Hiển nhiên, với x ∈ X 00 thì trong x không có điểm màu xanh
nào thuộc
 tập {A1, A2, ..., Ap}. Do đó, theo khẳng định K ta có cardX 00 =
k
n−kp
.
Xét X 0 . Kí hiệu Xi0 = {x ∈ X 0 | trong x có điểm Ai được tô màu xanh,
S
p
i = 1, p. Thế thì Xi0 ∩ Xj0 = ∅ ∀i 6= j ∈ {1, 2, ..., p} và X = Xi0 .
i=1
k−1

Với mỗi i = 1, p, theo khẳng định K, ta có cardXi0 = n−1−p−(k−1)p =
k−1
 0 k−1

. Do đó card X = p n−kp−1 .
n−kp−1
k
 k−1

Vậy cardX = n−kp + p n−kp−1

2.7 Đáp án chọn đội tuyển năm học 1997 -


1998
Bài 1.

Bổ đề 1. Nếu P (x) ∈ R[x] thoả mãn

|P (x)| ≤ cx1998 với x ∈ R c là hằng số (1)

thì P (x) = ax1998, với a là hằng số.


2.7. Đáp án chọn đội tuyển năm học 1997 - 1998 67

Nếu P (x) = a hằng số thì P (x) = a = 0. Giả sử deg p = n ≥ 1

P (x) = an xn + an−1 xn−1 + · · · + a1 x + a0 , an 6= 0.

Khi đó (1) tương đương với

|an xn + an−1 xn−1 + · · · + a1x + a0| ≤ cx1998 (2)

Ta sẽ chứng minh bằng phản chứng rằng n ≤ 1998. Thật vậy, nếu n > 1998
thì từ (2) ta có


an + an−1 + · · · + a0 ≤ c
, (∀x > 0)
x xn x n−1998

Cho n → ∞, suy ra an = 0, trái với giả thiết. Vậy n ≤ 1998.


Ta chứng minh ak = 0 với k = 0, 1, ..., n − 1 bằng quy nạp. Với k = 1
thì từ (2) cho x = 0 suy ra a0 = 0 đúng với k = 1. Giả sử a0 = a1 = · · · =
ak−1 = 0 và k < n. Thay vào (2) được

|an xn + · · · + ak xk | ≤ cx1998

hay
|an xn−k + · · · + ak+1 x + ak | ≤ cx1998−k ∀x > 0
Cho x → ∞0, vì 1998 − k ≥ n − k > 0 nên vế phải tiến tới 0, suy ra |ak | ≤ 0
hay ak = 0, đúng. Vậy P (x) = an xn , do đó |an xn | ≤ cx1998 với mọi n ≤ 1998
và an 6= 0. Dễ thấy n = 1998 vì nếu trái lại thì an = 0.
Bây giờ ta chứng minh bài toán. Lấy c = 1, đặt q(x) = p1 (x) ∈ R[x], ta
có với mọi  > 0 thì

|q(x) − p (x)| = |p1 (x) − p (x)|


≤ |f (x) − p1 (x)| + |(f (x) − p (x)|
≤ x1998 + x1998
= (1 + )x1998.

Do đó theo bổ đề
p (x) − q(x) = a x1998 (3)
Lại có
x1998 ≥ |f (x) − p (x)| = |f (x) − q(x) − a x1998| (4)
Từ (4) ta có
|h(x) − a x1998| ≤ x1998, ∀x ∈ R (5)
Cho x = 0, suy ra h(0) = 0. Từ (5) suy ra ∀x1 6= 0, x2 6= 0
68 Chương 2. Đáp án tuyển sinh


h(x1) h(x2 )

x1998 − a ≤ , x1998 − a ≤ .
1 2

Thành thử,

h(x1) h(x2)

x1998 − x1998 ≤ 2 ∀x1, x2 6= 0, ∀ > 0.
1 2

h(x1 ) h(x2 ) h(x)


Cho  → 0 thì x1998
= x1998
, tức là x1998
= c, c là hằng số, tức là
1 2

h(x) = cx1998 ∀x 6= 0 (6)

Vì h(0) = 0, suy ra h(x) = cx1998, ∀x ∈ R. Từ đó f (x) = q(x) + h(x) =


q(x) + cx1998 là một đa thức vì q(x) là đa thức.

Bài 2. Hai đường tròn C và C 0 tiếp xúc nhau tại A. Phép nghịch đảo
f tâm A, phương tích AI 2 = (2R)2 biến đường tròn C thành đường thẳng
d tiếp xúc với C tại I, biến đường tròn C 0 thành đường thẳng d0 vuông góc
với AI và cắt đường thẳng AI tại J , với AO.AJ = 4R2 , suy ra AJ = 4R,
và do đó IJ = 2R. f biến mỗi đường tròn Ci ∈ H thành đường tròn Γi ,
i = 1, 2, ..., n tiếp xúc với d và d0 , do đó Γi có bán kính R = IJ
2
. C∞ tiếp xúc
với C tại M1 , suy ra f (M1 ) = P1 là tiếp điểm của Γ1 và d, P1 = (AM1 ) ∩ d.

C\ tiếp xúc với C tại Mn , suy ra f (Mn ) = Pn là tiếp điểm của Γn


và d. Vì có n đường tròn C∞ , ..., C\ nên có n đường tròn Γ1 , Γ2 , ..., Γn với
Γi = f (Ci ). Suy ra P1 Pn = (n − 1)2R. Chú ý rằng P1 Pn = P1 I + IPn hoặc
P1 Pn = IP1 − IPn .
\1 = α (0 ≤ α < π ) thì IP1 = 2R tan α.
Bây giờ ta tính IP1. Đặt IAM 2
Sử dụng định lý hàm số Cosine trong tam giác OO0 O1 cho ta

O0 O12 = OO02 + OO12 − 2OO0 .OO1 cos(π − 2α).


2.7. Đáp án chọn đội tuyển năm học 1997 - 1998 69

Từ đó ta có các đẳng thức tương đương sau


R R 2 R2  1 2 R 1
+ = + R2 1 − +2. .R(1 − ) cos 2α
2 p1 4 p1 2 p1
 p + 2 2 1  p − 1 2 p − 1
1 1 1
= + + cos 2α.
2p1 4 p1 p1
(p1 + 2)2 = p21 + 4(p1 − 1)2 + 4p1 (p1 − 1) cos 2α
3 − p1
cos 2α = . (1)
p1 − 1

Đặt tan α = t thì


1 − t2
cos 2α = (2)
1 + t2
Từ (1) và (2) ta có

(3 − p1 )(1 + t2) = (p1 − 1)(1 − t2 )


√ √
suy ra tan2 α = p1 − 2. Vậy IP1 = 2R p1 − 2. Tương tự IPn = 2R pn − 2.
Do IP1 ≥ 0 và IPn ≥ 0 nên p1 ≥ 2 và pn ≥ 2.

Điều kiện
√ cần. Chú ý rằng P 1 Pn = IP1 − IPn . và IP1 = 2R p1 − 2,
IPn = 2R pn − 2 suy ra điều kiện cần và đủ để có họ C1, C2 , ..., Cn thuộc
H là p p
2R p1 − 2 + 2R pn − 2 = (n − 1)2R
hay p p
p1 − 2 + pn − 2 = n − 1 (2a)
khi I nằm trong [p1, pn ].
p p
|2R p1 − 2 − 2R pn − 2| = (n − 1)2R.

hay p p
| p1 − 2 − pn − 2| = n − 1 (2b)
khi I nằm ngoài (p1 , pn ).
Bình phương hai vế của (2a) và (2b) ta được
p
± (p1 − 2)(pn − 2) = (n − 1)2 − (p1 − 2) − (pn − 2)

Lại bình phương hai vế và rút gọn ta được

(p1 − pn )2 = (n − 1)2 (2(p1 + pn ) − (n − 1)2 − 8) (3)


70 Chương 2. Đáp án tuyển sinh

Điều kiện đủ. Giả sử ta có (3). Coi p1 = p là ẩn và pn là tham số thì (3)


tương đương với

p2 − 2p((n − 1)2 + pn ) + p2n − 2pn (n − 1)2 + (n − 1)2 ((n − 1)2 + 8) = 0.

Để phương trình này có nghiệm thì ∆ ≥ 0 hay

((n − 1)2 + pn )2 − (p2n − 2pn (n − 1)2 + (n − 1)4 + 8(n − 1)2 ) ≥ 0.

Giải bất phương trình này cho ta pn ≥ 2, vì n ≥ 3.


Tương tự, nếu ta coi pn là ẩn thì suy ra p1 ≥ 2.
Nếu có số pi√≥ 2, (i = 1, n) thì trên tia gốc I của đường thẳng d có điểm
Pi để IPi = 2 pi − 2. Xét dãy đường tròn ξ1 , ξ2 , ..., ξn lần lượt tiếp xúc
ngoài với nhau và tiếp xúc với cả hai đường thẳng d, d0 . Phép nghịch đảo
tâm A, phương tích 4R2 biến dãy đường tròn ξ1 , ξ2 , ...ξn thành dãy đường
tròn C∞ , C∈ , ..., C\ lần lượt tiếp xúc nhau và tiếp xúc với hai đường tròn
C, C 0 .

Bài 3. Bất đẳng thức cần chứng minh tương đương với mỗi bất đẳng
thức sau
nX k 
1 1 o
exp + ≥ ln m.
i=1
pi p2i
1 1
Yk + 2
e pi pi > ln m.
i=1

Để chứng minh bất đẳng thức trên ta sử dụng các bổ đề


1
Bổ đề 1. Chứng minh rằng nếu 0 < x ≤ 2
thì

2 1
ex+x >
1−x
2
Ta viết bất đẳng thức cần chứng minh dưới dạng (1 − x)ex+x > 1. Xét
2
hàm số f (x) = (1 − x)ex+x trên [0, 12 ]. Ta có f (0) = 1, và tính đạo hàm
2
f 0 (x) = xex+x (1 − 2x). Dễ thấy rằng f 0 (x) > − với mọi 0 < x ≤ 12 . Từ đó
ta có
f (x) > f (0) = 1.
Từ bổ đề 1 suy ra
1 1
+ 2  1 −1
e pi pi > 1 − .
pi
2.7. Đáp án chọn đội tuyển năm học 1997 - 1998 71

hay là
1 1 k 
Y + 2 Y 1 −1
k
e pi p i > 1− .
i=1 i=1
pi

Bổ đề 2. Chứng minh bất đẳng thức, m, n, k ∈ N


k 
Y 1 −1 X 1
n
1− > .
i=1
pi m=1
m

Dễ thấy
 1 −1 1 1
1− > 1 + + · · · + s , ∀s ∈ N.
pi pi pi
Giả sử s ∈ N thoả mãn 2s ≤ n < 2s+1 . Khi đó
k 
Y 1 −1 Y 1
k
1
A= 1− > 1 + + ··· + s
i=1
pi i=1
pi pi
X 1
= ,
pα1 1 · · · pαk k

trong đó 0 ≤ αi ≤ s. Tổng chạy trên tất cả các bộ (α1 , ..., αs) với 0 ≤ αi ≤ s.
Mặt khác với mỗi m ≤ n, ta có

m = pα1 1 pα2 2 · · · pαk k với αi ≥ 0

mà 2s+1 > n ≥ m > pαi i ≥ 2αi . Do đó αi < s + 1 suy ra αi ≤ s ∀i.


1
Vậy m là một số hạng của tổng A. Vậy

X 1 Xn
1
α1 αk > .
p1 ...pk m=1
m

Bài toán được chứng minh nếu ta chứng minh được rằng

Bổ đề 3.
Xn
1
> ln n.
m=1
m

Thật vậy,
Z m+1 Z m+1
dx dx 1
< = ∀m = 1, 2, ...
m x m i m
72 Chương 2. Đáp án tuyển sinh

Suy ra
Xn Xn Z m+1 Z n+1
1 dx dx
> = = ln(1 + n) > ln n.
m=1
m m=1 m x 1 x

Bây giờ chứng minh


Xn
1
> ln(n + 1).
m=1
m
 1 n  1 n  n
1
limn→∞ 1 + = e và dãy 1 + tăng nên 1 + n < e ∀n ≥ 1.
 n n
1
Suy ra n ln 1 + < 1 hay
n
1  1
> ln 1 + , ∀n ≥ 1.
n n

Xn
1 1 1 1 1 1 1
= 1 + + + · · · + > ln 2+ln(1 + ) +ln(1 + ) + · · · +ln(1 + )
m=1
m 2 3 n 2 3 n
3 4 n+1 2.3.4.n(n + 1)
= ln 2 + ln + ln + · · · + ln = ln = ln(n + 1).
2 3 n 1.2.3...n

Bài 4. Rõ ràng đa thức bậc 0, P (x) = 1 và đa thức bậc nhất không có


tính chất trên. Xét đa thức

P (x) = xn + · · · + a1 x + a0

với ai ∈ Z, n > 1. Ta chứng minh P (x) luôn có tính chất đã nêu.


Ký hiệu

A = {p ∈ P : P > P (1) − P (0), P > −P (0)}

Khi đó |A| = ∞ với mọi p ∈ A, xét đa thức

Qo (x) = P (x) − P (0) − p

Vì limx→∞ Qp (x) = +∞, Qp (1) = P (1) − P (0) − p < 0 nên ∃xp ∈ R, xp > 1
để Q(xp) = 0 hay

P (xp ) = P (0) + p ∈ N∗ , ∀p ∈ A.

Có hai trường hợp sau


2.7. Đáp án chọn đội tuyển năm học 1997 - 1998 73

1) Nếu tồn tại vô số p ∈ A để xp ∈ / Q thì bài toán được chứng minh. (Vì
nếu p1 6= p2 ∈ A thì P (xp1 ) 6= P (xp2 ) do đó xp1 6= xp2 ).
2) Nếu chỉ có hữu hạn p ∈ A để xp ∈ / Q thì tồn tại p0 để xp ∈ Q, ∀p ≥ p0 ,
p ∈ A. Khi đó xp là nghiệm hữu tỉ của Qp(x) → xp ∈ Z và xp | Qp (0) = −p.
Mà xp > 1 nên xp = p. Vậy ∀p ≥ p0 , p ∈ A : Qp(p) = 0 hay P (p) = P (o)+p
hay P (x) = P (0) + x, mâu thuẫn vì deg P > 2.

Bài 5. Điều kiện cần. Giả sử d = 2x2 + 2xy + 3y 2. Đặt N = 5 + 19981998


Vì N lẻ, N không chia hết cho 5 nên d lẻ và d không chia hết cho 5. Ta có
.
d = 2x(x + y) + 3y 2. d lẻ, suy ra y lẻ. Nếu x chẵn thì 2x(x + y)..4. Nếu x lẻ
.
thì 2x(x + y)..4 .
Vậy luôn có d ≡ 3y 2 ≡ 3 (mod 4). Lại có d = 2x2 + 2xy + 3y 2 suy ra
2d = (2x + y)2 + 5y 2 do đó 2d ≡ (2x + y)2 (mod 5). Dẫn đến 2d ≡ {0, ±1}
(mod 5) suy ra 4d ≡ ±2 (mod 5) do đó d ≡ ±2 (mod 5).
Nếu ( (
d ≡ 2 (mod 5) d ≡ 7 (mod 5)
hay
d ≡ 3 (mod 4) d ≡ 7 (mod 4)

thì d ≡ 7 (mod 20).


Nếu (
d≡3 (mod 5)
d≡3 (mod 4)

thì d ≡ 3 (mod 20).


Điều kiện đủ. Kí hiệu 1998999 . Khi đó N = a2 + 5. Xét tập
√ √
A = {ax + y}, x = 0, 1, 2, ..., [ d], y = 0, 1, 2, ..., [ d]

Đặt q = [ d], ta có √
q ≤ d < q + 1.
Vì |A| = (q + 1)2 > d nên ∃(x1, y1), (x2 , y2) với (x1, y1 ) 6= (x2 , y2) để

ax1 + y ≡ ax2 + y2 (mod d)

suy ra a(x1 − x2 ) + y1 − y2 ≡ 0 (mod d). Đặt x0 = x1 − x2, y0 = y1 − y2 thì

ax0 + y0 ≡ 0 (mod d).

Do đó a2 x20 − y02 = (ax0 + y0)(ax0 − y0 ) ≡ 0 (mod d). Mà a2 ≡ −5


(mod d), suy ra, −5x20 − y02 ≡ 0 (mod d), hay
.
5x20 + y02..d (1)
74 Chương 2. Đáp án tuyển sinh
√ √
Lại có |x0 | = |x1 − x2| ≤ q ≤ d, |y0| = |y1 − y2| ≤ q ≤ d. Do đó
x20 ≤ d, y02 ≤ d. Vậy
5x20 + y02 ≤ 6d (2)
Từ (1) và (2) suy ra

5x20 + y02 = kd với k = 0, 1, 2, 3, 4, 5, 6.

Ta chứng minh rằng không xảy ra k = 0, 1, 4, 5, 6.


Thậy vậy,
Nếu 5x20 + y02 = 0 thì x0 = y0 = 0 suy ra x1 = x2 và y1 = y2 , loại.
Nếu 5x20 + y02 = d thì yo2 ≡ d (mod 4) dẫn đến y02 ≡ ±2 (mod 5), loại.
Nếu 5x20 + y02 = 4d thì y02 ≡ 4d (mod 5), dẫn đến y02 ≡ −d (mod 5) và
y02 ≡ ±2 (mod 5), loại.
Nếu 5x20 +y02 = 5d thì y0 = 5y1, do vậy 5x20 +25y12 = 5d suy ra x20 +5y12 = d
hay x20 ≡ d ≡ ±2 (mod 5) loại.
Nếu 5x20 + y02 = 6d thì y02 ≡ 6d (mod 5) suy ra y02 ≡ d (mod 5) hoặc
2
y0 ≡ ±2 (loại)
Vậy chỉ xảy ra  2
5x0 + y02 = 2d
5x20 + y02 = 3d
Nếu 5x20 + y02 = 2d suy ra x20 + y02 ≡ 2d (mod 4). Vì d ≡ 3 (mod 4) suy
ra x20 + y02 ≡ 2 (mod 4) nên x0, y0 phải cùng lẻ. Đặt y0 − x0 = 2y ta có
y0 = x0 + 2y suy ra

5x20 + y02 = 5x20 + (x0 + 2y)2 = 2d


→ 6x20 + 4x0 y + 4y 2 = 2d
→ 3x20 + 2x0 y + 2y 2 = d
→ d = 2y 2 + 2yx0 + 3x20 biểu diễn được
.
Nếu 5x20 + y02 = 3d suy ra y02 − x20 ≡ 0 (mod 3) hay (y0 − x0 )(y0 + x0)..3
.
Nếu y0 + x0 ..3 suy ra x0 + y0 = 3y hay

y0 = 3y − x0 → 5x20 + y02 = 5x20 + (3y − x0 )2 = 6x20 − 6yx0 + 9y02 = 3d


→ 2x20 − 2yx0 + 3y02 = d
→ 2x20 + 2(−y)x0 + 3(−y)2 = d biểu diễn được

Nếu y0 − x0 = 3y suy ra y0 = x0 + 3y khi đó ta có

5x20 + y02 = 5x20 + (x0 + 3y)2 = 6x20 + 6x0 y + 9y 2 = 3d


→ d = 2x20 + 2x0 y + 3y 2
2.7. Đáp án chọn đội tuyển năm học 1997 - 1998 75

Điều phải chứng minh.

Bài 6. Chuyển bài toán sang ngôn ngữ Graph, trong đó mỗi người coi
là một điểm trên mặt phẳng, còn quan hệ quen nhau coi là một cạnh (1
đoạn thẳng với giả thiết rằng các đoạn thẳng này không cắt nhau trừ hai
điểm đâu mút), ta có graph G đơn, vô hướng với tập đỉnh h gồmi n điểm
n+2
P = {A1, A2, . . . , An} và bậc của đỉnh A bất kỳ là d(A) ≥ 3 .
Điều kiện "hai người bất kỳ quen nhau hoặc quen nhau gián tiếp" chứng
tỏ Graph G là liên thông.
Trong G (hữu hạn) xét đường gấp khúc nhiều cạnh nhất P0 , giả sử P0
có k đỉnh là P0 = {A1, A2 , . . . , Ak } với Ai Ai+1 (i = 1, 2, . . . , k − 1) là các
cạnh (Ai kề với Ai+1 ).
Do điều kiện (3) thì k ≤ n − 1.
Gọi N (A) là tập các đỉnh kề với đỉnh A. Ta có

N (A1 ) ⊂ {A2, . . . , Ak } và N (Ak ) ⊂ {A1, . . . , Ak−1 }

vì trái lại thì tồn tại đường gấp khúc khác có nhiều cạnh hơn P0 .
Giả sử N (Ai ) = {Ai1 , Ai2 , . . . , Ais }, i ∈ {1, 2, . . . , n} ký hiệu

N (Ai)+ = {Ai1+1 , Ai2 +1 , . . . , Ais+1 }


N (Ai)− = {Ai1−1 , Ai2 −1 , . . . , Ais−1 }

Do k ≤ n − 1 nên tồn tại đỉnh B ∈ / P0 . Ta có N (B) ∩ N (Ak )+ = ∅


Thật vậy nếu ∃Aj ∈ N (B) ∩ N (Ak )+ thì tồn tại đường gấp khúc
(A1, . . . , Aj−1, Ak , Ak−1 , . . . , Aj+1 , Aj , B) có k + 1 cạnh, trái giả thiết đối
với P0 . Lập luận tương tự có N (B) ∩ N (A1)− = ∅. Ta cũng có N (A1)− ∩
N (Ak )+ 6= ∅ vì nếu trái lại thì

|N (B) ∪ N (A1 )− ∪ N (Ak )+ | = |N (B)| + |B(A1)− | + |N (Ak )+ | ≥


hn + 2i n + 2 
≥3 >3 −1 =n−1
3 3
76 Chương 2. Đáp án tuyển sinh

suy ra số đỉnh của tập hợp này lớn hơn hoặc bằng n mà tập hợp đó không
chứa đỉnh B. Mâu thuẫn.
Vậy ∃Ai ∈ N (A1)− ∩ N (Ak )+
Khi đó tồn tại đường gấp khúc khép kín có k − 1 đỉnh thuộc tập Pc \{Ai}
là (A1, A2, . . . , Ai−1, Ak , Ak−1 , . . . , Ai+1 )

Tập còn lại chứa các đỉnh đôi một không kề nhau (không có đoạn thẳng
nối chúng) vì nếu trái lại, chẳng hạn có B1 , B2 ∈
/ P0 \{Ai} mà B1 kề với B2
do tính liên thông tồn tại đường gấp khúc chứa B1 , B2 và P0 \{Ai } có nhiều
cạnh hơn P0 , mâu thuẫn.

2.8 Đáp án chọn đội tuyển năm học 2001 -


2002
Bài 1. Trước hết ta chứng minh H thuộc tia MB. Thật vậy, giả sử ngược
lại H thuộc tia MC (xem hình 1). Khi đó gọi K là trung điểm của AB ta có
BK = KH = AB/2 = HM và KM//AC. Suy ra: M c1 = Kc1 = (1800 − B)/2
b
và M b Do đó:
c2 = C.

c1 + M
1800 = M c2 = (1800 − B)/2
b b
+C

b = 900 + (B)/2
⇔C b
b là góc nhọn. Vì vậy ta có điều phải
Điều này mâu thuẫn với giả thiết C
chứng minh.
Gọi A1 và H1 tương ứng là điểm đối xứng với A và H qua trung trực
của BC. (Xem hình 2). Ta có:

AA1 = HH1 = 2HM = AB = A1C ⇒ \


A \ [
1 CA = A1 AC = ACB

⇒ ABC \1 = 2ACB
[ = BCA [

⇒ \
A [ \ 0 [ [
1 AN = (BAC)/3 + A1 AC = (180 − 3ACB)/3 + ACB = 60
0
2.8. Đáp án chọn đội tuyển năm học 2001 - 2002 77

Suy ra 4ANA1 là tam giác đều ⇒ NA = NA1 = AA1 = AB =


N P ⇒ N là tâm đường tròn ngoại tiếp 4AP A1 ⇒ AP \ \1)/2 =
A1 = (ANA
[
300 ⇒ (BAC)/3 =P \ AN = NP \ \
A = (AP A1)/2 = 150 ⇒ BAC[ = 450 ⇒
[ = 450 và ABC
ACB [ = 900 . Như vậy, 4ABC là tam giác vuông cân tại B.
Ngược lại, dễ thấy 4ABC vuông cân tại B thoả mãn tất cả các yêu cầu
của đề bài.
Tóm lại, tất cả các tam giác ABC phải tìm là các tam giác vuông cân
tại B.
Bài 2. Trước hết ta nhận xét rằng, với mỗi giá trị N0 cho trước thì hoặc
người A hoặc người B sẽ có chiến lược thắng cuộc.
Xét hàm số f : N∗ → {0, 1} xác định như sau:
f (n) = 1 nếu người A có chiến lược thắng cuộc khi số ghi ở thời điểm
ban đầu là n.
f (n) = 0 nếu người B có chiến lược thắng cuộc khi số ghi ở thời điểm
ban đầu là n.
Dễ thấy: f (1) = f (2) = 1, f(3) = 0 và với mỗi n ≥ 3 ta có:
+) Nếu f (n − 1) = 0 hoặc f ([n/3]) = 0 thì f (n) = 1
+) Nếu f (n − 1) = 1 và f ([n/3]) = 1 thì f (n) = 0
Suy ra: f (1) = f (2) = 1, f(3) = 0 và
f (n) = 1 − f (n − 1)f ([n/3]) ∀n ≥ 3 (1)
Từ đó với mọi k ≥ 1 ta có:
f (3k) = 1 − f (3k − 1)f (k)
f (3k + 1) = 1 − f (3k)f (k) = 1 − (1 − f (3k − 1)f (k))f (k)
= 1 − f (k) + f (3k − 1)f (k)

Cộng vế theo vế hai đẳng thức trên ta được:


f (3k) + f (3k + 1) + f (k) = 2 ∀k ≥ 1 (2)
Hoàn toàn tương tự ta chứng minh được:
f (3k + 1) + f (3k + 2) + f (k) = 2 ∀k ≥ 1 (3)
Từ đó suy ra: f (3k + 2) = f (3k) ∀k ≥ 1 Hơn nữa, từ (1),(2) và (3) ta
còn có: ∀k ≥ 1 nếu f (k) = 0 thì f (k + 1) = 1 và f (3k) = f (3k + 1) =
f (3k + 2) = 1. Suy ra, nếu f (k) = 0 thì f (3k + 3) = 0. Từ đây, vì f (3) = 0
nên f ((3k − 3)/2 = 0 ∀k ≥ 2. Do đó:
f (120) = f ((35 − 3)/2) = 0
f ((32002 − 1)/2 = 1 − f ((32002 − 3)/2)f ([(32002 − 1)/6]) = 1
f ((32002 + 1)/2) = f (2 + (32002 − 3)/2 = f ((32002 − 3)/2) = 0
78 Chương 2. Đáp án tuyển sinh

Vậy:
Nếu N0 = 120 thì người B có chiến lược thắng cuộc.
Nếu N0 = (32002 − 1)/2 thì người A có chiến lược thắng cuộc.
Nếu N0 = (32002 + 1)/2 thì người B có chiến lược thắng
√ cuộc.
Bài 3. Dễ thấy, nếu gọi p là ước nguyên tố lớn hơn 2m + 1 của m thì
p là duy nhất và
p>3 (*)
Xét số nguyên dương M mà đối với nó tồn tại tập T = {x1 = m, x2; . . . ; xk =
M} thoả mãn các điều √ kiện của đề bài. Ta sẽ chứng minh: M ≥ m + p.
Thật vậy, vì p > 2m + 1 nên p có số mũ 1 trong phân tích chuẩn của
m. Do đó từ giả thiết tích x1x2 . . . x + k là số chính phương suy ra trong
các số x2 , . . . , xk phải có ít nhất 1 số là bội của p. Vì vậy M = xk ≥ m + p.
Với M = m + p, xét tập T = {m; (m/p)(p + 1); ((2m + p)/2p)(p −
1); ((2m + p)/2p)(p + 1); ((m + p)/p)(p1 ); m + p}. Từ (*) dễ dàng chứng
minh được:
+) Tất cả các số thuộc T đều là số nguyên dương.
+) m < (m/p)(p + 1), ((2m + p/2p)(p − 1), ((2m + p)/2p)(p + 1), ((m +
p)/p)(p − 1) < m + p
+) Tích tất cả các số thuộc T là số chính phương.
Hơn nữa, nếu trong bốn số (m/p)(p + 1), ((2m + p)/2p)(p − 1), ((2m +
p)/2p)(p + 1), ((m + p)/p)(p − 1) có hai số trùng nhau thì khi bỏ hai số đó
ra khỏi tập T ta sẽ nhận được tập T1 có đầy đủ các tính chất nêu trên.
Các lập luận nêu trên chứng tỏ với M = m + p tồn tại tập T thoả mãn
tất cả các điều kiện của đề bài.
Vậy, số nguyên √ dương M nhỏ nhất cần tìm là m + p, trong đó p là ước
nguyên tố lớn hơn 2m + 1 của m.
Bài 4. Xét số k bất kỳ thoả mãn điều kiện của đề bài. Với mỗi bộ
(i1, i2, . . . , ik ) mà
1 ≤ i 1 < i2 < · · · < ik ≤ n (*)
Ký hiệu M(i1 , . . . , ik ) là số cột chỉ gồm các ô được đánh dấu của bảng k ×2n
tạo nên từ k hàng i1, i2, . . . , ik . Đặt:
X
M= M(i1 , . . . , ik ) (1)

ở đây tổng lấy theo tất cả các bộ (i1 , i2, . . . , ik ) thoả mãn (*).
Với mỗi i ∈ {1, 2, . . . , 2n, gọi mi là số hàng (của bảng n × 2n) có ô thứ i
được đánh dấu và gọi si là số bảng k × 2n có cột thứ i chỉ gồm các ô được
đánh dấu. Ta có:
X
2n
si = Cm ∀i ∈ {1, 2, . . . , 2n}, mi = n2
i=1
2.8. Đáp án chọn đội tuyển năm học 2001 - 2002 79


X
2n X
2n X
2n
M= si = Cm ≥ (mi −k+1) = n2 −2kn+2n = n(n−2k+2) (2)
i=1 i=1 i=1

Từ (1) và (2) suy ra tồn tại (i1, i2 , . . . , ik ) sao cho:


k!(n − 2k + 2)
M(i1 , i2, . . . , ik ) ≥ (n(n − 2k + 2))/Cn =
(n − k + 1)(n − k + 2) . . . (n − 1)
(Đpcm)
Bài 5. Yêu cầu của bài ra tương đương với việc tìm tất cả các cặp đa
thức P (x), H(x) ∈ Z[x] sao cho:

(H(x))2 − (x2 + 6x + 10)(P (x))2 = −1 (1)

Nhận thấy, nếu cặp đa thức (P, H) thoả mãn (1) thì các cặp đa thức
(−P, H), (P, −H), (−P, −H) cũng thoả mãn (1). Vì vậy ta chỉ cần xét
các cặp đa thức (P, H) với P, H thuộc tập hợp Z + [x] các đa thức có hệ số
cao nhất√là số nguyên dương.
Đặt x2 + 6x + 10 = α. Từ tính bất khả quy của tam thức f (x) =
2
x + 6x + 10 ta có thể dễ dàng chứng minh, bằng phương pháp phản chứng,
rằng α không thể biểu diễn dưới dạng A(x)/B(x), trong đó A(x) và B(x)
là các đa thức với hệ số thực. Từ đó suy ra với mỗi n ∈ N, tồn tại duy nhất
cặp đa thức M(x), T (x) ∈ Z[x] sao cho:

(x + 3 + α)2n+1 = M(x) + αT (x)

Tiếp theo ta có các nhận xét sau:


Nhận xét 1: Nếu M(x), T (x) ∈ Z[x] và

(x + 3 + α)2n+1 = M(x) + αT (x), n∈N (2)

thì (M(x))2 − (x2 + 6x + 10)(T (x))2 = −1.


Chứng minh. Với n ∈ N, từ tính duy nhất của biểu diễn (2) ta có

(x + 2 − α)2n+1 = M(x) − αT (x) (3)

Nhân (2) với (3) vế theo vế, ta được điều cần chứng minh.
Nhận xét 2: Nếu đa thức hằng S(x) và đa thức G(x) ∈ Z + [x] thoả
mãn hệ thức

(G(x))2 − (x2 + 6x + 10)(S(x))2 = 1 ∀x ∈ R

thì S(x) = 0 và G(x) = 1.


80 Chương 2. Đáp án tuyển sinh

Chứng minh. Dễ thấy phải có degG ≤ 1. Từ đây dễ dàng suy ra điều


cần chứng minh.
Nhận xét 3: Nếu P (x), H(x) ∈ Z + [x] thoả mãn (1) thì tồn tại số tự
nhiên n sao cho P (x) = T (x) và H(x) = M(x), trong đó M(x) và T (x)
được xác định theo (2).
Chứng minh. Dễ thấy tồn tại số tự nhiên n sao cho

degT ≤ degP < degT ∗ (*)

trong đó T ∗(x) là đa thức được xác định theo (2) ứng với n + 1. Xét các đa
thức H ∗ (x) và P ∗ (x) xác định bởi:

(H(x) + αP (x))(M(x) − αT (x)) = H ∗ (x) + αP ∗ (x)

Ta có: (H(x))2 − (x2 + 6x + 10)(P ∗ (x))2 = 1.


Mặt khác, lại có:

P ∗ (x) = P (x)M(x)−H(x)T (x) = [(T (x))2−(P (x))2 ]/[P (x)M(x)+H(x)T (x)]


(4)
Vì degT = 2n và degT ∗ = 2n +2 nên từ (*) ta thấy có thể xảy ra hai trường
hợp sau:
Trường hợp 1: degP = degT = 2n. Khi đó deg[(T (x))2 − (P (x))2 ] ≤ 4n
và deg[P (x)M(x) + H(x)T (x)] = 4n + 1. Vì vậy (4) tương đương với

(T (x))2 − (P (x))2 = 0 và P ∗ (x) = 0.

Trường hợp 2: degP = degT + 1 = 2n + 1.deg[(T (x))2 − (P (x))2 ] =


4n + 2 và deg[P (x)M(x) + H(x)T (x)] = 4n + 2. Vì thế từ (5.4) suy ra
P ∗ (x) là đa thức hằng. Do đó, theo nhận xét 2, ta phải có P ∗ (x) = 0 và
(T (x))2 − (P (x))2 = 0. Suy ra degP = degT , trái với giả thiết ban đầu.
Mâu thuẫn nhận được cho thấy trường hợp này không thể xẩy ra.
Nhận xét 3 được chứng minh.
Từ ba nhận xét nêu trên suy ra tất cả các đa thức P (x) cần tìm theo
yêu cầu của đề ra:

P (x) = ±((x + 3 + α)2n+1 − (x + 3 − α)2n+1 )/2α

Bài 6. Với mỗi tập hữu hạn T các số nguyên dương đặt:
Y X
D(T ) = t− t2

trong đó tích và tổng lấy theo tất cả các số t thuộc T .


2.9. Đáp án chọn đội tuyển năm học 2003 - 2004 81
Q
Xét tập A0 = {1, 2, 4, . . . , 22002}. Dễ thấy max A0 < a−
Q 1 (tích lấy
theo tất cả a ∈ A0 ) và D(A0 ) > 0. Xây dựng tập A1 = A0 ∪ { a − 1}. Ta
có |A1| = |A0| + 1 và
Y X Y Y X Y
D(A1 ) = a− a2 = a( a−1)−( a2 +( a−1)2 ) = D(A0 )−1 > 0

Trên cơ sở tập A1 ta xây dựng tập A2 theo các xây dựng A1 trên cơ sở
A0. Quá trình xây dựng trên được tiếp tục một khi ta còn nhận được tập
Ai có D(Ai ) > 0. Vì sau mỗi lần xây dựng số phần tử của Ai tăng 1 và
D(Ai ) giảm 1 nên tồn tại k sao cho |Ak | > 2002 và D(Ak ) = 0. Giả sử
Ak = {a1 < a2 < · · · < am }. Khi đó am sẽ là nghiệm nguyên dương của tam
thức với hệ số nguyên:

Y 
 m−1 X
m−1
2
f (X) = X − ai X + a2
i=1 i=1

Từ đó suy ra biệt thức 4 của f phải là số chính phương và ta có điều phải


chứng minh.

2.9 Đáp án chọn đội tuyển năm học 2003 -


2004
Bài 1. Xét tập hợp

S0 = {p1 , p2 , ..., p1001, p1002, p1 .p1003, p2 .p1004, ..., p1001.p2003, p1002.p2004}

trong đó p1 , p2 , ..., p2003, p2004 là 2004 số nguyên tố phân biệt lớn hơn 1.
Dễ thấy tập S0 nêu trên có các tính chất như đề bài yêu cầu. Hơn nữa,
rõ ràng trong 1002-tập con {p1 , p2 , ..., p1001, p1002} của S0 không có hai số
nào mà ước chung lớn nhất của cúng khác 1. Suy ra k ≥ 1003.
Xét một tập S tuỳ ý thoả mãn các yêu cầu của đề bài. Với mỗi số s ∈ S,
ký hiệu g(s) là số các số thuộc S không nguyên tố cùng nhau với s. Từ các
giả thiết của bài toán suy ra g(s) ≥ 1 và g(s) = g(t) với mọi s, t ∈ S. Nghĩa
là g(s) = m ∀s ∈ S, trong đó m là một hằng số nguyên dương.
Xét một 1003-tập con T tuỳ ý của S. Ta sẽ chứng minh rằng trong T
tồn tại hai số phân biệt mà ước số chung lớn nhất của chúng khác 1.
Thật vậy, giả sử ngược lại rằng trong T không có hai số nào mà ước số
chung lớn nhất của chúng khác 1. Khi đó, kí hiệu

A = {a ∈ S | ∃t ∈ T, (a, t) 6= 1}
82 Chương 2. Đáp án tuyển sinh

ta có A ∩ T = ∅. Và vì thế

|A| ≤ 2004 − 1003 = 1001 (1)

Mặt khác, do số số thuộc S (kể cả lặp) mà mỗi số đều không nguyên tố


cùng nhau với ít nhất một số thuộc T là 1003m và mỗi số bị tính lặp tối đa
m lần, nên
1003.m
|A| ≥ = 1003 (2)
m
Mâu thuẫn giữa (1) và (2) chứng tỏ điều giả sử ở trên là sai. Vì thế, ta có
điều muốn chứng minh.
Tóm lại, từ các chứng minh trên ta được kmin = 1003.

Bài 2. i) Dễ thấy, nếu α = 0 thì các hàm số f (x) ≡ 0 và f (x) ≡ 1 là


các hàm số thoả mãn các điều kiện của đề bài. Vì thế α = 0 không phải là
giá trị cần tìm.
ii) Xét α 6= 0.
Thay x = 0 vào hệ thức của đề bài

f (x2 + y + f (y)) = (f (x))2 + αy (1)

ta được
f (x + f (y)) = (f (0))2 + αy ∀y ∈ R (2)
Từ đó suy ra f là toàn ánh từ R lên R. Do đó, tồn tại x0 sao cho f (x0 ) = 0.
Thay y = 0 vào (1), ta được

f (x2 + f (0)) = (f (x))2 ∀y ∈ R (3)

Lần lượt thay x = x0 và x = −x0 vào (3), ta được

0 = f (x20 + f (0)) = (f (−x0))2 .

Từ đây suy ra f (−x0 ) = 0. Từ đó, lần lượt thay y = x0 và y = −x0 vào (2),
ta được
(f (0))2 + αx0 = 0 và (f (0))2 + α(−x0 ) = 0
hay f (0) = 0 và x0 = 0. Như vậy,

f (x) = 0 ⇔ x = 0 (4)

Do đó, từ (2) ta có
f (y + f (y)) = αy ∀y ∈ R (5)
và từ (3) ta có
f (x2) = (f (x))2 ∀x ∈ R (6)
2.9. Đáp án chọn đội tuyển năm học 2003 - 2004 83

Từ (1) và (6) suy ra

f (x + y + f (y)) = f (x) + αy ∀x ≥ 0, ∀y ∈ R (7)

Xét X ≥ 0 tuỳ ý. Trong (7), thay x = X, y = − f (X)


α
, ta được
 f (X)  f (X) 
f X− +f − =0
α α
Từ đó, theo (4) ta có

f (X)  f (X) 
X− +f − =0
α α
hay
f (X)  f (X) 
−X = − +f − (8)
α α
Suy ra
 f (X)   f (X)   f (X) 
f (−X) = f − +f − =α − = −f (X)
α α α
Từ đó, do X tuỳ ý và do (4), suy ra f (x) là hàm số lẻ trên R. Từ (8), do
X ≥ 0 tuỳ ý, suy ra hàm số g(x) = x + f (x), ∀x ∈ R, nhận mọi giá trị thực
không dương. Mà, dễ thấy, g(x) là hàm số lẻ trên R nên suy ra g(x) nhận
mọi giá trị thực. Vì thế, hàm số g là một toàn ánh từ R lên R.
Xét x ≥ 0, y ∈ R tuỳ ý. vì g là một toàn ánh từ R lên R nên tồn tại
y0 ∈ R sao cho y0 + f (y0 ) = y. Do đó

f (x + y) = f (x + y0 + f (y0 )) = f (x) + αy0


= f (x) + f (y0 + f (y0 ) = f (x) + f (y)

Suy ra
f (x + y) = f (x) + f (y), ∀x, y ≥ 0 (9)
Hơn nữa từ (6) và (4) suy ra f (x) > 0, ∀x > 0. Từ đó với x > y ≥ 0, ta
có 0 < f (x − y) = f (x) − f (y), hay f (x) > f (y). Như vậy

f (x) là hàm số đồng biến trên R+ (10)

Từ (9) và (10) suy ra f (x) = ax ∀x ∈ R+ trong đó a là một hằng số thực.


Từ đây, vì f là hàm số lẻ trên R nên f (x) = ax, ∀x ∈ R.
Ngược lại, thay f (x) = ax vào (1) ta được

a(x2 + y + ay) = a2x2 + αy, ∀x, y ∈ R.


84 Chương 2. Đáp án tuyển sinh

Suy ra a = a2 và a + a2 = α, hay a = 1, và α = 2, do α 6= 0. Dễ thấy, với


α = 2 thì hàm số f (x) = x, x ∈ R, thoả mãn tất cả các điều kiện của đề
bài.
Tóm lại, các kết quả đã thu được ở trên cho thấy: Có một và chỉ một
hàm số f : R → R thoả mãn hệ thức của đề bài khi và chỉ khi α = 2.
iii) Vậy, tất cả các số thực α thoả mãn yêu cầu của đề bài là α = 2.

Bài 3. 1) Gọi H là giao điểm của CM và P Q. Ta cần chứng minh P H =


HQ. Xét hai tam giác BCK và MBK có MBK \ = CKB, \ CBK \ = KMB \
vì cùng chắn cung BC của dường tròn tâm O1 . Do đó 4BCK v 4MBK,
suy ra
BC CK
= (1)
MB BK
Tương tự ta có 4ACK v MAK, suy ra
AC CK
= (2)
MA AK
Mà AK = BK nên từ (1) và (2) ta được
BC AC
= (3)
MB MA
Bây giờ ta chứng minh hai tam giác MP B và CQB đồng dạng. Thật vậy,
\
BM P =\ [ MP
BCQ, vì cùng bù với ACB, \ \ vì cùng chắn cung AB
B = CQB
của đường tròn tâm O2 . Do đó, 4MP B v 4CQB. Suy ra,
BC CQ
= (4)
BM MP
Từ (3) và (4) ta được

AC CQ CQ MA
= hay . =1 (5)
MA MP CA MP

Áp dụng định lý Menelauss cho tam giác AP Q với cát tuyết MCH, ta được
HP CQ MA
. . =1 (6)
HQ CA MP

Từ (5) và (6) suy ra P H = HQ.


\ = P[
2) Ta có CBM AQ = \ \ = BAM
QBP , BCM \ = \ BQP . Do đó
4BCM v 4BQP . Vì thế, tồn tại phép đồng dạng f tâm B biến 4BCM
thành 4BQP .
2.9. Đáp án chọn đội tuyển năm học 2003 - 2004 85

Vì các điểm B, C, M nằm trên (O1 ) và các điểm B, Q, P nằm trên (O2 )
nên f biến (O1 ) thành (O2 ). Gọi N là giao điểm thứ hai của AK và đường
tròn (O2 ), ta có
[ = QAN
CBA \ = QBN \
Mà f biến C thành Q nên từ đó suy ra f biến A thành N . Vì thế, phép
đồng dạng f biến giao điểm K của các tiếp tuyến tại A và B của đường
tròn (O1 ) thành giao điểm L của các tiếp tuyến tại N và B của đường tròn
(O2 ). Vì A, K là các điểm cố định nên N cũng là điểm cố định. Do đó L là
điểm cố định.
Vì f biến các điểm M, C thành các điểm P, Q và K nằm trên MC nên
L nằm trên P Q. Vì thế, đường thẳng P Q luôn đi qua một điểm cố định khi
điểm M di động trên đường tròn (O1 ).

Bài 4. 1) Bằng cách tính toán trực tiếp, ta có x3 = 1201 và x4 = 2003.


Viết lại hệ thức xác định dãy (xn ) dưới dạng
p
xn+2 − xn+1 − xn = 2 xn+1 xn − 2 ∀n ≥ 1.

Từ đó suy ra

x2n+2 + x2n+1 + x2n − 2xn+2 xn+1 − 2xn+2 xn + 2xn+1 xn = 4(xn+1 xn − 2)

Do đó

x2n+2 + x2n+1 − 2xn+2 xn+1 − 2xn+2 xn − 2xn+1 xn + 8 = 0 ∀n ≥ 1 (1)


2 2
xn+3 + xn+2 − 2xn+3 xn+2 − 2xn+3 xn+1 − 2xn+2 xn+1 + 8 = 0 ∀n ≥ 0 (2)

Lấy (2) trừ (1), ta được

(xn+3 − xn )(xn+3 + xn − 2xn+2 − 2xn+1 ) = 0 ∀n ≥ 1 (3)

Dễ thấy dãy (xn ) là một dãy tăng. Vì thế, từ (3) ta được

xn+3 + xn − 2xn+2 − 2xn+1 = 0 ∀n ≥ 1.

hay
xn+3 = 2xn+2 + 2xn+1 − xn , ∀n ≥ 1 (4)
Mà x1, x2, x3 là các số nguyên dương nên từ (4) suy ra xn là số nguyên
dương với mọi n ≥ 1.
2) Với mỗi số nguyên dương n, ký hiệu rn là số dư trong phép chi xn
cho 104 . Khi đó, từ (4) ta có

rn+3 ≡ 2rn+2 − rn (mod 104 ), ∀n ≥ 1 (5)


86 Chương 2. Đáp án tuyển sinh

hay
rn ≡ 2rn+2 + 2rn+1 − rn+3 (mod 104 ), ∀n ≥ 1 (6)
Xét dãy các bộ ba số

(r1 , r2 , r3), (r2 , r3, r4 ), ..., (rn, rn+1 , rn+2 ), (rn+1 , rn+2 , rn+3 ), ...

Dãy trên có vô số bộ số. Tuy nhiên, do rn ∈ {0, 1, 2, ..., 104 − 1} ∀n ≥ 1


nên chỉ có hữu hạn bộ đôi một khác nhau (tối đa là 1012 bộ). Vì thế, theo
nguyên lý Dirichlet, tồn tại hai bộ trùng nhau. Nghĩa là, tồn tại các số
nguyên dương m, k sao sao cho

(rm , rm+1 , rm+2 ) = (rm+k , rm+k+1 , rm+k+2 ).

Từ đó, nhờ (6) dễ chứng minh được

(r1 , r2 , r3) = (rk+1 , rk+2 , rk+3 ) (7)

Từ (7), nhờ (5), bằng phương pháp quy nạp theo n dễ dàng chứng minh
được dãy (rn ) là dãy tuần hoàn, kể từ số hạng thứ nhất. Mà r4 = 2003 nên
suy ra có vô số số nguyên dương n sao cho rn = 2003. Nói cách khác, tồn
tại vô số số nguyên dương n sao cho biểu diễn thập phân của xn có bốn chữ
số tận cùng là 2003.
3) Với mỗi số nguyên dương, ký hiệu sn là số dư trong phép chia xn cho
4. Khi đó, từ (4) ta được

sn+3 ≡ 2sn+2 + 2sn+1 − sn (mod 4) ∀n ≥ 1 (8)

Bằng cách tính toán trực tiếp, ta có s1 = 3, s2 = 2, s3 = 1, s4 = 3, s5 =


2, s6 = 1. Từ đó suy ra s1 = s4 , s2 = s5 và s3 = s6. Từ đây, nhờ (8), bằng
phương pháp quy nạp theo n, dễ dàng chứng minh được dãy sn ) là dãy tuần
hoàn theo chu kỳ 3. Vì thế sn 6= 0 với mọi n ∈ N∗ . Suy ra, với mọi n ∈ N∗ ,
xn − 2004 không chia hết cho 4, và do đó không chia hết cho 104 . Nói cách
khác, không tồn tại số nguyên dương n sao cho xn có bốn chữ số tận cùng
là 2004.
Bài 5. Đặt AA1 = a, BB1 = b, CC1 = c, DD1 = d, EE1 = e, F F1 =
f, A1B1 = x, B1C1 = y, C1D1 = z, D1 E1 = t, E1F1 = u, F1A1 = v.
Gọi B2 là điểm đối xứng với B qua A1B1 , C2 là điểm đối xứng với C
qua B1 C1. Khi đó, do A\ ◦ \ ◦
1 B1 C1 = 120 nên B2 B1 C2 = 60 . Suy ra tam giác
B2 B1C2 là tam giác đều. Vì thế B2 C2 = b. Do đó

a + b + c = A1B2 + B2 C2 + C2C1 ≥ A1 C1 (1)


2.9. Đáp án chọn đội tuyển năm học 2003 - 2004 87

Mặt khác, theo định lý hàm số cosin ta có



p 3
A1 C1 = x2 + y 2 + xyy ≥ (x + y) (2)
2
Từ (1) và(2) suy ra

3
a+b+c ≥ (x + y) (3)
2
Vì các cạnh của lục giác BACDEF có vai trò như nhau nên từ (3) suy
ra ta có các bất đẳng thức
√ √ √
3 3 3
b+c+d≥ (y + z); c + d + e ≥ (z + t); d + e + f ≥ (t + u);
2 √ 2 √ 2
3 3
e+f +a≥ (u + v); f + a + b ≥ (u + x).
2 2
Cộng vế theo vế bất đẳng thức (3) và năm bất đẳng thức trên, ta được

3
a+b+c+d+e+f ≥ (x + y + z + t + u + v)
3

hay p ≥ 2 33 p1 .
Dấu đẳng thức xảy ra khi và chỉ khi lục giác ABCDEF là lục giác đều.

Bài 6. Đặt s = |S| và f (n) = |Sn | ∀n ∈ N. Với mỗi n ∈ N, kí hiệu


Rn = {a+x | x ∈ Sn−1 }. Rõ ràng, Rn là một tập con của Sn và |Rn | = |Sn−1 |
nên
f (n) = |Rn | + |Sn \Rn | = f (n − 1) + |Sn \Rn |, ∀n ∈ N.
Đặt m = s(a − 1). Dễ thấy tổng của n > m số thuộc S phải chứa ít nhất a
số hạng trùng nhau. Do đó, với lưu ý rằng

d
|+d+
{z· · · + d} = a
| +a+
{z· · · + a} ∀ d ∈ S.
a số hạng d số hạng

suy ra tổng của n > m số thuộc S phải thuộc Rn . Vì vậy, Sn \Rn là một tập
con của Sm . Do đó
Sn \Rn = Sm \Rn ∀ n > m.
Đặt k = (m − 1)a. Rõ ràng, với n > k thì mỗi số thuộc Rn \Rn−1 đều lớn
hơn hoặc bằng

a + n − 1 ≥ a + k = a + (m − 1)a = ma.
88 Chương 2. Đáp án tuyển sinh

Do mọi số của Sm \Rk phải nhỏ hơn ma nên Sm \Rn = Sm \Rn−1 với mọi
n > k. Từ đó suy ra Sn \Rn = Sm \Rk , và do đó

|Sn \Rn | = |Sm \Rk | = |Sk \Rk | ∀n > k.

Vì thế, đặt b = f (k) − k|Sk \Rk |, ta có

f (n) = f (n − 1) + |Sk \Rk | = (n − k)|Sk \Rk | + f (k) = n|Sk \Rk | + b ∀ n > k.

Tiếp theo, ta chứng minh |Sk \Rk | = a. Thật vậy, xét các trường hợp sau:
Trường hợp 1. s = 2. Khi đó k = 2a2 − 3a ≥ a. Giả sử S = {c, a}. Do
(c, a) = 1 theo giả thiết của đề bài, nên Sk \Rk = {0, c, 2c, ..., (a − 1)c}. Vì
vậy |Sk \Rk | = a.
Trường hợp 2. s > 2. Gọi c là số nhỏ nhất trong S, ta có (c, a) = 1.
Vì thế, theo chứng minh ở trường hợp 1, tồn tại số nguyên tố b0 sao cho
f (n) ≥ an + b0 với mọi n > 2a2 . Do đó, nếu |Sk \Rk | ≤ a − 1 thì

an + b0 ≤ f (n) = n|Sk \Rk | ≤ (a − 1)n + b.

Suy ra n ≤ b − b0 , là điều vô lí vì n có thể lớn tuỳ ý. Như vậy, phải có


|Sk \Rk | ≥ a. Vậy, tóm lại, tồn tại số nguyên dương k và số nguyên b sao
cho
|Sn | = an + b với mọi n > k.

Chapter II:2005 to 2010


ĐỀ THI VÀ LỜI GIẢI
ĐỀ CHỌN ĐỘI TUYỂN QUỐC GIA
DỰ THI OLYMPIC TOÁN QUỐC TẾ
CỦA VIỆT NAM
TỪ NĂM 2005 ĐẾN NĂM 2010

1
PHẦN I
*****

ĐỀ BÀI

2
ĐỀ THI CHỌN ĐỘI TUYỂN QUỐC GIA
DỰ THI IMO 2005

*Ngày thi thứ nhất.


Bài 1. Cho tam giác ABC có (I) và (O) lần lượt là các đường tròn nội tiếp, ngoại tiếp.
Gọi D, E, F lần lượt là tiếp điểm của (I) trên các cạnh BC, CA, AB. Gọi ω A , ωB , ωC lần lượt là
các đường tròn tiếp xúc với hai đường tròn (I) và (O) lần lượt tại các điểm D, K (với đường
tròn ω A ); tại E, M (với đường tròn ωB ) và tại F, N (với đường tròn ωC ). Chứng minh rằng:
1. Các đường thẳng DK , EM , FN đồng quy tại P.
2. Trực tâm của tam giác DEF nằm trên đoạn OP.

Bài 2. Trên một vòng tròn có n chiếc ghế được đánh số từ 1 đến n. Người ta chọn ra
k chiếc ghế. Hai chiếc ghế được chọn gọi là kề nhau nếu đó là hai chiếc ghế được chọn liên
tiếp. Hãy tính số cách chọn ra k chiếc ghế sao cho giữa hai chiếc ghế kề nhau, không có ít
hơn 3 chiếc ghế khác.
Bài 3. Tìm tất cả các hàm số f : ℤ → ℤ thỏa mãn điều kiện:
f ( x 3 + y 3 + z 3 ) = ( f ( x))3 + ( f ( y ))3 + ( f ( z ))3

*Ngày thi thứ hai.


Bài 4. Chứng minh rằng:
a3 b3 c3 3
+ + ≥
(a + b) (b + c) (c + a) 8
3 3 3

trong đó a, b, c là các số thực dương.


Bài 5. Cho số nguyên tố p ( p > 3) . Tính:
p −1
2  2k 2  k2 
a) S = ∑   − 2  p  nếu p ≡ 1 (mod 4) .
k =1  p   
p −1
2 k2 
b) S = ∑   nếu p ≡ 1 (mod 8) .
k =1  p 

Bài 6. Một số nguyên dương được gọi là “số kim cương 2005” nếu trong biểu diễn
thập phân của nó có 2005 số 9 đứng cạnh nhau liên tiếp. Dãy ( an ) , n = 1, 2,3,... là dãy tăng
ngặt các số nguyên dương thỏa mãn an < nC (C là hằng số thực dương nào đó).
Chứng minh rằng dãy số ( an ) , n = 1, 2,3,... chứa vô hạn “số kim cương 2005”.

3
ĐỀ THI CHỌN ĐỘI TUYỂN QUỐC GIA
DỰ THI IMO 2006

* Ngày thi thứ nhất.


Bài 1. Cho tam giác ABC có H là trực tâm. Đường phân giác ngoài của góc BHC cắt các
cạnh AB, AC lần lượt tại D và E. Đường phân giác trong của góc BAC cắt đường tròn ngoại
tiếp tam giác ADE tại điểm K. Chứng minh rằng đường thẳng HK đi qua trung điểm của BC.
Bài 2. Hãy tìm tất cả các cặp số tự nhiên ( n ; k ) với n là số nguyên không âm và k là
số nguyên lớn hơn 1 sao cho số : A = 17 2006 n + 4.17 2 n + 7.195 n có thể phân tích được thành
tích của k số nguyên dương liên tiếp.
Bài 3. Trong không gian cho 2006 điểm mà trong đó không có 4 điểm nào đồng
phẳng. Người ta nối tất cả các điểm đó lại bởi các đoạn thẳng. Số tự nhiên m gọi là số tốt nếu
ta có thể gán cho mỗi đoạn thẳng trong các đoạn thẳng đã nối bởi một số tự nhiên không
vượt quá m sao cho mỗi tam giác tạo bởi ba điểm bất kì trong số các điểm đó đều có hai
cạnh được gán bởi hai số bằng nhau và cạnh còn lại gán bởi số lớn hơn hai số đó.
Tìm số tốt có giá trị nhỏ nhất.
* Ngày thi thứ hai .
Bài 4. Chứng minh rằng với mọi số thực x, y, z ∈ [1; 2] , ta luôn có bất đẳng thức sau :
1 1 1 x y z
( x + y + z )( + + ) ≥ 6( + + )
x y z y+z z+x x+ y .
Hỏi đẳng thức xảy ra khi và chỉ khi nào ?
Bài 5. Cho tam giác ABC là tam giác nhọn, không cân, nội tiếp trong đường tròn tâm
O bán kính R. Một đường thẳng d thay đổi sao cho d luôn vuông góc với OA và luôn cắt các
tia AB, AC. Gọi M, N lần lượt là giao điểm của đường thẳng d và các tia AB, AC. Giả sử các
đường thẳng BN và CN cắt nhau tại K; giả sử đường thẳng AK cắt đường thẳng BC.
1. Gọi P là giao của đường thẳng AK và đường thẳng BC. Chứng minh rằng đường tròn
ngoại tiếp của tam giác MNP luôn đi qua một điểm cố định khi d thay đổi.
2. Gọi H là trực tâm của tam giác AMN. Đặt BC = a và l là khoảng cách từ điểm A đến HK.
Chứng minh rằng đường thẳng HK luôn đi qua trực tâm của tam giác ABC.
Từ đó suy ra: l ≤ 4 R 2 − a 2 . Đẳng thức xảy ra khi và chỉ khi nào?
Bài 6. Cho dãy số thực (an ) được xác định bởi:
1 1
a0 = 1, an +1 = ( an + ) với mọi n = 1, 2, 3, …
2 3an
3
Chứng minh rằng với mọi số nguyên n, số An = là một số chính phương và nó có ít
3a − 1
2
n

nhất n ước nguyên tố phân biệt.


4
ĐỀ THI CHỌN ĐỘI TUYỂN QUỐC GIA
DỰ THI IMO 2007

*Ngày thi thứ nhất.

Bài 1. Cho hai tập hợp A,B là tập hợp các số nguyên dương thỏa mãn A = B = n
(với n là số nguyên dương) và có tổng các phần tử bằng nhau. Xét bảng ô vuông n × n .
Chứng minh rằng ta có thể điền vào mỗi ô vuông của bảng một số nguyên không âm
thỏa mãn đồng thời các điều kiện:
i/ Tổng của các phần tử ở mỗi hàng là các phần tử của tập A.
ii/ Tổng của các phần tử ở mỗi cột là các phần tử của tập B.
iii/ Có ít nhất (n − 1)2 + k số 0 trong bảng với k là số các phần tử chung của A và B.
Bài 2. Cho tam giác nhọn ABC với đường tròn nội tiếp I. Gọi (ka ) là đường tròn có
tâm nằm trên đường cao của góc A, đi qua điểm A và tiếp xúc trong với đường tròn (I) tại A1 .
Các điểm B1 , C1 xác định tương tự .
1/ Chứng minh AA1 , BB1 , CC1 đồng qui tại P.
2/ Gọi ( J a ), ( J b ), ( J c ) lần lượt là các đường tròn đối xứng với đường tròn bàng tiếp
các góc A, B, C của tam giác ABC qua trung điểm BC, CA, AB.
Chứng minh P là tâm đẳng phương của 3 đường tròn nói trên.
Bài 3. Cho tam giác ABC. Tìm giá trị nhỏ nhất của biểu thức sau:
A B B C C A
cos 2 cos 2 cos 2 cos 2 cos 2 cos 2
S= 2 2 + 2 2 + 2 2.
2 C 2 A 2 B
cos cos cos
2 2 2
*Ngày thi thứ hai.

Bài 4. Tìm tất cả các hàm số liên tục f : ℝ → ℝ thỏa mãn:


x 1
+ ) với mọi x ∈ ℝ .
f ( x) = f ( x 2 +
3 9
Bài 5. Cho A là tập con chứa 2007 phần tử của tập: {1, 2, 3,..., 4013, 4014} thỏa mãn
với mọi a, b ∈ A thì a không chia hết cho b. Gọi mA là phần tử nhỏ nhất của A.
Tìm giá trị nhỏ nhất của mA với A thỏa mãn các điều kiện trên.
Bài 6. Cho đa giác 9 cạnh đều (H). Xét ba tam giác với các đỉnh là các đỉnh của đa giác
(H) đã cho sao cho không có hai tam giác nào có chung đỉnh.
Chứng minh rằng có thể chọn được từ mỗi tam giác 1 cạnh sao cho 3 cạnh này bằng nhau.

5
ĐỀ THI CHỌN ĐỘI TUYỂN QUỐC GIA
DỰ THI IMO 2008
*Ngày thi thứ nhất.
Bài 1. Trong mặt phẳng cho góc xOy. Gọi M, N lần lượt là hai điểm lần lượt nằm trên
các tia Ox, Oy. Gọi d là đường phân giác góc ngoài của góc xOy và I là giao điểm của trung
trực MN với đường thẳng d. Gọi P, Q là hai điểm phân biệt nằm trên đường thẳng d sao
cho IM = IN = IP = IQ , giả sử K là giao điểm của MQ và NP.
1. Chứng minh rằng K nằm trên một đường thẳng cố định.
2. Gọi d1 là đường thẳng vuông góc với IM tại M và d2 là đường thẳng vuông góc với IN
tại N. Giả sử các đường thẳng d1, d2 cắt đường thẳng d tại E, F. Chứng minh rằng các đường
thẳng EN, FM và OK đồng quy.
Bài 2. Hãy xác định tất cả các số nguyên dương m sao cho tồn tại các đa thức với hệ
số thực P ( x ), Q ( x ), R ( x, y ) thỏa mãn điều kiện:
Với mọi số thực a, b mà a m − b 2 = 0 , ta luôn có P ( R ( a, b)) = a và Q ( R ( a, b)) = b .
Bài 3. Cho số nguyên n > 3. Kí hiệu T là tập hợp gồm n số nguyên dương đầu tiên.
Một tập con S của T được gọi là tập khuyết trong T nếu S có tính chất: Tồn tại số nguyên
n
dương c không vượt quá sao cho với s1 , s2 là hai số bất kì thuộc S ta luôn có s1 − s2 ≠ c .
2
Hỏi tập khuyết trong T có thể có tối đa bao nhiêu phần tử ?
*Ngày thi thứ hai.
Bài 4. Cho m, n là các số nguyên dương. Chứng minh rằng (2m + 3) n + 1 chia hết cho
6m khi và chỉ khi 3n + 1 chia hết cho 4m.
Bài 5. Cho tam giác ABC nhọn, không cân có O là tâm đường tròn ngoại tiếp.
Gọi AD, BE, CF là các đường phân giác trong của tam giác. Trên các đường thẳng AD, BE, CF
AL BM CN
lần lượt lấy các điểm L, M, N sao cho = = = k (k là một hằng số dương).
AD BE CF
Gọi (O1), (O2), (O3) lần lượt là các đường tròn đi qua L, tiếp xúc với OA tại A ; đi qua M, tiếp
xúc với OB tại B và đi qua N, tiếp xúc với OC tại C.
1
1. Chứng minh rằng với k = , ba đường tròn (O1), (O2), (O3) có đúng hai điểm chung
2
và đường thẳng nối hai điểm chung đó đi qua trọng tâm tam giác ABC.
2. Tìm tất cả các giá trị k sao cho 3 đường tròn (O1), (O2), (O3) có đúng hai điểm chung.
Bài 6. Kí hiệu M là tập hợp gồm 2008 số nguyên dương đầu tiên. Tô tất cả các số
thuộc M bởi ba màu xanh, vàng, đỏ sao cho mỗi số được tô bởi một màu và mỗi màu đều
được dùng để tô ít nhất một số. Xét các tập hợp sau:
S1 = {( x, y , z ) ∈ M 3 , trong đó x, y, z có cùng màu và ( x + y + z ) ≡ 0 (mod 2008)} ;
S 2 = {( x, y , z ) ∈ M 3 , trong đó x, y, z đôi một khác màu và ( x + y + z ) ≡ 0 (mod 2008)} .
Chứng minh rằng 2 S1 > S2 . (Kí hiệu M 3 là tích Đề - các M × M × M ) .
6
ĐỀ THI CHỌN ĐỘI TUYỂN QUỐC GIA
DỰ THI IMO 2009

*Ngày thi thứ nhất.


Bài 1. Cho tam giác nhọn ABC nội tiếp đường tròn (O). Gọi A1 , B1 , C1 và A2 , B2 , C2
lần lượt là các chân đường cao của tam giác ABC hạ từ các đỉnh A, B, C và các điểm đối xứng
với A1 , B1 , C1 qua trung điểm của các cạnh BC , CA, AB . Gọi A3 , B3 , C3 lần lượt là các giao
điểm của đường tròn ngoại tiếp các tam giác AB2C2 , BC2 A2 , CA2 B2 với (O).
Chứng minh rằng: A1 A3 , B1 B3 , C1C3 đồng quy.
Bài 2. Cho đa thức P ( x) = rx 3 + qx 2 + px + 1 trong đó p , q , r là các số thực và r > 0 .
Xét dãy số ( an ) xác định như sau:

 a1 = 1, a2 = − p, a3 = p − q
2


 an +3 = − p.an + 2 − q.an +1 − r.an , n ≥ 0
Chứng minh rằng: nếu đa thức P ( x ) có một nghiệm thực duy nhất và không có
nghiệm bội thì dãy số ( an ) có vô số số âm.
Bài 3. Cho các số nguyên dương a, b sao cho a, b và ab đều không phải là số chính
phương. Chứng minh rằng trong hai phương trình sau:
ax 2 − by 2 = 1
ax 2 − by 2 = −1
có ít nhất một phương trình không có nghiệm nguyên dương.
*Ngày thi thứ hai.
Bài 4. Tìm tất cả các số thực r sao cho bất đẳng thức sau đúng với mọi a, b, c dương:
3
 a  b  c   1
r + r +  r +  ≥ r + 
 b + c  c + a  a+b  2
Bài 5. Cho đường tròn (O) có đường kính AB và M là một điểm bất kì nằm trong (O),
M không nằm trên AB. Gọi N là giao điểm của phân giác trong góc M của tam giác AMB với
đường tròn (O). Đường phân giác ngoài góc AMB cắt các đường thẳng NA, NB lần lượt tại P,
Q. Đường thẳng MA cắt đường tròn đường kính NQ tại R, đường thẳng MB cắt đường tròn
đường kính NP tại S và R, S khác M.
Chứng minh rằng: đường trung tuyến ứng với đỉnh N của tam giác NRS luôn đi qua
một điểm cố định khi M di động phía trong đường tròn.
Bài 6. Một hội nghị toán học có tất cả 6n + 4 nhà toán học phải họp với nhau đúng
2n + 1 lần ( n ≥ 1) . Mỗi lần họp, họ ngồi quanh một cái bàn 4 chỗ và n cái bàn 6 chỗ, các vị
trí ngồi chia đều khắp mỗi bàn. Biết rằng hai nhà toán học đã ngồi cạnh hoặc đối diện nhau
ở một cuộc họp này thì sẽ không được ngồi cạnh hoặc đối diện nhau ở một cuộc họp khác.
a/ Chứng minh rằng Ban tổ chức có thể xếp được chỗ ngồi nếu n = 1 .
b/ Hỏi rằng Ban tổ chức có thể sắp xếp được chỗ ngồi được hay không với mọi n > 1 ?

7
ĐỀ THI CHỌN ĐỘI TUYỂN QUỐC GIA
DỰ THI IMO 2010

* Ngày thi thứ nhất.

Bài 1. Cho tam giác ABC không vuông tại A có đường trung tuyến AM. Gọi D là một
điểm di động trên đường thẳng AM. Gọi (O1 ), (O2 ) là các đường tròn đi qua D, tiếp xúc với
BC lần lượt tại B và C. Gọi P, Q lần lượt là giao điểm của đường thẳng AB với đường tròn
(O1 ) , đường thẳng AC với đường tròn (O2 ) . Chứng minh rằng:
1. Tiếp tuyến tại P của (O1 ) và tiếp tuyến tại Q của (O2 ) phải cắt nhau tại một điểm.
Gọi giao điểm đó là S.
2. Điểm S luôn di chuyển trên một đường thẳng cố định khi D di động trên AM.
Bài 2. Với mỗi số n nguyên dương, xét tập hợp sau :
Tn = {11( k + h) + 10( n k + n h ) |1 ≤ k , h ≤ 10} .
Tìm tất cả giá trị của n sao cho không tồn tại a, b ∈ Tn ; a ≠ b sao cho ( a − b ) chia hết cho 110.
Bài 3. Gọi một hình chữ nhật có kích thước 1 × 2 là hình chữ nhật đơn và một hình
chữ nhật có kích thước 2 × 3 , bỏ đi 2 ô ở góc chéo nhau (tức là có 4 ô vuông nhỏ) là hình chữ
nhật kép. Người ta ghép khít các hình chữ nhật đơn và hình chữ nhật kép này lại với nhau
được một bảng hình chữ nhật có kích thước là 2008 × 2010 .
Tìm số bé nhất các hình chữ nhật đơn có thể dùng để ghép.

* Ngày thi thứ hai.


1 1 1
Bài 4. Cho a, b, c là các số thực dương thỏa mãn điều kiện: 16(a + b + c ) ≥ + + .
a b c
Chứng minh rằng:
1 1 1 8
+ + ≤ .
( a + b + 2( a + c )) (b + c + 2(b + a )) (c + a + 2(c + b))
3 3 3
9
Hỏi đẳng thức xảy ra khi nào?
Bài 5: Trong một hội nghị có n nước tham gia, mỗi nước có k đại diện ( n > k > 1) .
Người ta chia n.k người này thành n nhóm, mỗi nhóm có k người sao cho không có hai
người nào cùng nhóm đến từ cùng một nước.
Chứng minh rằng có thể chọn ra một nhóm gồm n người sao cho họ thuộc các nhóm khác
nhau và đến từ các nước khác nhau.
Bài 6: Gọi Sn là tổng bình phương các hệ số trong khai triển của nhị thức (1 + x)n ,
trong đó n là số nguyên dương; x là số thực bất kì.
Chứng minh rằng: S2 n + 1 không chia hết cho 3 với mọi n.
8
PHẦN II
*****

LỜI GIẢI

9
LỜI GIẢI ĐỀ THI

CHỌN ĐỘI TUYỂN QUỐC GIA DỰ THI IMO 2005

Bài 1. Cho tam giác ABC có (I) và (O) lần lượt là các đường tròn nội tiếp, ngoại tiếp.
Gọi D, E, F lần lượt là tiếp điểm của (I) trên các cạnh BC, CA, AB. Gọi ω A , ωB , ωC lần lượt
là các đường tròn tiếp xúc với hai đường tròn (I) và (O) lần lượt tại các điểm D, K (với đường
tròn ω A ); tại E, M (với đường tròn ωB ) và tại F, N (với đường tròn ωC ). Chứng minh rằng:
1. Các đường thẳng DK, EM, FN đồng quy tại P.
2. Trực tâm của tam giác DEF nằm trên đoạn OP.
1. Trước hết, ta sẽ chứng minh bổ đề sau:
B
Cho ba đường tròn (O1), (O2), (O3) có bán
kính đôi một khác nhau; A, B, C lần lượt là tâm vị
tự của các cặp đường tròn (O1) và (O2), (O2) và
(O3), (O3) và (O1).
Chứng minh rằng nếu trong các tâm vị tự đó, có
ba tâm vị tự ngoài hoặc hai tâm vị tự trong, một
O2 tâm vị tự ngoài thì A, B, C thẳng hàng.
O1 A *Chứng minh:
Gọi R1 , R2 , R3 lần lượt là bán kính của các đường
O3
tròn (O1 ), (O2 ), (O3 ) , các giá trị R1 , R2 , R3 này đôi
một khác nhau.
AO1 R
Theo tính chất về tâm vị tự, ta có: = (−1) a 1 .
C AO2 R2
BO2 R CO3 R
Tương tự: = (−1)b 2 , = (−1)c 3 , trong
BO3 R3 CO1 R1
đó, mỗi số a, b, c nhận giá trị là 0 (khi nó là tâm vị tự ngoài) hoặc 1 (khi nó là tâm vị tự trong).
Theo giả thiết trong a, b, c có ba giá trị là 0 hoặc hai giá trị 0, một giá trị 1. Từ đó:
AO1 BO2 CO3
. . = 1 , theo định lí Menelaus đảo cho tam giác O1O2O3 , ta có: A, B, C thẳng hàng.
AO2 BO3 CO1
Bổ đề được chứng minh.
*Trở lại bài toán:
Gọi P’ là tâm vị tự trong của hai đường tròn (O) và (I). Dễ thấy: D là điểm tiếp xúc ngoài
của ω A và (I) nên cũng chính là tâm vị tự trong của hai đường tròn này; K là điểm tiếp xúc trong
của hai đường tròn ω A và (O) nên là tâm vị tự ngoài của hai đường tròn này. Theo bổ đề trên thì
P ', D, K thẳng hàng hay đường thẳng DK đi qua P’. Tương tự, các đường thẳng EM và FN cũng
đi qua P’; tức là ba đường thẳng DK, EM, FN đồng quy và điểm P’ chính là điểm P của đề bài.
10
2. Ta chứng minh bổ đề sau:
Cho tam giác ABC có (O), (I) lần lượt là tâm đường tròn ngoại tiếp, nội tiếp của tam giác
ABC. Đường tròn (I) tiếp xúc với các cạnh BC, CA, AB lần lượt tại D, E, F. Chứng minh rằng
trực tâm H của tam giác DEF nằm trên đường thẳng OI.

* Chứng minh:
A Gọi M, N, P lần lượt là trung điểm của các
đoạn EF, FD, DE. Dễ thấy AI là trung trực của
đoạn EF nên M thuộc đường thẳng AI hay A, M, I
thẳng hàng. Tương tự: B, N, I và C, P, I cũng
M E
thẳng hàng. Xét phép nghịch đảo Φ tâm I, phương
F tích r 2 với r là bán kính đường tròn (I).
H O Dễ thấy: tam giác IEA vuông tại E có EM là
I P
đường cao nên: IM .IA = IE 2 = r 2 , suy ra:
N Φ : M → A . Tương tự: Φ : N → B, P → C .
B D C Do đó: Φ : ∆MNP → ∆ABC . Gọi E là tâm
đường tròn ngoại tiếp của tam giác MNP thì
Φ : E → O , suy ra: E, I, O thẳng hàng.

Hơn nữa, I là tâm đường tròn ngoại tiếp tam giác DEF, E là tâm đường tròn ngoại tiếp tam
giác MNP cũng chính là tâm đường tròn Euler của tam giác DEF này nên E, I, H thẳng hàng.
Từ đó suy ra H, I, O thẳng hàng. Bổ đề được chứng minh.

A M
* Trở lại bài toán:
Gọi H là trực tâm tam giác DEF thì theo bổ đề trên:
H, I, O thẳng hàng.
Theo câu 1/, điểm P nằm trên đoạn OI. E
Suy ra: 4 điểm H, I, P, O thẳng hàng. N F
Từ đó suy ra trực tâm H của tam giác DEF nằm trên I P
H O
đường thẳng OI.
Ta có đpcm.
B D C

11
Bài 2. Trên một vòng tròn có n chiếc ghế được đánh số từ 1 đến n. Người ta chọn ra k
chiếc ghế. Hai chiếc ghế được chọn gọi là kề nhau nếu đó là hai chiếc ghế được chọn liên tiếp.
Hãy tính số cách chọn ra k chiếc ghế sao cho giữa hai chiếc ghế kề nhau, không có ít hơn 3
chiếc ghế khác.

*Trước hết, ta chứng minh bổ đề sau:


Cho n điểm phân biệt nằm trên đường thẳng được tô một trong hai màu, xanh hoặc đỏ
thỏa mãn các điều kiện sau:
- Có đúng k điểm được tô màu xanh.
- Giữa hai điểm màu xanh liên tiếp có ít nhất p điểm được tô màu đỏ (tính từ trái sang).
- Ở bên phải điểm màu xanh cuối cùng có ít nhất p điểm được tô màu đỏ.
Khi đó, số cách tô màu là: Cnk−kp .
*Chứng minh: Đánh số các điểm đã cho là 1, 2,3,..., n . Đặt tương ứng mỗi cách tô màu
với một bộ k các số nguyên dương (i1 , i2 ,..., ik ) trong đó i1 , i2 ,..., ik là các điểm được tô màu xanh.
Dễ thấy tương ứng nói trên chính là một song ánh từ tập các cách tô màu đến tập hợp T sau:
T = {(i1 , i2 ,..., ik ) | is ∈{1, 2,..., n − p}, ∀s = 1, k ; is +1 − is > p, ∀i = 1, k − 1} .
Xét ánh xạ sau:
T → T ' = {( j1 , j2 ,..., jk ) | jt ∈{1, 2,..., n − kp}, jt +1 > jt , ∀t = 1, k} .
Ta sẽ chứng minh ánh xạ này là một song ánh.
Thật vậy:
*Xét một bộ ( j1 , j2 ,..., jk ) ∈ T ' . Khi đó, ta xét tiếp bộ: ( j1 , j2 + p, j3 + 2 p,..., jk + ( k − 1) p ) .
Do 1 ≤ jt ≤ n − kp nên phần tử lớn nhất của bộ này là jk + ( k − 1) p có giá trị không vượt quá
n − kp + ( k − 1) p = n − p . Từ đó suy ra: jt + (t − 1) p ∈ {1, 2,..., n − kp}, ∀t ≥ 1 .
Hơn nữa: [ jt +1 + tp ] − [ jt + (t − 1) p ] = ( jt +1 − jt ) + p > p .
Từ đó suy ra bộ ( j1 , j2 + p, j3 + 2 p,..., jk + ( k − 1) p ) ∈ T .
Do đó, tương ứng này là một toàn ánh.
*Xét bộ (i1 , i2 ,..., ik ) ∈ T . Khi đó, hoàn toàn tương tự trên, ta cũng chứng minh được bộ
(i1 , i2 − p, i2 − 2 p,..., ik − ( k − 1) p ) ∈ T ' .
Ta sẽ chứng minh rằng nếu có hai bộ khác nhau (i1 , i2 ,..., ik ), (i '1 , i '2 ,..., i 'k ) ∈ T thì các bộ tương
ứng thuộc T= T ' của chúng cũng phải khác nhau. Nhưng điều này là hiển nhiên do hai bộ này là
khác nhau nên tồn tại chỉ số s sao cho is ≠ i 's , khi đó is − ( s − 1) p ≠ i 's − ( s − 1) p .
Suy ra, tương ứng này cũng là một đơn ánh.
Vậy tương ứng T → T ' là một song ánh.
Nhận xét trên được chứng minh.
Do đó: | T |=| T ' |= Cnk−kp . Bổ đề được chứng minh.

12
*Trở lại bài toán:
Ta xét tổng quát giá trị 3 trong đề bài bởi giá trị p tương ứng với bổ đề trên.
Đánh số các ghế trong đề bài theo chiều kim đồng hồ là A1 , A2 ,..., An (xem như là các điểm nằm
trên một vòng tròn) ; mỗi ghế được chọn xem như được tô màu xanh và không được chọn xem
như được tô màu đỏ; gọi X là tập hợp tất cả các cách tô màu k điểm trong n điểm đã cho thỏa
mãn đề bài.
Xét phân hoạch: X = X '∪ X '' ⇒ X = X ' + X " .
trong đó X ' là cách tô màu thỏa mãn có một điểm được tô màu xanh thuộc { A1 , A2 , A3 ,..., Ap } và
X '' = X \ X ' , khi đó rõ ràng, với mọi phần tử thuộc X " thì không có điểm nào được tô màu
xanh thuộc { A1 , A2 , A3 ,..., Ap } , tức là mọi điểm trong tập này đều được tô màu đỏ. Ta cắt đường
tròn ngay tại điểm Ap , Ap +1 thì rõ ràng sẽ tạo được một đường thẳng thỏa mãn tất cả điều kiện
của bổ đề đã nêu ở trên, suy ra: X '' = Cnk−kp . Ta chỉ còn cần tính số phần tử của X ' .
Xét tập hợp X 'i trong đó mỗi phần tử của X 'i có đúng một điểm Ai được tô màu xanh,
p
i = 1, p ; khi đó rõ ràng X 'i ∩ X ' j = ∅, ∀i ≠ j và ∪X ' i = X '.
i =1

Với mỗi i = 1, p , theo bổ đề trên, ta thấy:


Cnk−−11− p −( k −1) p = Cnk−−kp
1 k −1
−1 , tức là các tập X 'i này có cùng số phần tử. Suy ra: X ' = pCn − kp −1 .

Do đó: X = X ' + X '' = Cnk− kp + pCnk−−kp


1
−1 .

Thay p = 3 , ta được số cách chọn ghế tương ứng trong đề bài là Cnk−3k + 3Cnk−−31k −1 .
Vậy số cách chọn ghế thỏa mãn tất cả các điều kiện của đề bài là: Cnk−3k + 3Cnk−−31k −1 .

13
Bài 3. Tìm tất cả các hàm số f : ℤ → ℤ thỏa mãn điều kiện:
f ( x 3 + y 3 + z 3 ) = ( f ( x))3 + ( f ( y ))3 + ( f ( z ))3

* Trước hết, ta sẽ chứng minh bổ đề sau:


Với mọi số nguyên dương lớn hơn 10, lập phương của nó đều có thể biểu diễn được dưới
dạng tổng của 5 lập phương của các số nguyên khác có giá trị tuyệt đối nhỏ hơn nó.

* Thật vậy:
Ta cần tìm mối liên hệ đó với số n > 10 trong từng trường hợp n chẵn và n lẻ.
- Với n là số lẻ, đặt n = 2k + 1 .
Ta cần tìm một đẳng thức đúng với mọi k mà trong đó 2k + 1 là biểu thức có giá trị tuyệt
đối lớn nhất, các biểu thức còn lại phải là nhị thức bậc nhất có hệ số của k lớn nhất là 2. Khi đó
để khử 8k 3 xuất hiện ở trong ( 2k + 1) , ta chọn − ( 2k − 1) ; ta thấy vẫn còn số hạng chứa k bậc
3 3

hai trong đó, ta chọn tiếp hai biểu thức khác có chứa k cùng hai hằng số bằng cách dùng tham số
như sau:
Giả sử hai biểu thức cần tìm có dạng (ak + b), ( ak − b); a, b ∈ ℤ và hai số cần tìm là
c, d ∈ ℤ , tức là:
(2k + 1)3 − (2k − 1)3  = (ak + b)3 − (ak − b)3  + c 3 + d 3 
⇔ ( 24k 2 + 2 ) = ( 6a 2bk 2 + 2b3 ) + ( c 3 + d 3 )
⇔ k 2 (24 − 6a 2b) + (2 − 2b3 − c3 − d 3 ) = 0
Ta cần chọn a, b, c, d sao cho a 2b = 4, 2b3 + c 3 + d 3 = 2 trong đó a ≤ 2 .
Dễ thấy a ≠ 2 vì nếu a = 2 thì từ a 2b = 4 ⇒ b = 1 , trùng với biểu thức cần đánh giá; do đó,
a = 1, b = 4 , suy ra: c 3 + d 3 = −126 , ta chọn được c = −5, d = −1 .
Do đó:
(2k + 1)3 = (2k − 1)3 + (k + 4)3 + (4 − k )3 + ( −5)3 + ( −1)3 (1)
Thử lại, ta thấy biểu thức này đúng với mọi k.
- Với n là số chẵn, đặt n = 2k + 2 .
Lập luận hoàn toàn tương tự, ta có được đẳng thức sau:
(2k + 2)3 = (2k − 2)3 + (k + 8)3 + (8 − k )3 + ( −10)3 + ( −2)3 (2)
Bổ đề được chứng minh.

*Trở lại bài toán:


Trong đẳng thức đã, thay x = y = z = 0 , ta được:
f (0) = 3 f 3 (0) ⇔ f (0) = 0 ∨ 3 f 2 (0) = 1 .
Do hàm này chỉ lấy giá trị trên f : ℤ → ℤ nên không thể có 3 f 2 (0) = 1 , tức là f (0) = 0 .

14
- Thay y = z = 0 , ta có: f ( x 3 ) = ( f ( x))3 + ( f (0))3 + ( f (0))3 = ( f ( x))3 .
- Lại thay y = − z , ta có:
f ( x 3 ) = ( f ( x))3 + ( f ( y ))3 + ( f (− y ))3 ⇔ ( f ( y ))3 + ( f ( − y ))3 = 0 ⇔ f ( y ) = − f ( − y ), ∀y ∈ ℤ .
Ta sẽ chứng minh rằng: ∀k ∈ ℤ : f (k ) = k . f 3 (1) bằng quy nạp. (*)
*Thật vậy:
- Với k = 1 , trong giả thiết, thay x = 1, y = z = 0 , ta có f (1) = f 3 (1) ⇒ f (−1) = − f 3 (1)
- Với k = 2 , trong giả thiết, thay x = y = 1, z = 0 , ta có f (2) = 2 f 3 (1) ⇒ f (−2) = −2 f 3 (1)
- Với k = 3 , trong giả thiết, thay x = y = z = 1 , ta có f (3) = 3 f 3 (1) ⇒ f (−3) = −3 f 3 (1)
- Thay x = 2, y = z = 0 , ta có f (8) = f 3 (2) = (2 f (1))3 = 8 f 3 (1) ⇒ f ( −8) = −8 f 3 (1) .
- Thay x = 2, y = 1, z = 0 , ta có f (9) = f 3 (2) + f 3 (1) = 9 f 3 (1) ⇒ f ( −9) = −9 f 3 (1) .
- Thay x = 2, y = z = 1 , ta có: f (10) = f 3 (2) + 2 f 3 (1) = 10 f 3 (1) ⇒ f ( −10) = −10 f 3 (1) .
- Thay x = 2, y = −1, z = 0 , ta có: f (7) = f 3 (2) − f 3 (1) = 7 f 3 (1) ⇒ f ( −7) = −7 f 3 (1) .
- Thay x = 2, y = z = −1 , ta có: f (6) = f 3 (2) − 2 f 3 (1) = 6 f 3 (1) ⇒ f ( −6) = −6 f 3 (1) .
- Trong đẳng thức (1) của bổ đề trên, ta thay k = 2 , suy ra:
53 = 33 + 63 + 23 + ( −5)3 + ( −1)3 hay f (53 + 53 + 13 ) = f (33 + 63 + 23 ) .
⇒ 2 f 3 (5) + f (13 ) = f 3 (3) + f 3 (6) + f 3 (2) ⇒ f (5) = 5 f 3 (1) ⇒ f ( −5) = −5 f 3 (1) .
- Trong đẳng thức (2) của bổ đề trên, ta thay k = 1 , suy ra:
43 = 03 + 93 + 73 + ( −10)3 + (−2)3 hay f (43 + 103 + 23 ) = f (93 + 73 + 03 ) .
⇒ f 3 (4) + f 3 (10) + f 3 (2) = f 3 (9) + f 3 (7) + f 3 (0) ⇒ f (4) = 4 f 3 (1) ⇒ f ( −4) = −4 f 3 (1) .
Như thế, ta đã chứng minh được (*) đúng với mọi k ≤ 10 .
Với k > 10 , xét k > 0 thì theo bổ đề ở trên, lập phương của k đều có thể biểu diễn được
dưới dạng tổng của 5 lập phương khác có giá trị tuyệt đối nhỏ hơn nó.
Hơn nữa, dễ thấy rằng với a, b, c, d , e, f ∈ ℤ thỏa a 3 + b3 + c3 = m3 + n3 + p 3 và ta đã có:
f (b) = bf 3 (1), f (c) = cf 3 (1), f ( m) = mf 3 (1), f ( n) = nf 3 (1), f ( p ) = pf 3 (1) thì f (a ) = af 3 (1) .
Từ đó, suy ra f (k ) = kf 3 (1), ∀k > 10 .
Với k < −10 thì f (k ) = − f (− k ) = −( − kf 3 (1)) = kf 3 (1) .
Do đó, theo nguyên lí quy nạp (*) được chứng minh.
Mặt khác, trong giả thiết đã cho, thay x = 1, y = z = 0 , ta có: f (1) = f 3 (1) ⇔ f (1) = ±1 ∨ f (1) = 0 .
- Nếu f (1) = −1 thì f ( k ) = − k , ∀k ∈ ℤ , thử lại thấy thỏa.
- Nếu f (1) = 0 thì f ( k ) = 0, ∀k ∈ ℤ , thử lại thấy thỏa.
- Nếu f (1) = 1 thì f ( k ) = k , ∀k ∈ ℤ , thử lại thấy thỏa.
Vậy tất cả hàm số cần tìm là f ( k ) = k , ∀k ∈ ℤ ; f ( k ) = − k , ∀k ∈ ℤ và f ( k ) = 0, ∀k ∈ ℤ .

15
Bài 4. Chứng minh rằng:
a3 b3 c3 3
+ + ≥
(a + b) (b + c) (c + a) 8
3 3 3

trong đó a, b, c là các số thực dương.

*Trước hết, ta sẽ chứng minh bổ đề sau:


“Nếu a, b, c, d là các số thực dương có tích bằng 1 thì:
1 1 1 1
+ + + ≥ 1 .”
(1 + a ) (1 + b) (1 + c) (1 + d ) 2
2 2 2

Thật vậy:
1 1 2
Ta thấy với hai số thực dương tùy ý thì: + ≥ (*)
(1 + x) (1 + y ) 1 + xy
2 2

( x + 1) 2 + ( y + 1) 2 1
(*) ⇔ ≥ ⇔ ( x + 1) 2 + ( y + 1) 2  (1 + xy ) ≥ ( xy + x + y + 1) 2
( xy + x + y + 1) 2
1 + xy
⇔ ( x 2 + y 2 + 2 x + 2 y + 2)(1 + xy ) ≥ ( xy + x + y )2 + 2( xy + x + y ) + 1
⇔ ( x 2 + y 2 + 2 x + 2 y + 2) + ( x3 y + xy 3 + 2 x 2 y + 2 xy 2 + 2 xy ) ≥
≥ ( x 2 y 2 + x 2 + y 2 + 2 x 2 y + 2 xy 2 + 2 xy ) + (2 xy + 2 x + 2 y ) + 1
⇔ 1 + xy ( x 2 + y 2 ) ≥ 2 xy + x 2 y 2 ⇔ xy ( x − y ) 2 + (1 − xy )2 ≥ 0
Do đó:
1 1 1 1 1 1 2 + ab + cd 2 + ab + cd
+ + + ≥ + = = = 1.
(1 + a ) (1 + b) (1 + c) (1 + d ) 1 + ab 1 + cd 1 + ab + cd + abcd 2 + ab + cd
2 2 2 2

Do đó bổ đề được chứng minh.


Đẳng thức xảy ra khi và chỉ khi a = b = c = d = 1 .
Trong bổ đề trên, thay a = x, b = y, c = z , d = 1 , ta có kết quả sau:
Với x, y, z là các số thực dương và xyz = 1 thì:
1 1 1 3
+ + ≥ . Đẳng thức xảy ra khi và chỉ khi x = y = z = 1 .
(1 + x) (1 + y ) (1 + z )
2 2 2
4

*Trở lại bài toán đã cho:


b c a
Đặt x = , y = , z = ⇒ x, y, z > 0; xyz = 1 .
a b c
BĐT đã cho ban đầu tương đương với:
1 1 1 3 1 1 1 3
+ + ≥ ⇔ + + ≥ .
b 3 c 3
(1 + ) (1 + ) (1 + )
a 3 8 (1 + x ) (1 + y ) (1 + z )
3 3 3
8
a b c
Theo BĐT Cauchy cho các số dương, ta có:

16
1 1 1 1 3 1 1 3 1 1
+ + ≥ 33 = . ⇔ ≥ . − .
(1 + x) (1 + x ) 8
3 3
8.(1 + x ) 6
2 (1 + x) 2
(1 + x) 3
4 (1 + x) 16
2

Hoàn toàn tương tự:


1 3 1 1 1 3 1 1
≥ . − , ≥ . − .
(1 + y ) 3
4 (1 + y ) 16 (1 + z )
2 3
4 (1 + z ) 16
2

Cộng từng vế các BĐT này lại, ta được:


1 1 1 3  1 1 1  3
+ + ≥ . + + − .
(1 + x ) (1 + y ) (1 + z )
3 3 3
4  (1 + x ) (1 + x ) (1 + x) 2  16
2 2

Ta cần chứng minh:


3  1 1 1  3 3 1 1 1 3
. + + 2 
− ≥ ⇔ + + ≥
4  (1 + x ) (1 + x ) (1 + x )  16 8
2 2
(1 + x ) (1 + x ) (1 + x)
2 2 2
4
với x, y, z thỏa mãn các điều kiện đã nêu. (**)
Theo bổ đề trên thì (**) đúng.
Vậy ta có đpcm.
Dấu đẳng thức xảy ra khi và chỉ khi x = y = z = 1 ⇔ a = b = c .

17
Bài 5. Cho số nguyên tố p ( p > 3) . Tính:
p −1
2   2k 2  k2 
a. S = ∑    − 2    nếu p ≡ 1 (mod 4) .
k =1   p   p 
p −1
2 k2 
b. P = ∑   nếu p ≡ 1 (mod 8) .
k =1  p 

*Trước hết, ta sẽ chứng minh hai bổ đề sau:

p −1
(1) Bổ đề 1: Với p là số nguyên tố thỏa p ≡ 1 (mod 4) thì mỗi số tự nhiên a với: 1 ≤ a ≤
2
p +1
sẽ tồn tại duy nhất số tự nhiên b thỏa ≤ b ≤ p − 1 và: a 2 + b 2 ≡ 0(mod p ) .
2

*Chứng minh: Theo định lí Wilson: ( p − 1)! ≡ −1(mod p ) .

p −1
Với mỗi k = 1, 2,3,..., , ta thấy: p − k ≡ − k (mod p ) ⇒ k ( p − k ) ≡ −k 2 (mod p ) .
2

p −1
Kết hợp với giả thiết p ≡ 1(mod 4) ⇒ ⋮ 2 , ta được:
2
p −1 2 2
  p −1     p −1    p −1 
−1 ≡ ( p − 1)! ≡ (−1)  ! ≡    ! (mod p ) . Đặt ϕ =   ! ⇒ ϕ ≡ −1(mod p ) .
2 2
. 
  2     2    2 

p −1 p +1
Với mỗi 1 ≤ a ≤ , ta chọn ≤ b ≤ p − 1 thỏa b 2 ≡ a 2 .ϕ 2 (mod p ) , dễ thấy b tồn tại và duy
2 2
nhất. Khi đó: a + b ≡ a (1 + ϕ ) ≡ 0(mod p ) . Bổ đề được chứng minh.
2 2 2 2

(2) Bổ đề 2:
1 1
Với x là số thực bất kì thì [ 2 x ] − 2 [ x ] bằng 1 nếu ≤ {x} < 1 và bằng 0 nếu 0 ≤ { x} < .
2 2
*Chứng minh: Ta có: x = [ x ] + {x} . Suy ra:
[ 2 x] − 2 [ x] = [ 2[ x] + 2{x}] − 2 [[ x] + {x}] = [ 2{x}] − 2 [{x}] = [ 2{x}] . Do đó:
1
-Nếu ≤ {x} < 1 thì 1 ≤ 2{x} < 2 ⇒ [ 2{x}] = 1 ⇒ [ 2 x ] − 2 [ x ] = 1 .
2
1
- Nếu 0 ≤ { x} < thì 0 ≤ 2{x} < 1 ⇒ [ 2{x}] = 0 ⇒ [ 2 x] − 2 [ x ] = 0 .
2
Bổ đề được chứng minh. *Trở lại bài toán:

18
p −1
2   2k 2  k2  p −1
1. Ta thấy tổng đã cho là: S = ∑    − 2    có đúng 2 số hạng.
k =1   p   p 
Theo bổ đề 2 thì mỗi số hạng trong tổng đó nhận hai giá trị là 0 hoặc 1. (1)
p −1
Theo bổ đề 1 thì với mỗi số tự nhiên a thỏa 1 ≤ a ≤ thì tồn tại duy nhất số tự nhiên b thỏa
2
p +1
≤ b ≤ p − 1 sao cho a 2 + b 2 ≡ 0(mod p ) ⇒ a 2 + ( p − b) 2 ≡ 0(mod p ) ; do đó, tồn tại duy nhất
2
p −1
số tự nhiên a ' thỏa 1 ≤ a ' ≤ sao cho a 2 + a '2 ≡ 0(mod p ) .
2
p −1 p −1
Gọi x , y lần lượt là số các số dư của phép chia k 2 cho p ( 1 ≤ k ≤ ) có giá trị lớn hơn
2 2
p −1 p −1 p −1
và nhỏ hơn . Theo nhận xét trên thì x = y , hơn nữa x + y = ⇒x= y= . (2)
2 2 4
p −1
Từ (1) và (2), ta có: S = x.1 + y.0 = .
4
p −1
Do đó, tổng cần tìm là .
4
2. Do p ≡ 1 (mod 8) nên tồn tại a sao cho a 2 ≡ 2(mod p ) .
(ta cũng thấy rằng p ≡ 1 (mod 8) ⇒ p ≡ 1 (mod 4) ).
p −1 p −1 p −1 p −1
2 k 
2 2   2k 2   k 2   2   2 k 2   k 2   2   2k 2   k 2  
Ta có: P = ∑   = ∑    −   − ∑  − 2    = ∑   −  − S .
k =1  p  k =1   p   p   k =1   p   p   k =1   p   p  
Ta cần tính:
p −1 p −1 p −1 p −1 p −1
2   2k 2   k 2   2  2k 2 k 2  2   2 k 2   k 2   2  k 2  2   2 k 2   k 2  
∑ 
k =1   p 
 −   = ∑ −  − ∑  −   = ∑  − ∑   −   ,
 
p k =1  p p  k =1  p   
p k =1  
p k =1  p   p 
trong đó p ≡ 1 (mod 8) .
p −1
Theo nhận xét trên thì tập hợp các số dư khi chia k 2 ,1 ≤ k ≤ cho p trùng với tập hợp các số
2
p −1
p −1 2   2k 2   k 2  
dư khi chia 2k 2 ,1 ≤ k ≤
2
cho p, tức là: ∑ 
k =1   p 
 −    = 0 , suy ra:
 p 
p −1 p −1
2   2k 2   k 2   2  k 2  p 2 − 1
∑ 
k =1   p 
 −   = ∑  = .
 p   k =1  p  24
p 2 − 1 p − 1 ( p − 1)( p − 5)
Vậy P = − = .
24 4 24

19
Bài 6. Một số nguyên dương được gọi là “số kim cương 2005” nếu trong biểu diễn
thập phân của nó có 2005 số 9 đứng cạnh nhau liên tiếp. Dãy ( an ) , n = 1, 2,3,... là dãy tăng
ngặt các số nguyên dương thỏa mãn an < nC (C là hằng số thực dương nào đó).
Chứng minh rằng dãy số ( an ) , n = 1, 2,3,... chứa vô hạn “số kim cương 2005”.

Trước hết, ta sẽ chứng minh các bổ đề sau:


n
1
(1) lim ∑ = +∞ .
i =1 n

(2) Nếu trong hệ cơ số m (m ∈ ℕ , m > 1) : dãy số (an ) tăng và trong dãy đó không có số
n
1
hạng nào có chứa chữ số m − 1 thì tổng sau ∑a
i =1
hội tụ khi n tiến tới vô cực.
i

*Chứng minh bổ đề (1):


Ta cần chứng minh BĐT: x > ln( x + 1), ∀x > 0 . Thật vậy:
1 x
Xét hàm số: f ( x ) = x − ln( x + 1), x > 0 ⇒ f ′( x) = 1 − = > 0, ∀x > 0 .
x +1 x +1
Do đó, hàm số f ( x ) đồng biến trên (0; +∞) . Suy ra: f ( x ) > f (0) = 0 ⇒ x > ln( x + 1), ∀x > 0 .
1
Trong BĐT này, thay x bởi > 0 , ta cũng có:
x
1 1 1 x +1 1
> ln( + 1) ⇔ > ln( ) ⇔ > ln( x + 1) − ln x, ∀x > 0 . Áp dụng vào tổng cần chứng minh:
x x x x x
n n
1
∑ > ∑ [ ln( n + 1) − ln( n) ] = ln( n + 1) − ln1 = ln( n + 1) , mà lim [ ln(n + 1)] = +∞ nên:
i =1 n i =1
n
1
lim ∑ = +∞ . Bổ đề được chứng minh.
i =1 n

*Chứng minh bổ đề (2):


1
Đặt sk = ∑ là tổng các số tự nhiên có chứa k chữ số viết trong hệ cơ số m và không có
n
chứa chữ số m − 1 nào.
Giả sử một số hạng có k chữ số nào đó có dạng: b1b2 ...bk −1bk , chữ số thứ 1 phải khác 0 và
khác m − 1 nên có m − 2 cách chọn, các chữ số còn lại phải khác m − 1 nên có m − 1 cách chọn.
Do đó, có đúng (m − 2).(m − 1) k −1 số có k chữ số mà trong biểu diễn trong hệ cơ số m không có
chứa chữ số m − 1 , mà mỗi số trong đó đều lớn hơn m k −1 nên tổng nghịch đảo tương ứng của
(m − 2).(m − 1) k −1
chúng sẽ bé hơn .
m k −1

20
1
Hơn nữa: sk = ∑ là tổng các số hạng có chứa k chữ số trong hệ số m và không có chứa
n
chữ số m − 1 nào nên nó không vượt quá tổng của tất cả các số tự nhiên có cùng dạng đó mà ta
(m − 2).(m − 1) k −1
vừa đánh giá được, suy ra: sk < .
m k −1
Do đó:
n
1 n n
(m − 2).(m − 1) k −1 n
m − 1 k −1 m−2
lim ∑ = lim ∑ sk < lim ∑ = lim ∑ (m − 2).( ) = = m(m − 2) .
i =1 ai m k −1
m m −1
k =1 k =1 k =1
1−
m
Tức là tổng này hội tụ khi n tiến tới vô cực. Bổ đề (2) được chứng minh.

*Trở lại bài toán:


Đặt m = 102005 ⇒ m − 1 là số tự nhiên có chứa đúng 2005 số 9 liên tiếp khi viết trong hệ
thập phân.
Ta cần chứng minh trong dãy đã cho, có vô số số hạng chứa chữ số m − 1 .
Giả sử trong dãy này không có chứa số hạng nào có chữ số m − 1 . Khi đó, theo bổ đề (2)
n
1
ở trên: lim ∑ là hữu hạn.
i =1 ai
n n n
1 1 1 1
Hơn nữa, theo giả thiết: an < nC , ∀n nên lim ∑ > lim ∑ = .lim ∑ . Theo bổ
i =1 ai i =1 nC C i =1 n

đề (1), giới hạn này tiến tới vô cực. Hai điều này mâu thuẫn với nhau chứng tỏ điều giả sử ở trên
là sai, tức là dãy đã cho có ít nhất một số hạng chứa chữ số m − 1 , giả sử đó là: an0 .
Ta lại xét dãy con của dãy ban đầu: an0 +1 , an0 + 2 , an0 +3 ,...
Dãy này có đầy đủ tính chất của dãy đã cho nên cũng chứa ít nhất một số hạng có chứa chữ số
m − 1 khác với số an0 ở trên (do đây là dãy tăng).
Lập luận tương tự như thế, dãy con này có thêm một số hạng có chứa chữ số m − 1 .
Từ đó suy ra dãy đã cho có vô số số hạng chứa chữ số m − 1 .
Vậy dãy số ( an ) , n = 1, 2,3,... chứa vô hạn “số kim cương 2005”. Đây chính là đpcm.

21
LỜI GIẢI ĐỀ THI CHỌN ĐỘI TUYỂN QUỐC GIA

DỰ THI IMO 2006

Bài 1. Cho tam giác ABC có H là trực tâm. Đường phân giác ngoài của góc BHC cắt
các cạnh AB, AC lần lượt tại D và E. Đường phân giác trong của góc BAC cắt đường tròn
ngoại tiếp tam giác ADE tại điểm K.
Chứng minh rằng đường thẳng HK đi qua trung điểm của đoạn BC.

Trước hết ta sẽ chứng minh ∆ADE cân tại A.


 nên:
Thật vậy: Vì HD là phân giác góc ngoài của BHC
 = 1 ( HBC
DHB  + HCB ) = 1  (900 − 
ABC ) + (90 − 
1
ACB )  = BAC .
2 2  2

Do đó:   + DHB
ADE = DBH  = 900 − BAC + 1 BAC = 900 − 1 BAC .
2 2
1
Tương tự, ta cũng có: 
AED = 900 − BAC , suy ra: 
ADE =  AED , tức là tam giác ADE cân tại A.
2

Mặt khác AK là phân giác DAE
A nên cũng là trung trực của đoạn DE, do đó
AK chính là đường kính của đường tròn
ngoại tiếp ∆ADE .
B'
Từ đó, ta có: KD ⊥ AB , tương tự ta cũng
có: KE ⊥ AC .
E
Gọi P là giao điểm của KD và HB, Q là
C'
H giao điểm của KE và HC.
D
Q Ta có: KP ⊥ AB, QH ⊥ AB ⇒ KP // QH.
P Tương tự, ta cũng có: KQ // PH. Suy ra:
K KPHQ là hình bình hành, tức là HK đi qua
B C trung điểm của PQ.
Gọi BB’, CC’ là các đường cao của
PB DB QC EC
tam giác ABC. Theo định lí Thalès: DP // HC’ ⇒ = , QE // HB’ ⇒ = .
PH DC ' QH EB '
DB HB EC HC
Theo tính chất đường phân giác: = , = .
DC ' HC ' EB ' HB '
Vì B, C, B’, C’ cùng thuộc đường tròn đường kính BC nên theo tính chất phương tích:
HB HC PB QC
HB.HB ' = HC.HC ' ⇒ = . Từ các điều này, ta được: = ⇒ PQ // BC.
HC ' HB ' PH QH
Vì HK đi qua trung điểm của PQ nên cũng đi qua trung điểm của BC. Ta có đpcm.
22
Bài 2. Hãy tìm tất cả các cặp số tự nhiên (n ; k) với n là số nguyên không âm và k là số
nguyên lớn hơn 1 sao cho số : A = 17 2006 n + 4.17 2 n + 7.195 n có thể phân tích được thành tích
của k số nguyên dương liên tiếp.

Trước hết ta thấy rằng tích của 4 số tự nhiên liên tiếp phải chia hết cho 8 vì trong 4 số đó
có 1 số chia hết cho 4 và một số chia 4 dư 2.
Từ A = 17 2006 n + 4.17 2 n + 7.195 n , suy ra :
- Nếu n là số chẵn, ta có :
17 2006 n ≡ 1 (mod 8), 4.17 2 n ≡ 4.1 (mod 8), 7.195 n ≡ 7.32 n ≡ 7.310 ≡ 7 (mod 8)
Suy ra : A ≡ 12 ≡ 4(mod 8) , tức là A không chia hết cho 8.
- Nếu n là số lẻ, cũng tương tự :
17 2006 n ≡ 1 (mod 8), 4.17 2 n ≡ 4.1 (mod 8), 7.19 2 n ≡ 7.35 ≡ 7.3 ≡ 5(mod 8)
Suy ra : A ≡ 10 ≡ 2(mod 8) , tức là A cũng không chia hết cho 8.
Tức là trong mọi trường hợp luôn có A không chia hết cho 8.
Suy ra nếu k thỏa mãn đề bài thì k < 4 ⇒ k ∈ {2, 3} .
Xét từng trường hợp :
- Nếu k = 2 : tồn tại x tự nhiên sao cho A = x ( x + 1) .
+ Nếu n = 0 thì A = 12, x = 3, thỏa mãn đề bài.
+ Nếu n > 0 thì rõ ràng 171003n > 4.17 2 n + 7.195 n . Ta thấy :
A = x( x + 1) = 17 2006 n + 4.17 2 n + 7.195 n > 17 2006 n , suy ra x > 171003 n nhưng
x( x + 1) > 17 2006 n + 171003 n > A , mâu thuẫn.
Do đó, trong trường hợp này không có n thỏa mãn đề bài.
- Nếu k = 3 : tồn tại x tự nhiên sao cho A = x ( x − 1)( x + 1), x ≥ 1 ; dễ thấy x phải là số chẵn (vì
nếu ngược lại thì A chia hết cho 8, mâu thuẫn). Ta thấy :
A ≡ 12.(−1) n ≡ 2.( −1) n (mod 5) trong khi x( x − 1)( x + 1) = x( x 2 − 1) ≡ 0, ±1(mod 5) , mâu thuẫn.
Do đó, trong trường hợp này không có n thỏa mãn đề bài.
Vậy tất cả các cặp số thỏa mãn đề bài là (n; k ) = (0; 2) .

23
Bài 3. Trong không gian cho 2006 điểm mà trong đó không có 4 điểm nào đồng phẳng. Người
ta nối tất cả các điểm đó lại bởi các đoạn thẳng. Số tự nhiên m gọi là số tốt nếu ta có thể gán
cho mỗi đoạn thẳng trong các đoạn thẳng đã nối bởi một số tự nhiên không vượt quá m sao
cho mỗi tam giác tạo bởi ba điểm bất kì trong số các điểm đó đều có hai cạnh được gán bởi
hai số bằng nhau và cạnh còn lại gán bởi số lớn hơn hai số đó.
Tìm số tốt có giá trị nhỏ nhất.

Do trong các điểm đã cho không có bốn điểm nào đồng phẳng nên ba điểm bất kì trong
chung luôn tạo thành một tam giác. Gọi S(n) là giá trị nhỏ nhất của số tốt ứng với n điểm trong
không gian (n là số tự nhiên), ta sẽ xác định giá trị của S(2006). Ta chỉ xét các giá trị n ≥ 4 .
1
- Với n = 4 thì thử trực tiếp, ta thấy S(4) = 2. Bởi vì S(4) = 1 không
thỏa mãn nên S (4) ≥ 2 , ta sẽ chỉ ra rằng S(4) = 2 thỏa mãn. Cụ thể
2 ta có thể gán các đoạn thẳng như sau : gán 4 đoạn bất kì bởi số 1 và
2 1
1 2 đoạn còn lại bởi số 2, rõ ràng các tam giác tạo thành đều thỏa
mãn đề bài.
- Với một giá trị n > 4 bất kì, ta sẽ chứng minh rằng :
1
  n + 1 
S (n) ≥ 1 + S    .
 2 
Gọi a là số nhỏ nhất được gán cho các đoạn thẳng trong trường hợp có n điểm. Trong
trường hợp tối thiểu, không mất tính tổng quát, ta giả sử rằng a = 1, ta gọi hai đầu mút của đoạn
thẳng nào đó được gán số 1 là X và Y.
Trong n – 2 điểm còn lại, nếu có một điểm được nối với X và Y bởi một đoạn thẳng gán
bởi số 1 thì điểm đó cùng với X và Y sẽ tạo thành một tam giác đều không thỏa mãn đề bài.
Do đó, nếu gọi A là tập hợp tất cả các điểm nối với X bởi một đoạn thẳng gán số 1 (có tính luôn
điểm Y) và B là tập hợp tất cả các điểm nối với Y bởi một đoạn thẳng gán số 1 (có tính luôn
điểm X) thì giữa A và B không có phần tử nào chung hay A + B = n .
*Ta có các nhận xét sau :
- Nếu lấy một điểm bất kì trong tập A và một điểm bất kì trong B thì hai điểm đó cũng phải
được nối bởi đoạn thẳng gán số 1 vì nếu không thì hai điểm đó sẽ cùng với X sẽ tạo thành một
tam giác không thỏa mãn đề bài (tam giác đó hoặc không có hai số được gán trên hai cạnh bằng
nhau hoặc có hai cạnh bằng nhau nhưng cạnh còn lại gán số 1 nhỏ hơn).
- Hai điểm bất kì trong A được nối với nhau bởi một đoạn thẳng gán số lớn hơn 1 bởi nếu
không thì khi chọn thêm một điểm trong B, ta sẽ có một tam giác không thỏa mãn đề bài (tam
giác đó đều). Tương tự với tập hợp B. Tức là trong các tập A và B đều có chứa các số lớn hơn 1.
Tiếp theo, ta lại thấy trong mỗi tập A, B như vậy đều cần thêm S ( A ), S ( B ) số nữa để
 n − 1  n + 1
gán cho các đoạn thẳng. Giả sử A ≥ B thì A ≥   +1 =  .
 2   2 

24
Ta hoàn toàn có thể gán các số ở tập A trùng với các số ở tập B nên các số cần có thêm
  n + 1 
nữa là S     , tính thêm số 1 nhỏ nhất đã được gán cho đoạn XY ban đầu, ta được:
 2 
  n + 1 
S (n) ≥ 1 + S    .
 2 
Từ đó, áp dụng liên tiếp kết quả này, ta có: (chú ý rằng S(4) = 2).
S (2006) ≥ 1 + S (1003) ≥ 2 + S (502) ≥ ... ≥ 9 + A(4) = 11 .
Tiếp theo, ta sẽ chứng minh rằng giá trị 11 này thỏa mãn đề bài.
* Thật vậy :
Ta xây dựng cách gán các điểm từ thấp đến cao bằng cách ghép các bộ điểm ít hơn lại. Cụ thể
như sau :
- Đầu tiên ta xây dựng cho bộ 4 điểm. Cách gán tương tự như ở trên, nhưng trong trường hợp
này gán 4 đoạn bởi số 11 và 2 đoạn bởi số 10.
- Ghép 2 bộ này lại và tách ra từ một trong hai bộ đó ra 2 điểm, gán cho đoạn thẳng nối 2 điểm
đó bởi số 10, ta đã có tất cả 8 điểm.
- Tiếp tục ghép tương tự như vậy theo thứ tự như sau :
4 → 8 → 16 → 32 → 63 → 126 → 251 → 502 → 1003 → 2006
(Các trường hợp từ 32 đến 63 hoặc tương tự ta phải bỏ đi 1 điểm nào đó ở một trong hai bộ
ra ngoài). Mỗi lần ghép hai bộ điểm lại thì số gán trên đoạn được tách ra lại giảm đi 1 đơn vị,
đến khi ghép được 2006 điểm thì số đó chính là 1.
Dễ thấy cách gán số cho các đoạn thẳng này thỏa mãn đề bài.
Vậy giá trị nhỏ nhất của số tốt cần tìm là 11.

25
Bài 4. Chứng minh rằng với mọi số thực x, y, z ∈ [1; 2] , ta luôn có bất đẳng thức sau :
1 1 1 x y z
( x + y + z )( + + ) ≥ 6( + + ).
x y z y+z z+x x+ y
Hỏi đẳng thức xảy ra khi và chỉ khi nào ?

Trước hết ta thấy rằng :


1 1 1 ( x − y )2 x y z ( x − y )2
( x + y + z )( + + ) − 9 = ∑ , 6( + + )−9 = ∑ .
x y z xy y+ z z+ x x+ y ( y + z )( z + x)
Ta cần chứng minh :
1 1 1 x y z ( x − y)2 3( x − y ) 2
( x + y + z )( + + ) ≥ 6( + + )⇔∑ ≥∑
x y z y+z z+x x+ y xy ( y + z )( z + x)
với mọi số thực x, y, z thuộc đoạn [1 ; 2].
1 3 1 3 1 3
Đặt S x = − , Sy = − , Sz = − .
yz ( x + y )( x + z ) zx ( y + x )( y + z ) xy ( z + x)( z + y )
Bất đẳng thức đã cho viết dưới dạng tương đương là:
S x ( y − z ) 2 + S y ( z − x) 2 + S z ( x − y ) 2 ≥ 0 .
Không mất tính tổng quát, ta giả sử 2 ≥ x ≥ y ≥ z ≥ 1 .
Ta sẽ chứng minh rằng S x , S y ≥ 0 . Thật vậy:
S x ≥ 0 ⇔ x 2 + xy + xz − 2 yz ≥ 0 , đúng.
S y ≥ 0 ⇔ y 2 + yx + yz − 2 zx ≥ x( y − z ) + z ( z + y − x) ≥ 0 (do x, y, z ∈ [1; 2] nên y + z − x ≥ 0 ).
- Nếu S z ≥ 0 , ta có đpcm.
- Nếu S z < 0 , ta chứng minh được rằng S x + 2 S z ≥ 0, S y + 2 S z ≥ 0 .
Khi đó dễ dàng thấy rằng vì: ( x − y ) 2 ≤ 2  ( y − z ) 2 + ( z − x ) 2  và S z < 0 nên
S x ( y − z ) 2 + S y ( z − x)2 + S z ( x − y ) 2 ≥ ( S x + 2S z )( y − z ) 2 + ( S y + 2S z )( z − x) 2 ≥ 0
Vậy trong mọi trường hợp, ta luôn có đpcm.
Đẳng thức xảy ra khi x = y = z hoặc y = z = 1, x = 2 và các hoán vị của chúng.

26
Bài 5. Cho tam giác ABC là tam giác nhọn, không cân, nội tiếp trong đường tròn tâm O bán
kính R. Một đường thẳng d thay đổi sao cho d luôn vuông góc với OA và luôn cắt các tia AB,
AC. Gọi M, N lần lượt là giao điểm của đường thẳng d và các tia AB, AC. Giả sử các đường
thẳng BN và CN cắt nhau tại K; giả sử đường thẳng AK cắt đường thẳng BC.
1. Gọi P là giao của đường thẳng AK và đường thẳng BC. Chứng minh rằng đường tròn
ngoại tiếp của tam giác MNP luôn đi qua một điểm cố định khi d thay đổi.
2. Gọi H là trực tâm của tam giác AMN. Đặt BC = a và l là khoảng cách từ điểm A đến đường
thẳng HK. Chứng minh rằng đường thẳng HK luôn đi qua trực tâm của tam giác ABC.
Từ đó suy ra: l ≤ 4 R 2 − a 2 . Đẳng thức xảy ra khi và chỉ khi nào?

K O

B
Q P I C

1. Không mất tính tổng quát, giả sử AB < AC (trường hợp còn lại hoàn toàn tương tự).
Do tam giác ABC không cân nên AO không vuông góc với BC và MN không song song với BC,
do đó MN phải cắt đường thẳng BC tại một điểm, giả sử là Q; gọi I là trung điểm BC.
NA MB QB
Theo định lí Menelaus cho ba điểm Q, M, N thẳng hàng: . . =1.
NC MA QC
NA MB PB
Mặt khác, theo định lí Céva cho các đoạn AP, BN, CM đồng quy, ta có: . . = 1.
NC MA PC
PB QB
Từ đó, suy ra: = hay Q, B, P, C là một hàng điểm điều hòa, suy ra: IP.IQ = IB 2 = IC 2
PC QC
Do I là trung điểm BC nên OI ⊥ BC ⇒ QI 2 − BI 2 = OQ 2 − OB 2 , do đó:
QI .QP = QI 2 − QI .PI = QI 2 − IB 2 = OQ 2 − OB 2 = QB.QC
(do theo tính chất phương tích của Q đối với (O) thì OQ 2 − OB 2 = OQ 2 − R 2 = QB.QC ).
 = xAB
Mà tứ giác BMNC cũng nội tiếp vì có NCB = AMN (với Ax là tia tiếp tuyến của (O)).
Suy ra QM .QN = QB.QC .

27
Từ đó suy ra QM .QN = QP.QI , suy ra tứ giác MNIP nội tiếp hay đường tròn ngoại tiếp tam giác
MNP luôn đi qua điểm I cố định. Ta có đpcm.
2. Gọi BD, CE là hai đường cao của tam giác ABC, L là trực tâm của tam giác ABC; gọi MF,
NG là hai đường cao của tam giác AMN, H là trực tâm của tam giác AMN. Ta cần chứng
minh rằng H, K, L thẳng hàng.

F
G
D
H N
E

M
O
K

Q B P I C

Xét đường tròn (O1) đường kính BN và (O2) đường kính CM.
Ta thấy: KM.KC = KB. KN nên K có cùng phương tích đến (O1), (O2), tức là K thuộc trục đẳng
phương của hai đường tròn này.
Đồng thời, dễ thấy rằng các điểm D, G thuộc (O1) và M, F thuộc (O2).
Do H, L là trực tâm của tam giác ABC và AMN nên LB. LD = LC. LE, HN. HG = HE. HM; tức
là H, L cùng thuộc trục đẳng phương của hai đường tròn (O1), (O2).
Từ đó suy ra H, K, L cùng thuộc trục đẳng phương của (O1), (O2) nên chúng thẳng hàng.
Từ đó suy ra l ≤ AL .
BC 2
Mặt khác do tam giác ABC nhọn nên AL = 2OI = R 2 − = 4R2 − a2 .
4

Do đó AL = l ≤ 4 R 2 − a 2 . Đây chính là đpcm.


Đến đây, ta sẽ tìm vị trí của d sao cho đẳng thức xảy ra.
AN AM
Giả sử d cắt AB, AC tại M và N thỏa mãn = = k ⇒ MN = k .BC .
AB AC

28
Gọi R, S lần lượt là trung điểm của BN và CM; suy ra R, S cũng chính là tâm của hai
đường tròn (O1), (O2).
Ta thấy khi đẳng thức xảy ra thì AL vuông góc với trục đẳng phương của (O1), (O2), tức
là AL song song với đường nối tâm RS của hai đường tròn này, mà AL vuông góc với BC nên
RS phải vuông góc với BC.
       
Ta có: 2RS = BC + NM , mà RS .BC = 0 ⇒ ( BC + NM ).BC = 0 . Do góc tạo bởi MN và BC
=
chính là MQB ANM − 
ACB = 
ABC − 
ACB nên từ đẳng thức trên suy ra:
  1
BC 2 = BC.MN = BC.kBC.cos( B − C ) ⇒ k = .
cos( B − C )
1
Vậy đẳng thức xảy ra khi và chỉ khi k = , tức là đường thẳng d cắt AB tại M,
cos( B − C )
AN AM 1
AC tại N sao cho = = .
AB AC cos( B − C )

29
Bài 6. Cho dãy số thực (an ) được xác định bởi:
1 1
a0 = 1, an +1 = ( an + ) với mọi n = 1, 2, 3, …
2 3an
3
Chứng minh rằng với mọi số nguyên n, số An = là một số chính phương và nó có ít
3a − 1 2
n

nhất n ước nguyên tố phân biệt.

Trước hết, ta sẽ chứng minh rằng 3 An +1 = 4 An ( An + 3) với mọi n nguyên dương. (*)
Thật vậy:
3 9an2 9an2 108an2
Ta có: An + 3 = +3= nên 4 A ( A + 3) = 36 A . = .
3an2 − 1 3an2 − 1 3an2 − 1 ( 3a 2 − 1)2
n n n
n

Mặt khác:
9 9 108an2
3 An +1 = = = .
3an2+1 − 1 ( 3a − 1)
2
1 1 2 2
3. 2 ( an + ) −1 n
2 3an
Do đó: (*) được chứng minh, tức là 3 An +1 = 4 An ( An + 3) với mọi n nguyên dương.
Hơn nữa, ta tính được: An = 9 nên dễ dàng thấy rằng An chia hết cho 3 với mọi n.
An
Tiếp theo, ta sẽ chứng minh bằng quy nạp rằng: + 1 là số chính phương với mọi n (**).
3
A1
- Với n =1, + 1 = 4 là một số chính phương nên (**) đúng.
3
Ak
- Giả sử với n = k, (**) cũng đúng, tức là tồn tại số tự nhiên p sao cho + 1 = p2 .
3
Ak +1 4 A ( A + 3) A A
Ta có: +1 = k k + 1 = 4. k .(1 + k ) + 1 = 4 p 2 ( p 2 − 1) + 1 = (2 p 2 − 1) 2 cũng là
3 9 3 3
một số chính phương. Suy ra (**) cũng đúng với n = k + 1 .
Do đó, (**) được chứng minh.
A
Từ 3 An +1 = 4 An ( An + 3) ⇒ An +1 = 4 An .( n + 1) và (**), ta cũng dễ dàng chứng minh được bằng
3
quy nạp rằng An là số chính phương với mọi n.

Cuối cùng, ta sẽ chứng minh rằng An có ít nhất n ước nguyên tố đôi một khác nhau cũng bằng
phương pháp quy nạp. (***)
- Với n = 1, rõ ràng (***) đúng.
- Giả sử (***) đúng với n = k, tức là Ak có ít nhất k ước nguyên tố khác nhau.
Ta xét hai trường hợp:
30
+ Nếu k có ít nhất k + 1 ước nguyên tố khác nhau thì rõ ràng (***) đúng.
+ Nếu k có đúng k ước nguyên tố đôi một khác nhau, giả sử đó là p1 , p2 , p3 ,.., pk .
Khi đó: ( Ak , pi ), i = 1, k là 1 hoặc 3.
Giả sử Ak +1 chỉ có đúng k ước nguyên tố đôi một khác nhau là các giá trị ở trên thì cần phải có
Ak + 3 = 3m , m ∈ ℕ * , m ≥ 2 .
Nhưng khi đó thì Ak ≡ −3(mo d 9) không phải là số chính phương, mâu thuẫn.
Từ đó dẫn đến Ak +1 phải có một ước nguyên tố nào khác k ước đã có, tức là có ít nhất k + 1 ước
nguyên tố đôi một khác nhau hay (***) đúng với n = k + 1 .
Do đó (***) được chứng minh.
3
Vậy với mọi n nguyên dương, số An = 2 là một số chính phương và nó có ít nhất n
3an − 1
ước nguyên tố phân biệt, bài toán được giải quyết hoàn toàn.

31
LỜI GIẢI ĐỀ THI CHỌN ĐỘI TUYỂN QUỐC GIA

DỰ THI IMO 2007

Bài 1.
Cho hai tập hợp A,B là tập hợp các số nguyên dương thỏa mãn A = B = n (với n là số
nguyên dương) và có tổng các phần tử bằng nhau. Xét bảng ô vuông n × n .
Chứng minh rằng ta có thể điền vào mỗi ô vuông của bảng một số nguyên không âm
thỏa mãn đồng thời các điều kiện:
i/ Tập hợp tổng các số ở mỗi hàng là tập A.
ii/ Tập hợp tổng các số ở mỗi cột là tập B.
iii/ Có ít nhất (n − 1)2 + k số 0 trong bảng với k là số các phần tử chung của A và B.

1 2 i n Trước hết, ta thấy rằng nếu một giá trị k sao


cho tồn tại 2 phần tử bằng nhau ở mỗi tập là
1 0
ak = bk = t thì ta điền số t vào ô vuông nằm ở hàng thứ
2 0
k và cột thứ k, các ô còn lại của hàng thứ k và cột thứ k
0 đều điền vào số 0; như thế thì tổng các số ở hàng và cột
này thỏa mãn đề bài và không ảnh hưởng đến các hàng
i 0 0 0 0 0 0
và cột khác. Do đó, không mất tính tổng quát, ta xét
0 trường hợp A ∩ B = ∅ (trường hợp có các phần tử
chung thì điền thêm vào các hàng và cột theo cách
0
tương tự như trên), tức là số phần tử chung của hai tập
n 0 là k = 0 .

Ta sẽ chứng minh bài toán này bằng quy nạp. Gọi Τ là tập hợp các điều kiện i/, ii/, iii/
như trên (điều kiện iii/ tương ứng với trường hợp xét số nguyên dương n).

Với n = 1, bài toán hiển nhiên đúng.

Giả sử bài toán đúng với mọi số tập hợp có n – 1 phần tử. Ta sẽ chứng minh rằng với hai
tập A, B có n phần tử, ta cũng có thể xây dựng một bảng n × n thỏa mãn điều kiện Τ .

Thật vậy, xét hai tập hợp A = {a1 , a2 , a3 ,..., an }, B = {b1 , b2 , b3 ,..., bn } trong đó:
a1 < a2 < a3 < ... < an , b1 < b2 < b3 < ... < bn (hai tập này không có phần tử nào chung).

Giả sử a1 < b1 . Do tổng các phần tử ở hai tập bằng nhau nên tồn tại một chỉ số i thỏa mãn
ai > b1 > b1 − a1 ⇒ ai − (b1 − a1 ) > 0 . Ta xét hai tập hợp A*, B* như sau:

A* = {a2 , a3 ,..., ai −1 , ai − b1 + a1 ,..., an } , B* = {b2 , b3 ,..., bn } .


32
Hai tập hợp này có cùng số phần tử là n – 1 nên
1 2 3 n-1
theo giả thiết quy nạp, tồn tại một bảng có kích thước
1 (n − 1) × ( n − 1) thỏa mãn điều kiện Τ (trong bảng này có

2 ít nhất ( n − 2) 2 số 0).

Ta thêm vào bên trái bảng một cột và bên trên bảng một
hàng nữa như hình vẽ. Ở ô góc bên trái và phía trên, ta
i
điền số a1 , ở hàng thứ i của bảng ban đầu (hàng có tổng
các phần tử bằng ai − b1 + a1 ), ta điền số b1 − a1 ; còn tất
n-1 cả các ô còn lại của hàng và cột vừa thêm vào, ta điền
vào các số 0. Khi đó, bảng này có tổng các phần tử ở mỗi
hàng là tập A và tổng các phần tử ở mỗi cột là tập B, số các số 0 ở bảng vừa lập được không nhỏ
hơn ( n − 2) 2 + 2( n − 1) − 1 = ( n − 1) 2 và do đó nó thỏa mãn điều kiện Τ .

Do đó, bài toán cũng đúng với mọi tập hợp có n phần tử.

Theo nguyên lí quy tạp, bài toán này đúng với mọi số nguyên dương n.

Vậy ta có đpcm.

33
Bài 2.
Cho tam giác nhọn ABC với đường tròn nội tiếp I. Gọi (ka ) là đường tròn có tâm nằm
trên đường cao của góc A và tiếp xúc trong với đường tròn (I) tại A1 . Các điểm B1 , C1 xác
định tương tự .
1/ Chứng minh AA1 , BB1 , CC1 đồng qui tại P.
2/ Gọi ( J a ), ( J b ), ( J c ) lần lượt là các đường tròn đối xứng với đường tròn bàng tiếp các
góc A, B, C của tam giác ABC qua trung điểm BC, CA, AB.
Chứng minh P là tâm đẳng phương của 3 đường tròn nói trên.

1/ Trước hết, ta sẽ chứng minh bổ đề sau:


Cho tam giác ABC ngoại tiếp đường tròn (I) có D là tiếp điểm của đường tròn bàng tiếp
góc A lên BC. Gọi M, N là giao điểm của AD với (I) (N nằm giữa A và M). Giả sử IM cắt đường
cao AH tại K. Chứng minh rằng: KA = KM.
* Thật vậy:
A Gọi E là tiếp điểm của (I) lên
BC. Giả sử IE cắt (I) tại điểm thứ hai là
N’ khác E. Qua N’ vẽ đường thẳng
N C' song song với BC cắt AB và AC lần
B'
lượt tại B’ và C’. Dễ thấy tồn tại một
K
I phép vị tự biến tam giác AB’C’ thành
M tam giác ABC. Phép vị tự đó cũng biến
D C tiếp điểm N’ của đường tròn bàng tiếp
B (I) của ∆ AB’C’ lên B’C’ thành tiếp
H E điểm D của đường tròn bàng tiếp (J)
của ∆ ABC lên BC. Suy ra A, N’, D
thẳng hàng hay N’ trùng với N. Khi đó,
tam giác IMN đồng dạng với ∆ KMA
(do IN // AK), mà ∆ IMN cân tại I nên
∆ KAM cân tại K hay KA = KM.
Ta có đpcm.
Từ đây suy ra: đường tròn có tâm
J
thuộc đường cao góc A, đi qua A và
tiếp xúc với (I) tại M thì M thuộc AD.
Dễ thấy đường tròn đó là duy nhất.
*Trở lại bài toán:
Gọi D, E, F lần lượt là tiếp điểm của đường tròn bàng tiếp các góc A, B, C của tam giác
ABC lên các cạnh BC, CA, AB. Theo bổ đề trên, ta thấy: A1 ∈ AD, B1 ∈ CF , C1 ∈ BE .
Suy ra: AA1 , BB1 , CC1 đồng quy khi và chỉ khi AD, BE, CF đồng quy. (1)

34
B2 Mặt khác: nếu ta đặt
A BC = a, CA = b, AB = c,
AB + BC + CA
=p
2
C2 F thì có thể dễ dàng tính được:
DB = EC = p − c
C1 E
I P DC = AF = p − b
B1 AE = BF = p − a .
A1
DB EC FA
Suy ra: . . = 1,
B C DC EA FB
D
theo định lí Ceva đảo, ta có
AD, BE, CF đồng quy. (2)
Từ (1) và (2), ta có
AA1 , BB1 , CC1 đồng quy.
Ta có đpcm.

A2
B2
2/ Gọi A’, B’ lần lượt là
trung điểm của BC, CA; Ja
A2, B2, C2 lần lượt là tâm A
đường tròn bàng tiếp các
góc A, B, C của ∆ABC .
C2 E'
Gọi D’, E’ lần lượt là tiếp F
điểm của (I) lên BC, CA. B'
Dễ thấy D đối xứng với D’ I P
qua trung điểm A’ của BC,
A2 đối xứng với Ja qua A’
nên JaD’ // A2D, mà B D' A' C
A2 D ⊥ BC ⇒ J a D ' ⊥ BC .
Do đó: (Ja) tiếp xúc với BC
tại D’. Jb
Hoàn toàn tương tự:
(Jb) tiếp xúc với CA tại E’.
Ta có:
CD ' = CE ' nên phương tích
từ C đến (Ja) và (Jb) bằng A2
nhau, tức là C thuộc trục
đẳng phương của hai đường tròn này.
35
Ta sẽ chứng minh rằng CP, cũng chính là CF, vuông góc với đoạn nối tâm JaJb của hai
đường tròn (Ja), (Jb).
Theo cách xác định các điểm Ja, Jb, ta thấy A’ là trung điểm của A2Ja, B’ là trung điểm của B2J b.
       
Do đó: 2 A ' B ' = A2 B2 + J a J b hay J a J b = 2 A ' B ' − A2 B2 = BA − A2 B2 . Ta cũng có:
  
CF = CC2 + C2 F .
             
Ta có: J a J b .CF = ( BA − A2 B2 )(CC2 + C2 F ) = BA.CC2 + BA.C2 F − A2 B2 .CC2 − A2 B2 .C2 F =
   
= BA.CC2 − A2 B2 .C2 F (do C2F vuông góc với AB, A2B2 vuông góc với CC2). (1)
Mặt khác, ta thấy A, B, C chính là chân các đường cao của tam giác A2B2C2 nên rõ ràng:
∆C2 AB ∼ ∆C2 B2 A2 , mà C2F là đường cao của ∆C2 AB , C2C là đường cao của ∆C2 B2 A2 nên:
C2 F C2 C
= ⇒ C2 F . A2 B2 = AB.CC2 . Cũng từ hai tam giác ∆C2 AB , ∆C2 B2 A2 đồng dạng; ta có:
AB A2 B2
       
( BA, CC2 ) = ( A2 B2 , C2 F ) . Do đó: BA.CC2 = A2 B2 .C2 F . (2)
 
Từ (1) và (2), suy ra: J a J b .C2 F = 0 hay C2 F ⊥ J a J b .
Do đó C2F chính là trục đẳng phương của hai đường tròn (Ja), (Jb), tức là P thuộc trục
đẳng phương của hai đường tròn (Ja), (Jb).
Hoàn toàn tương tự, ta cũng có P thuộc trục đẳng phương của hai đường tròn (Jc), (Jb).
Từ đó suy ra P chính là tâm đẳng phương của (Ja), (Jb), (Jc).
Đây chính là đpcm.

36
Bài 3.
Cho tam giác ABC. Tìm giá trị nhỏ nhất của biểu thức sau:
A B B C C A
cos 2 cos 2 cos 2 cos 2 cos 2 cos 2
S= 2 2 + 2 2 + 2 2
C A B
cos 2 cos 2 cos 2
2 2 2
* Trước hết, ta sẽ chứng minh bổ đề:

“Với mọi a, b, c không âm và không đồng thời bằng 0, ta có:


1 1 1 9
+ + ≥ .
( a + b ) (b + c ) ( c + a )
2 2 2
4( ab + bc + ca )

Dấu đẳng thức xảy ra khi a = b = c hoặc a = b, c = 0 và các hoán vị.”

* Chứng minh:

Quy đổng và khai triển BĐT trên, ta cần chứng minh rằng:

∑ (4a b − a b
sym
5 4 2
− 3a 3b3 ) + ∑ ( a 4bc − 2a 3b 2 c + a 2b 2 c 2 ) ≥ 0
sym
(*)

Theo BĐT Schur cho các số không âm a, b, c, ta có:

∑ a(a − b)(a − c) ≥ 0 , nhân vào hai vế cho số abc không âm và khai triển, ta có:
sym

∑ (a bc − 2a b c + a b c
sym
4 3 2 2 2 2
)≥0. (1)

Hơn nữa, theo BĐT Cauchy, ta có:

∑ (4a b − a b
sym
5 4 2
− 3a 3b3 ) = ∑ ( a 5b − a 4b 2 ) + ∑ 3( a 5b − a 3b3 ) ≥ 0
sym sym
. (2)

Cộng từng vế các BĐT (1) và (2) lại, ta có BĐT (*).

Đẳng thức xảy ra trong (*) khi và chỉ khi đẳng thức xảy ra trong (1) và (2), tức là khi

a = b = c hoặc a = b, c = 0 và các hoán vị. Bổ đề được chứng minh.

* Trở lại bài toán:

A 1 B C
Ta có: cos 2 = . Tương tự với cos 2 , cos 2 .
A
2 tan 2 + 1 2 2
2

37
Thay các biến đổi này vào biểu thức đã cho, ta được:

x2 + 1 A B C
S =∑ , trong đó x = tan , y = tan , z = tan ; x, y , z > 0 .
( y + 1)( z + 1)
2 2
2 2 2

(Ở đây kí hiệu ∑ x là tổng đối xứng lấy theo các biến x, y, z).
Mặt khác, trong tam giác ABC, ta luôn có:
A B B C C A
tan tan + tan tan + tan tan = 1 ⇒ xy + yz + zx = 1 .
2 2 2 2 2 2

Do đó:

x2 + 1 ( x 2 + ∑ xy ) ( x + y )( x + z )
S =∑ = ∑ =∑ =
( y + 1)( z + 1)
2 2
( y + ∑ xy )( z + ∑ xy )
2 2
( y + z )( y + x)( z + x)( z + y )
1
=∑ .
( x + y)2

9 9
Áp dụng bổ đề ở trên, ta được: S ≥ = .
4( xy + yz + zx ) 4

9
Vậy giá trị nhỏ nhất của S là đạt được khi và chỉ khi x = y = z hay ABC là tam giác đều.
4

38
Bài 4. Tìm tất cả các hàm số liên tục f : ℝ → ℝ thỏa mãn:

x 1
f ( x) = f ( x 2 + + ) với mọi x ∈ ℝ .
3 9
x 1 1 1 1 1
Ta có: x 2 + + = ( x + ) 2 + ⇒ f ( x ) = f (( x + ) 2 + ), ∀x ∈ ℝ .
3 9 6 12 6 12
1 1 1 1
Đặt y = x + ⇔ x = y − . Thay vào giả thiết đã cho, ta có: f ( y − ) = f ( y 2 + ) . (*)
6 6 6 12
1 1
Xét hàm số: g ( x ) : ℝ → ℝ thỏa mãn: g ( x + ) = f ( x ) ⇔ f ( x − ) = g ( x ), ∀x ∈ ℝ . (1)
6 6
1
Từ (*), ta được: g ( x ) = g ( x 2 + ), ∀x ∈ ℝ , rõ ràng g ( x ) cũng liên tục.
4
Ta sẽ xác định hàm số g ( x ) thỏa mãn điều kiện trên.
1 1
Ta thấy: ∀x ∈ ℝ, g ( x ) = g ( x 2 + ) = g (( − x ) 2 + ) = g ( − x ) nên g ( x ) là hàm số chẵn.
4 4
Ta chỉ cần xét x ≥ 0 . Ta có hai trường hợp:
1 1 1
- Với x0 ≤ : Xét dãy số: u1 = x0 , un +1 = un2 + , n ≥ 1 . Khi đó: un +1 − un = (un − ) 2 ≥ 0 nên
2 4 2
1
dãy (un ) tăng. Mặt khác, bằng quy nạp, ta chứng minh được un ≤ , ∀n , tức là dãy này bị chặn
2
1 1
trên. Từ đó suy ra nó có giới hạn. Gọi t là giới hạn đó thì t = t 2 + ⇔ t = .
4 2
1  1
Do đó: g ( x ) = g ( ), ∀x0 ∈  0;  .
2  2
1 1
- Với x0 > : Tương tự như trên, xét dãy số v1 = x0 , vn+1 = vn − ≥ 0, n ≥ 1 .
2 4
1
−(vn − ) 2
1 2
Khi đó: vn +1 − vn = vn − − vn = < 0 nên dãy đã cho là dãy giảm. Cũng bằng quy
4 1
vn − + vn
4
1
nạp, ta chứng minh được vn > , tức là dãy đã cho bị chặn dưới, suy ra nó có giới hạn.
2
1 1
Gọi k là giới hạn đó thì k = k − ⇔k= .
4 2
1 1 1
Do đó: g ( x ) = g ( ), ∀x0 ∈ ( ; +∞) . Đặt g ( ) = a ∈ ℝ .
2 2 2
Từ đó suy ra: g ( x ) = a, ∀x ≥ 0 hay g ( x ) = a, ∀x ∈ ℝ . (2)
Từ (1) và (2), ta có: f ( x ) = a, ∀x ∈ ℝ . Đây là tất cả hàm số cần tìm.
39
Bài 5.
Cho A là tập con chứa 2007 phần tử của tập: {1, 2,3,..., 4013, 4014} thỏa mãn với mọi
a, b ∈ A thì a không chia hết cho b. Gọi mA là phần tử nhỏ nhất của A.
Tìm giá trị nhỏ nhất của mA với A thỏa mãn các điều kiện trên.

Chia tập hợp {1, 2,3,..., 4013, 4014} thành 2007 phần P1 ∪ P2 ∪ ... ∪ P2007 (mỗi tập hợp
chứa ít nhất một phần tử của {1, 2,3,..., 4013, 4014} ) thỏa mãn tập hợp Pa chứa tất cả các số
nguyên dương có dạng 2 n (2a − 1) , trong đó n là một số không âm. Khi đó, tập hợp con A của
{1, 2,3,..., 4013, 4014} không thể chứa hai phần tử cùng thuộc một trong 2007 tập hợp đó vì nếu
không thì rõ ràng có một số sẽ chia hết cho số còn lại, mâu thuẫn. Mặt khác, A lại có đúng 2007
phần tử nên A chứa đúng 1 phần tử của mỗi tập Pa nói trên.

Gọi α i là một phần tử của A với α i ∈ Pi , i = 1, 2007 . Xét các phần tử α1 , α 2 , α 5 ,...α1094 lần
lượt có các dạng 2 n , 3.2n , 32.2n ,..., ,37.2n ; rõ ràng mỗi phần tử đó chỉ có hai ước nguyên tố là 2 và
3. Ta cũng thấy rằng lũy thừa lớn nhất của 2 trong α1 phải lớn hơn lũy thừa lớn nhất của 2 trong
α 2 vì nếu ngược lại thì α 2 ⋮α1 , mâu thuẫn.

Hoàn toàn tương tự với các phần tử khác trong dãy α1 , α 2 , α 5 ,...α1094 , tức là nếu i < j thì
lũy thừa của 2 trong α i phải lớn hơn lũy thừa của 2 trong α j . Do đó, lũy thừa của 2 trong dãy
α1 , α 2 , α 5 ,...α1094 là một dãy giảm thực sự. Do có 8 phần tử trong dãy trên (tương ứng với lũy
thừa của 3 thay đổi từ 0 đến 7) nên giá trị của α1 ít nhất là 27 = 128 . Hơn nữa, các phần tử của
dãy có giá trị của lũy thừa 3 tăng dần (nếu ngược lại, giá trị của lũy thừa 3 giảm mà lũy thừa 2
cũng giảm nên có một số chia hết cho số khác, mâu thuẫn).

Suy ra: α1 chính là giá trị nhỏ nhất trong dãy trên.

Hoàn toàn tương tự với các số nguyên tố khác, ta cũng xét số có dạng α i = 2 ni .3ni ... pknk ,
trong đó 2, 3, …, pk là các ước nguyên tố không vượt quá 4014. Từ đó suy ra α1 cũng chính là
phần tử nhỏ nhất trong các phần tử của tập A. Do đó phần tử nhỏ nhất mA của tập A chính là α1 ,
như chứng minh ở trên α1 ≥ 128 hay mA ≥ 128 .

Ta sẽ chứng minh rằng 128 chính là giá trị nhỏ nhất của mA bằng cách chỉ ra một tập hợp
con A của {1, 2,3,..., 4013, 4014} thỏa mãn điều kiện đề bài.

Xét tập hợp: A = {α i = 2 f ( i ) (2i − 1) |1338 ≤ 3 f ( i ).(2i − 1) ≤ 4014} .

40
Rõ ràng tập hợp này có đúng 2007 phần tử thuộc {1, 2,3,..., 4013, 4014} .

Ta sẽ chứng minh rằng α x không chia hết cho α y , với x > y (tức là tập hợp này thỏa mãn đề bài).

Thật vậy, giả sử ngược lại tồn tại x, y thỏa mãn α x ⋮α y , khi đó: 2 f ( x ) (2 x − 1) ⋮ 2 f ( y ) (2 y − 1) ,
tức là f ( x ) ≥ f ( y ) và (2 x − 1)⋮ (2 y − 1) .

Từ cách xác định các giá trị u, v; ta có: (2 x − 1)⋮ (2 y − 1) ⇒ 2 x − 1 ≥ 3(2 y − 1) , đồng thời
3 f ( y ) +1 (2 y − 1) ≥ 4014 > 3 f ( x ) (2 x − 1) ≥ 3 f ( x ) +1 (2 y − 1) ⇒ f ( y ) > f ( x) . Mâu thuẫn này chứng tỏ tất
cả các phần tử của A đều thỏa mãn với mọi a, b ∈ A thì a không chia hết cho b.

Vậy 128 chính là giá trị nhỏ nhất của mA cần tìm.

41
Bài 6.
Cho đa giác 9 cạnh đều (H). Xét ba tam giác với các đỉnh là các đỉnh của đa giác (H)
đã cho sao cho không có hai tam giác nào có chung đỉnh.
Chứng minh rằng có thể chọn được từ mỗi tam giác 1 cạnh sao cho 3 cạnh này bằng nhau.

A1 a1 A2 Kí hiệu hình (H) đã cho là đa giác


A1 A2 A3 ... A8 A9 như hình vẽ. Trước hết, ta thấy
a2
rằng độ dài các cạnh và các đường chéo của
A A3 hình (H) chỉ thuộc 4 giá trị khác nhau (nếu
9
a3 gọi R là bán kính đường tròn ngoại tiếp của
a4 (H) thì ta dễ dàng tính được các giá trị đó là
π 2π 3π 4π
2 R.sin , 2 R sin , 2 R sin , 2 R sin )
A 9 9 9 9
8 A4 ta đặt chúng là a1 , a2 , a3 , a4 theo thứ tự tăng
dần của độ dài. Rõ ràng các tam giác có đỉnh
thuộc các đỉnh của (H) sẽ có cạnh có độ dài
thuộc 1 trong 6 dạng sau:
A A
7 5 ( a1 , a1 , a2 ), ( a2 , a2 , a4 ), ( a1 , a3 , a4 ),
.
A ( a3 , a3 , a3 ), ( a2 , a3 , a4 ), ( a4 , a4 , a1 )
6

Giả sử 3 tam giác được lấy ra là ∆1 , ∆ 2 , ∆ 3 .


Xét các trường hợp sau:

- Nếu trong các tam giác đó có một tam giác đều, rõ ràng, tam giác này phải có độ dài các

cạnh là 2 R.sin ; không mất tính tổng quát, giả sử đó là tam giác A1 A4 A7 . Do các tam giác
9
∆1 , ∆ 2 , ∆ 3 không có hai đỉnh nào trùng nhau nên ta sẽ lập một tam giác có các đỉnh là một trong
hai đỉnh của các tập hợp { A2 , A3 },{ A4 , A5 },{ A7 , A8 } . Ta sẽ chứng minh rằng tam giác đó phải có ít

nhất 1 cạnh có độ dài là 2 R.sin , tức là hai đỉnh có chỉ số có cùng số dư khi chia cho 3. Giả sử
9

ngược lại, trong hai tam giác cần lập, không có tam giác nào có cạnh là 2 R.sin , khi đó đỉnh
9
A2 phải nối với A4 và A4 phải nối với A8, nhưng khi đó A8 được nối với A2 là hai đỉnh có chỉ số
chia cho 3 cùng dư là 2, mâu thuẫn. Do đó, trong hai tam giác lập được, luôn có một cạnh có độ

dài là 2 R.sin . Suy ra trường hợp này luôn có tam giác thỏa mãn đề bài.
9

42
- Nếu trong các tam giác đó, không có tam giác nào đều . Khi đó các tam giác được xét
không có ba đỉnh cùng thuộc một trong ba tập hợp sau: α1 = { A1 , A4 , A7 } , α 2 = { A2 , A5 , A8 } ,
α 3 = { A3 , A6 , A9 } . Ta thấy một đoạn thẳng nối hai điểm bất kì thuộc hai tập khác nhau sẽ nhận 1
trong 3 giá trị là a1 , a2 , a4 . Hơn nữa, không có tam giác nào có độ dài 3 cạnh là (a1 , a2 , a4 ) nên ta
có hai nhận xét:

(1) Một tam giác có các đỉnh thuộc cả ba tập α1 , α 2 , α 3 nói trên thì sẽ có hai cạnh nào đó có độ
dài bằng nhau (các cạnh của nó có thể là ( a1 , a1 , a2 ) , ( a2 , a2 , a4 ) , (a4 , a4 , a1 ) ) tức là nó phải cân.

(2) Một tam giác có hai trong ba đỉnh thuộc cùng một tập thì tam giác đó các cạnh có độ dài
là ( a2 , a3 , a4 ) hoặc là ( a1 , a3 , a4 ) , tức là tam giác đó không cân.

* Ta xét tiếp các trường hợp (các tam giác xét dưới đây là cân nhưng không đều):

+ Có hai tam giác cân và một tam giác không cân: khi đó theo nhận xét (1), hai tam giác cân đó
phải có đỉnh thuộc các tập hợp khác nhau trong ba tập α1 , α 2 , α 3 ; khi đó, rõ ràng tam giác còn lại
cũng phải có đỉnh thuộc các tập hợp khác nhau, tức là nó phải cân, mâu thuẫn.

Vậy trường hợp này không tồn tại.

+ Có một tam giác cân và hai tam giác không cân: khi đó theo nhận xét (2), hai tam giác không
cân đó phải có hai đỉnh thuộc cùng một tập hợp và đỉnh còn lại thuộc tập hợp khác, giả sử một
tam giác có hai đỉnh thuộc α1 và một đỉnh thuộc α 2 ; rõ ràng tam giác không cân còn lại phải có
hai đỉnh thuộc α 2 , một đỉnh thuộc α 3 , suy ra tam giác còn lại có hai đỉnh thuộc α 3 , một đỉnh
thuộc α1 nên nó là tam giác cân, mâu thuẫn.

Vậy tương tự như trên, trường hợp này không tồn tại.

+ Cả ba tam giác đều không cân: khi đó theo nhận xét (2), tam giác đó thuộc một trong hai dạng
( a2 , a3 , a4 ) hoặc là ( a1 , a3 , a4 ) , tức là các tam giác này luôn chứa 1 cạnh có độ dài là a3.

Trong trường hợp này, bài toán được giải quyết.

+ Cả ba tam giác đều cân: khi đó, các tam giác có độ dài là ( a1 , a1 , a2 ) , ( a2 , a2 , a4 ) , (a4 , a4 , a1 ) .
Rõ ràng không tồn tại trường hợp có độ dài các cạnh lần lượt nhận cả ba giá trị như ba bộ trên
nên phải có hai bộ trùng nhau, tức là có ít nhất hai tam giác cân bằng nhau và một tam giác cân
nhận một trong ba giá trị thuộc một trong các bộ trên làm cạnh, khi đó luôn có thể chọn từ tam
giác này một cạnh bằng với cạnh đáy hoặc cạnh bên của hai tam giác cân bằng nhau kia.
Trong trường hợp này, bài toán cũng được giải quyết.
Vậy trong mọi trường hợp, ta luôn có đpcm.

43
LỜI GIẢI ĐỀ THI CHỌN ĐỘI TUYỂN QUỐC GIA
DỰ THI IMO 2008

Bài 1. Trong mặt phẳng cho góc xOy. Gọi M, N lần lượt là hai điểm lần lượt nằm trên
các tia Ox, Oy. Gọi d là đường phân giác góc ngoài của góc xOy và I là giao điểm của trung
trực MN với đường thẳng d. Gọi P, Q là hai điểm phân biệt nằm trên đường thẳng d sao
cho IM = IN = IP = IQ , giả sử K là giao điểm của MQ và NP.
1. Chứng minh rằng K nằm trên một đường thẳng cố định.
2. Gọi d1 là đường thẳng vuông góc với IM tại M và d2 là đường thẳng vuông góc với IN tại
N. Giả sử các đường thẳng d1, d2 cắt đường thẳng d tại E, F. Chứng minh rằng các đường
thẳng EN, FM và OK đồng quy.

d
E
Q

I N
O
y

P F

1. Xét trường hợp các điểm M, Q và N, P nằm cùng phía với nhau so với trung trực của MN.
Khi đó giao điểm K của MP và NQ thuộc các đoạn này.:
Gọi I’ là giao điểm của d với đường tròn ngoại tiếp ∆MON . Do d là phân giác ngoài của
 nên I’ chính là trung điểm của cung MON
MON
, do đó: I’M = I’N hay I’ chính là giao điểm
của trung trực MN với d. Từ đó, suy ra: I ≡ I ' hay tứ giác MION nội tiếp.
 = NMO
Ta được: NIO .
Mặt khác: do IM = IN = IP = IQ nên tứ giác MNPQ nội tiếp trong đường tròn tâm I, đường kính
 = 2 PMN
PQ ⇒ PIN  (góc nội tiếp và góc ở tâm cùng chắn cung PN ).
 = 2 PMN
Từ các điều trên, ta có: NMO  ⇒ MP là phân giác trong của OMN
.

44
.
Tương tự, ta cũng có: NQ là phân giác trong của ONM
Do K là giao điểm của MP và NQ nên K chính là tâm đường tròn nội tiếp của ∆MON , suy ra K
 , tức là K thuộc một đường thẳng cố định (đpcm).
thuộc phân giác trong của xOy
- Nếu giao điểm K nằm ngoài các đoạn MP và NQ: ta cũng có lập luận tương tự và có
 của tam giác ∆MON , tức là K cũng thuộc phân giác
được K là tâm đường tròn bàng tiếp MON
 , là một đường thẳng cố định.
trong của xOy
Trong mọi trường hợp, ta luôn có đpcm.

2. Gọi J là giao điểm của d1 và d2. Ta thấy tứ giác MINJ nội tiếp trong đường tròn đường
kính IJ. Hơn nữa: MION cũng là tứ giác nội tiếp nên 5 điểm M, N, I, J, O cùng thuộc một đường
 đi qua trung điểm của cung MJN
tròn. Do đó: phân giác trong góc MON
.
Rõ ràng M, N đối xứng nhau qua trung trực của MN nên JM = JN, tức là J cũng là trung
.
điểm của cung MON
 hay O, K, J thẳng hàng.
Từ đó suy ra: J thuộc phân giác trong của MON
Ta cần chứng minh các đoạn JO, EN và MF trong ∆JEF đồng quy.
OE SOEJ JO.JE.sin OJE JE sin OJE
Thật vậy: = = = . .
OF SOFJ JO.JF .sin OJF JF sin OJF
JE sin JFE OM sin ONM
Trong ∆JEF và ∆MON , ta có : = , = .
JF sin JEF ON sin OMN
 = OJN
Mặt khác : OJE  = ONM  , OJF  = OJM  = OMN  ⇒ sin OJE = sin ONM .
sin OJF sin OMN
OE sin JFE OM sin OFN OM sin OFN OM
Kết hợp lại, ta được : = . = . = .
OF sin JEF ON sin OEM ON sin OEM ON
sin OFN OM .sin MOE ME
= . = .
ON .sin NOF sin OEM NF
OE FN OE NF MJ
Do đó : . =1⇒ . . = 1.
OF EM OF NJ ME
Theo định lí Ceva đảo, ta có OI, EN và MF đồng quy. Đây chính là đpcm.

45
Bài 2. Hãy xác định tất cả các số nguyên dương m sao cho tồn tại các đa thức với hệ số
thực P ( x ), Q ( x ), R ( x, y ) thỏa mãn điều kiện:
Với mọi số thực a, b mà a m − b 2 = 0 , ta luôn có P ( R ( a, b)) = a và Q ( R ( a, b)) = b .

Với m là một số nguyên dương, ta xét các trường hợp :


- Nếu m là số chẵn, đặt m = 2k , k ∈ ℕ * . Suy ra : a m − b 2 = 0 ⇔ b = ± a k . Khi đó cần tìm k
sao cho các đa thức P ( x ) , Q ( x ) , R ( x, y ) thỏa mãn cả hai điều kiện :
(1) P ( R ( x, x k )) = x, Q ( R ( x, x k )) = x k , ∀x ∈ ℝ .
(2) P ( R ( x, − x k )) = x, Q ( R ( x, − x k )) = − x k , ∀x ∈ ℝ .
Xét đa thức một biến T(x) thỏa mãn : R ( x, x k ) = T ( x), ∀x . Theo giả thiết thì
P (T ( x )) = P ( R ( x, x k )) = x, ∀x . Từ đó suy ra : deg P ( x ).deg T ( x ) = 1 hay deg P ( x ) = deg T ( x ) = 1 .
Giả sử T ( x ) = ux + v, u , v ∈ ℝ , u ≠ 0 , P ( x ) = u '.x + v ', u ', v ' ∈ ℝ, u ' ≠ 0 thì
1 v x−v
u '(ux + v ) + v ' = x, ∀x ⇒ u.u ' = 1, u ' v + v ' = 0 hay u ' = , v ' = − ⇒ P ( x) = .
u u u
k
 x−u 
Mặt khác : Q ( R ( x, x )) = Q (T ( x)) = Q (ux + v ) = x ⇒ Q ( x ) = 
k k
 = P ( x ), ∀x .
k

 u 
Suy ra : Q ( R ( a, b)) = P k ( R ( a, b)) ⇒ b = a k , ∀a, b thỏa a m − b 2 = 0 . Nhưng theo điều kiện ban
đầu thì b cũng có thể là − a k . Mâu thuẫn này cho thấy các giá trị m trong trường hợp này không
thỏa mãn đề bài.
- Nếu m là số lẻ. Đặt P ( x 2 , x m ) = S ( x ) . Suy ra :
P ( S ( x )) = x 2 , ∀x ∈ ℝ ⇒ deg P ( x ).deg S ( x ) = 2 .
Nếu như degS(x) = 2 thì deg Q ( S ( x)) là số chẵn, trong khi đó : deg Q ( S ( x )) = deg x m = m
với m là số lẻ, mâu thuẫn. Suy ra : degS(x) = 1, degP(x) = 2.
Mặt khác, trong đa thức R ( x 2 , x m ) , bậc của nó có thể đạt giá trị nhỏ nhất là min(2, m) mà
deg R ( x 2 , x m ) = deg S ( x ) = 1 nên m = 1.
Ta sẽ chứng minh rằng giá trị m = 1 này thỏa mãn đề bài bằng cách chỉ ra các đa thức
P(x), Q(x), R(x, y) thỏa mãn đề bài.
Thật vậy :
Xét các đa thức P ( x ) = x 2 , Q ( x ) = x, R ( x, y ) = y .
Khi đó với m = 1 thì ta có quan hệ của a với b chính là : a = b 2 . Suy ra :
R ( a, b) = b , P ( R ( a, b)) = P (b) = b 2 = a , Q ( R ( a, b)) = P (b) = b , thỏa mãn đề bài.
Vậy tất cả các giá trị m thỏa mãn đề bài là m = 1.

46
Bài 3. Cho số nguyên n > 3 . Kí hiệu T là tập hợp gồm n số nguyên dương đầu tiên.
Một tập con S của T được gọi là tập khuyết trong T nếu S có tính chất: Tồn tại số nguyên
n
dương c không vượt quá sao cho với s1 , s2 là hai số bất kì thuộc S ta luôn có s1 − s2 ≠ c .
2
Hỏi tập khuyết trong T có thể có tối đa bao nhiêu phần tử ?

Trước hết ta thấy rằng: Nếu S là tập khuyết trong T thì tập S ' = {n − x | x ∈ S } cũng là một
tập khuyết trong T.
Thật vậy: Giả sử ngược lại S’ không phải là tập khuyết, khi đó tồn tại hai số nguyên
n
dương s '1 , s '2 ∈ S ' sao cho | s '1 − s '2 |= c với c là một số nguyên dương nào đó không vượt quá ,
2
Khi đó xét tương ứng hai phần tử s1 = n − s '1 , s2 = n − s '2 thì rõ ràng s1 , s2 ∈ S và
| s1 − s2 |=| ( n − s '1 ) − ( n − s '2 ) |=| s '1 − s '2 |= c , tức là tồn tại hai phần tử s1 , s2 ∈ S và
n
| s1 − s2 |= c ≤trong khi S là tập khuyết. Mâu thuẫn này suy ra nhận xét trên được chứng minh.
2
Hơn nữa, do | S |=| S ' | nên khi S có số các phần tử là lớn nhất thì tương ứng cũng có tập
S’ có số phần tử lớn nhất bằng với S.
n
Từ đó, ta thấy có thể xét các tập khuyết S có số các số phần tử không vượt quá không
2
n
ít hơn số các số phần tử lớn hơn . Xét hai tập hợp sau:
2
n n
A = {x | x ∈ S , x ≤ } , B = {x | x ∈ S , x > } thì A ∩ B = ∅, A ∪ B = S và theo cách xác định S
2 2
như trên thì A ≥ B .
n
Khi đó với c là một số nguyên dương nào đó không vượt quá , ta xét tập hợp:
2
C = {x + c | x ∈ A} . Ta có: | A |=| C | . Do A ⊂ S nên A cũng là một tập khuyết và khi đó rõ ràng
A ∩ C = ∅, B ∩ C = ∅ (vì nếu ngược lại thì tồn tại hai phần tử thuộc S mà hiệu của chúng là c,
mâu thuẫn).
Suy ra tất cả các phần tử thuộc tập A hoặc B hoặc C đều là một số nguyên dương không
vượt quá n, tức là ( A ∪ B ∪ C ) ⊂ T ⇒| A | + | B | + | C |≤| T |= n .
4 | A | +2 | B | 2 n
Kết hợp các điều này lại, ta được: 2 | A | + | B |≤ n . Do đó: | A | + | B |≤ ≤ .
3 3
 2n 
Hơn nữa: A ∩ B = ∅, A ∪ B = S và | S | là số nguyên nên | S |=| A | + | B |≤   .
3
 2n 
Do đó số phần tử của tập khuyết S trong T luôn không vượt quá   .
 3
47
 2n 
*Ta sẽ chỉ ra một tập khuyết thỏa mãn đề bài có đúng   phần tử.
 3
Thật vậy, xét hai tập hợp A, B như sau:
  n + 1   n n 
A = 1, 2, 3,...,    , B =  n −   + 1, n −   + 2,..., n  và S = A ∪ B .
  3   3 3 
 n + 1 n
Chọn c =  ≤ . Ta thấy:
 3  2
 n + 1  n + 1
- Hiệu hai phần tử bất kì trong A không vượt quá   −1 <  .
 3   3 
n n  n + 1
- Hiệu hai phần tử bất kì trong B không vượt quá n − ( n −   + 1) =   − 1 <  .
3 3  3 
- Hiệu một phần tử bất kì thuộc B với một phần tử bất kì thuộc A không nhỏ hơn:
n  n + 1 n n +1 n + 2  n + 1
( n −   + 1) −   ≥ n− − +1 = > .
3  3  3 3 3  3 
 n + 1 n
Khi đó, rõ ràng S = A ∪ B là một tập khuyết trong T ứng với giá trị c =  ≤ .
 3  2
 2n   n + 1 n
Ta sẽ chứng minh rằng | S |=   . Từ cách xác định A, B, ta có: | A |=   ,B =  .
3  3  3
 n + 1   n   2n 
Ta cần có:  + = , ∀n ∈ ℕ (*).
 3   3   3 
*Xét các trường hợp:
- Nếu n chia hết cho 3, tức là n có dạng 3m, m ∈ ℕ . Suy ra:
 n + 1   n   3m + 1   3m  6m  2n 
 3  +  3  =  3  +  3  = m + m = 2m = 3 =  3  .
- Nếu n chia 3 dư 1, tức là n có dạng 3m + 1, m ∈ ℕ . Suy ra:
 n + 1   n   3m + 2   3m + 1  6 m  6 m + 2   2n 
 3  +  3  =  3  +  3  = m + m = 2m = 3 =  3  =  3  .
- Nếu n chia 3 dư 2, tức là n có dạng 3m + 2, m ∈ ℕ . Suy ra:
 n + 1   n   3m + 3   3m + 2  6m + 3  6 m + 4   2n 
 3  +  3  =  3  +  3  = m + 1 + m = 3 =  3  =  3  .

 2n 
Từ đó suy ra (*) được chứng minh hay tập hợp S đã cho là tập khuyết có   .
 3
 2n 
Vậy giá trị lớn nhất của số phần tử của tập khuyết S trong T là   .
 3

48
Bài 4. Cho m, n là các số nguyên dương.
Chứng minh rằng (2m + 3) n + 1 chia hết cho 6m khi và chỉ khi 3n + 1 chia hết cho 4m.

Theo khai triển nhị thức Newton thì:


n −1
(2m + 3) n = (2m) n + 3n + ∑ Cnk .(2m) n − k .3k ≡ (2m) n + 3n (mod 6m) .
k =1

Do đó, 6m | (2m + 3) + 1 ⇔ 6m | (2m) n + 3n + 1 ⇔ 2m | (3n + 1) và 3 | (2m) n + 1 .


n

Cần chứng minh rằng: 2m | (3n + 1) và 3 | (2m) n + 1 (1) ⇔ 4m | 3n + 1 (2) .


Xét các trường hợp:
* Nếu m là số chẵn:
- Xét điều kiện (2):
3n + 1 không chia hết cho 4m vì 3n + 1 ≡ 2(mod 8) hoặc 3n + 1 ≡ 4(mod 8) , trong khi 4m ⋮8 , tức là
không thể có điều kiện (2).
- Xét điều kiện (1): từ m là số chẵn, suy ra 3n + 1⋮ 4 ⇒ n là số lẻ. Ta biết rằng: số có dạng
3n + 1 chỉ có ước nguyên tố lẻ đồng dư với 1 modun 4. Từ đó, suy ra m thỏa mãn: 2m | (3n + 1)
phải có dạng m = 2(3k + 1), k ∈ ℤ . Suy ra: (2m) + 1 = 2n.2n.(3k + 1) n + 1 ≡ 2(mod 3) ⇒ (2m) + 1
không chia hết cho 3, tức là điều kiện (1) cũng không tồn tại.
* Nếu m là số lẻ:
- Xét điều kiện (1): từ 2m | (3n + 1) suy ra m không chia hết cho 3, từ 3 | (2m) n + 1 suy ra n
phải là số lẻ vì nếu ngược lại thì (2m) n + 1 ≡ 2(mod 3) , mâu thuẫn. Mà n là số lẻ thì 3n + 1 ⋮ 4 ,
kết hợp với ( m, 4) = 1 , ta được 4m | 3n + 1 , đây chính là điều kiện (2). Do đó: (1) ⇒ (2) .
- Xét điều kiện (2): từ 4m | 3n + 1 ⇒ (4 | 3n + 1) ∧ ( m | 3n + 1) suy ra n là số lẻ và m có dạng
3k + 1, k ∈ ℤ . Suy ra: (2m) n + 1 = 2 n.m n + 1 ≡ −1 + 1 ≡ 0(mod 3) ⇒ 3 | (2m) n + 1 ; từ (2) ta cũng trực
tiếp có 2m | (3n + 1) . Do đó: (2) ⇒ (1) .
Kết hợp các điều trên lại, ta được: (1) ⇔ (2) .
Vậy (2m + 3) n + 1 chia hết cho 6m khi và chỉ khi 3n + 1 chia hết cho 4m.
Đây chính là đpcm.

49
Bài 5. Cho tam giác ABC nhọn, không cân có O là tâm đường tròn ngoại tiếp.
Gọi AD, BE, CF lần lượt là các đường phân giác trong của tam giác. Trên các đường thẳng
AL BM CN
AD, BE, CF lần lượt lấy các điểm L, M, N sao cho = = = k (k là hằng số dương).
AD BE CF
Gọi (O1), (O2), (O3) lần lượt là các đường tròn đi qua L, tiếp xúc với OA tại A ; đi qua M tiếp
xúc với OB tại B và đi qua N tiếp xúc với OC tại C.
1
1. Chứng minh rằng với k = , ba đường tròn (O1), (O2), (O3) có đúng hai điểm chung và
2
đường thẳng nối hai điểm đó đi qua trọng tâm tam giác ABC.
2. Tìm tất cả các giá trị k sao cho 3 đường tròn (O1), (O2), (O3) có đúng hai điểm chung.

Trước hết, ta nêu 4 bổ đề sau:


(1) Cho ba đường thẳng đôi một phân biệt a, b, c và hai đường thẳng phân biệt d, d’. Các
đường thẳng d, d’ theo thứ tự cắt a, b, c tại A1 , B1 , C1 ; A2 , B2 , C2 thỏa mãn điều kiện:
A1 B1 A2 B2 AA BB CC
= = k . Các điểm A3, B3, C3 thuộc a, b, c sao cho: 1 2 = 1 2 = 1 2 .
A1C1 A2C2 A1 A3 B1B3 C1C3
A3 B3
Khi đó, A3, B3, C3 thẳng hàng và =k.
A3C3

(2) Cho ba đường thẳng phân biệt a, b, c và ba đường thẳng phân biệt khác a’, b’, c’ . Các
đường thẳng a’, b’, c’ theo thứ tự cắt a, b, c tại A1 , B1 , C1 ; A2 , B2 , C2 ; A3 , B3 , C3 (các điểm này đôi
A1 B1 A2 B2 A3 B3 AA BB CC
một phân biệt). Khi đó nếu = = thì hoặc 1 2 = 1 2 = 1 2 hoặc a, b, c đôi
A1C1 A2C2 A3C3 A1 A3 B1B3 C1C3
một song song.

(3) Cho tam giác ABC và M bất kì. Các tia AM, BM, CM lần lượt cắt BC, CA, AB ở A1, B1,
C1. Các đường thẳng A1B1, B1C1, C1A1 cắt các đường thẳng AB, BC, CA lần lượt ở A2, B2, C2.
Các điểm A3, B3, C3 theo thứ tự nằm trên các đường thẳng BC, CA, AB sao cho
A1 A3 B1 B3 C1C3 1
= = = k , k ≠ 0 . Khi đó, A3, B3, C3 thẳng hàng khi và chỉ khi k = 1 hoặc k = .
A1 A2 B1B2 C1C2 2

(4) Cho tam giác ABC không cân ngoại tiếp đường tròn (I). Đường tròn (I) tiếp xúc với các
cạnh BC, CA, AB lần lượt tại D, E, F. Đường thẳng EF cắt BC tại M, đường thẳng AD cắt (I) tại
N (khác D). Chứng minh rằng: MN tiếp xúc với (I).
Các bổ đề (1), (2) có thể chứng minh dễ dàng bằng các biểu diễn theo vectơ.
Dưới đây trình bày các chứng minh cho bổ đề (3), (4).

50
*Chứng minh bổ đề (3).
+ Điều kiện đủ:
- Với k = 1, ta có A3, B3, C3 theo thứ tự trùng với
C2
A2, B2, C2. Vì AA1, BB1, CC1 đồng quy nên theo định lí
A1 B B1C C1 A
Menelaus thì . . = −1 . Vì A2, B1, C1 thẳng
A A1C B1 A C1 B
A2 B B1C C1 A AB AB
C1
hàng nên . . = 1 . Suy ra: 1 = − 2 .
A2C B1 A C1 B A1C A2C

B1 B1C BC CA C A
M Tương tự: =− 2 , 1 =− 2 .
B1 A B2 A C1 B C2 B
A2 Nhân từng vế các đẳng thức trên,
B A1 C
A2 B B2C C2 A  A1 B   B1C   C1 A 
. . = −  . −  . −  = −1 .
A2C B2 A C2 B  A1C   B1 A   C1 B 
B2
Tức là A2, B2, C2 thẳng hàng hay A3, B3, C3 thẳng hàng.
1
- Với k =, A3, B3, C3 theo thứ tự là trung điểm của C2
2
A1A2, B1B2, C1C2. Theo chứng minh trên, ta đã có: C3

A1 B AB
= − 2 . Theo tính chất tỉ lệ thức thì: A
A1C A2C
C1
A1 B A B A B + A2 B A1 B − A2 B A B 2A B A A
=− 2 = 1 = ⇒ 1 = 3 = 2 1
A1C A2C A1C − A2C A1C + A2C A1C A2 A1 2 A3C B1
M
2
A3 B 2 A3 B A2 A1  A1 B 
Suy ra: = . =  . Tương tự: A3
A2
A3C A2 A1 2 A3C  A1C  B A1 C
2 2
B3
B3C  B1C  C3 A  C1 A 
=  , =  .
B3 A  B1 A  C3 B  C1 B 
B2
Nhân từng vế các đẳng thức trên, ta được:
2 2 2
A3 B B3C C3 A  A1 B   BC  C A
. . = .  1  .  1  = 1 . Do đó, A3, B3, C3 th ẳng hàng.

A3C B3 A C3 B  A1C   B1 A   C1 B 
+ Điều kiện cần: Khi k ≠ 1 , ta kí hiệu A3( k ) , B3( k ) , C3( k ) thay cho A3, B3, C3. Giả sử tồn tại số k
1
đồng thời khác 1 và mà A3( k ) , B3( k ) , C3( k ) thẳng hàng. Khi đó, các điểm: A3( k ) , B3( k ) , C3( k ) và
2
A2 A3(1/2) B2 B3(1/2) C2C3(1/ 2) 1/ 2 − 1
A3(1/2) , B3(1/ 2) , C3(1/ 2) đôi một khác nhau. Dễ thấy: = = = .
A2 A3( k ) B2 B3( k ) C2C3( k ) k −1

51
Theo chứng minh ở điều kiện đủ thì hai bộ điểm A2, B2, C2 và A3(1/2) , B3(1/ 2) , C3(1/ 2) thẳng
hàng, mà theo điều giả sử ở trên thì A3( k ) , B3( k ) , C3( k ) cũng thẳng hàng nên theo bổ đề (2), hoặc

A2 B2 A3(1/ 2) B3(1/ 2)
đường thẳng A2B2C2 và A3(1/ 2) B3(1/ 2) C3(1/ 2) song song hoặc = .
A2C2 A3(1/2)C3(1/2)

A2 B2 A3(1/ 2) B3(1/ 2) A1 A3(1/ 2) B1 B3(1/ 2) C1C3(1/ 2)


+ Nếu = thì chú ý rằng: = = , theo bổ đề (1) thì
A2C2 A3(1/2)C3(1/2) A1 A2 B1 B2 C1C2
A1, B1, C1 thẳng hàng, mâu thuẫn.
+ Nếu A2B2C2 và A3(1/ 2) B3(1/ 2) C3(1/ 2) song
C 3(k) C2 song với nhau thì chú ý rằng
A3(1/2) , B3(1/ 2) , C3(1/ 2) theo thứ tự là trung
C3(1/2)
điểm của A1 A2 , B1 B2 , C1C2 . Ta có:
A
 1  
A3(1/ 2) B3(1/ 2) = ( A1 B1 + A2 B2 ) ,
C1 2
 1  
A3(1/ 2)C3(1/ 2) = ( A1C1 + A2C2 ) . Suy ra:
2
M B1
A1B1 song song với A2B2 và A3(1/ 2) B3(1/ 2) ,
A3(1/2)

C
A2 A1C1 song song với A2C2 và A3(1/ 2) C3(1/ 2) .
B A1 A3(k)
B3(1/2) Từ đó suy ra, A1, B1, C1 cũng thẳng
B3(k) hàng, mâu thuẫn.
B2
1
Do đó chỉ có k = 1 và k = thỏa mãn.
2
Vậy bổ đề (3) được chứng minh.
*Chứng minh bổ đề (4).
Gọi H là giao điểm của A
EF và AI. Ta thấy: IA ⊥ EF .
Tam giác AIF vuông tại F có
N
đường cao FH nên : F
H
IF 2 = IH .IA ⇒ ID 2 = IH .IA .
Suy ra: ∆IDH ∼ ∆IAD (c.g .c) .
I E
 = IDA
Do đó: IHD .
Mặt khác: tam giác IDN cân tại
 = IDN
I nên IND  = IDA . Từ
B D C M
 = IHD
đó, ta được: IND .
 = IHM
⇒ Tứ giác IDNH nội tiếp. Hơn nữa, tứ giác IDMH cũng nội tiếp vì có IDM  = 900 .
Do đó: 5 điểm, I, D, M, N, H cùng thuộc một đường tròn. Suy ra: IMNH nội tiếp hay
 = IHM
INM  = 900 ⇒ MN ⊥ IN . Vậy MN là tiếp tuyến của (I). Bổ đề (4) được chứng minh.

52
*Trở lại bài toán.
1
1. Khi k = thì L, M, N lần lượt là trung điểm của các đoạn AD, BE, CF.
2
Gọi H là trực tâm của ∆ABC và δ là phương tích của H đối với đường tròn Euler đi qua chân 3
đường cao của ∆ABC . Gọi K là giao điểm của đường thẳng AO1 với đường thẳng BC.
Ta sẽ chứng minh rằng K nằm trên (O1).
Thật vậy:
Do ∆ABC là tam giác nhọn nên O nằm trong tam giác. Ta có:

AOB = 2   = 900 − 
ACB ⇒ OAB ACB .

E
L
O
1
F
O
M
N

H
C

B D
K

Không mất tính tổng quát, giả sử tia AD nằm giữa hai tia AO và AB. Khi đó:

BAC 
BAC
 = OAB
OAD  − DAB = 900 −  ACB −  = 900 − OAD
⇒ KAD = ACB +
2 2 .

Mặt khác:   + DCA


ADB = DAC  = 1 BAC +  = KDA
ACB nên KAD .
2
Ta cũng có O A = O L ⇒ ∆AO L cân tại O1 nên O  
AL = O LA .
1 1 1 1 1

 
Từ đó suy ra: O1 LA = KDA hay O1L // KD, mà L là trung điểm của AD nên O1 là trung điểm của

AK hay K thuộc đường tròn (O1). Do đó (O1) cắt BC tại chân đường cao của ∆ABC . Từ đó suy
ra phương tích của H đối với đường tròn (O1) chính là δ .
Hoàn toàn tương tự với các đường tròn (O2), (O3).
53
Do H có cùng phương tích đến các đường tròn (O1), (O2), (O3) nên H chính là tâm đẳng
phương của 3 đường tròn (O1), (O2), (O3) .
Hơn nữa: do OA là tiếp tuyến của (O1) tại A nên phương tích của O đối với (O1) chính là
2
OA . Tương tự như vậy, phương tích của O đối với đường tròn (O2) và (O3) lần lượt là
OB 2 , OC 2 , mà O là tâm đường tròn ngoại tiếp của ∆ABC nên OA = OB = OC hay O có cùng
phương tích đến các đường tròn (O1), (O2), (O3), suy ra: O cũng là tâm đẳng phương của 3 đường
tròn (O1), (O2), (O3).
Giả sử của 3 đường tròn (O1), (O2), (O3) có 3 trục đẳng phương khác nhau thì chúng phải
đồng quy tại tâm đẳng phương, mà O và H cùng là tâm đẳng phương của chúng nên O phải trùng
với H hay ∆ABC đều, mâu thuẫn với giả thiết ∆ABC không cân.
Do đó, điều giả sử trên là sai và 3 đường tròn đã cho phải có 1 trục đẳng phương chung,
trục đẳng phương đó chính là đường thẳng đi qua O và H. Ta cũng thấy rằng O nằm ngoài cả 3
đường tròn, H thì nằm giữa các đường cao của ∆ABC nên nó nằm trong cả 3 đường tròn.
Suy ra đường thẳng OH cắt cả 3 đường tròn tại 2 điểm nào đó.
Vậy 3 đường tròn (O1), (O2), (O3) có đúng 2 điểm chung, hơn nữa, đường thẳng đi qua
hai điểm chung đó chính là đường thẳng OH và do đó, nó cũng sẽ đi qua trọng tâm của tam giác
(đường thẳng Euler). Ta có đpcm.

2. Ta sẽ chứng minh rằng ba đường tròn (O1), (O2), (O3) có đúng hai điểm chung khi và chỉ
1
khi k = 0 hoặc k = . Thật vậy:
2
*Điều kiện đủ:
1
- Khi k = , khẳng định đã chứng minh ở câu 1/.
2
- Ta sẽ tiếp tục chứng minh rằng với k = 1, ba đường tròn (O1), (O2), (O3) lần lượt đi qua
L, tiếp xúc với OA tại A ; đi qua M tiếp xúc với OB tại B và đi qua N tiếp xúc với OC tại C cũng
có đúng hai điểm chung. Thật vậy:
- Khi k = 1, các điểm L, M, N tương ứng trùng với các điểm D, E, F.
Theo chứng minh ở câu 1/, đường tròn (K, KA) đi qua D và tiếp xúc với OA tại A nên
chính là đường tròn (O1) đang được xét. Gọi d1, d2, d3 là tiếp tuyến của đường tròn (O) lần lượt
tại A, B, C. Gọi X, Y, Z theo thứ tự là giao điểm của d 2 , d 3 ; d3 , d1 ; d1 , d 2 .
Vì O1 thuộc đường thẳng BC và OA tiếp xúc với (O1) tại A nên O1 thuộc d1, từ đó suy ra
O1 chính là giao điểm của BC và d1.
Tương tự: O2, O3 lần lượt chính là giao điểm của CA và d2, AB và d3.
Qua các điểm O1, O2, O3 vẽ các tiếp tuyến tới đường tròn (O) lần lượt là O1T1 , O2T2 , O3T3
(trong đó T1, T2, T3 là các tiếp điểm).
Ta có: O1T1 = O1 A, O2T2 = O2 B, O3T3 = O3C , tức là T1, T2, T3 cũng tương ứng thuộc các đường
tròn (O1), (O2), (O3).

54
Theo bổ đề (4) ở trên, (xét tam giác XYZ có (O) là đường tròn nội tiếp) các đường thẳng
AT1, BT2, CT3 tương ứng trùng với các đường thẳng AX, BY, CZ.
Hơn nữa, XB = XC, YC = YA, ZA = ZA nên:
AY BZ CX
. . = −1 ⇒ AX, BY, CZ đồng quy (theo định lí Ceva đảo trong tam giác XYZ).
AZ BX CY
Do đó: AT1, BT2, CT3 đồng quy. Đặt điểm chung của ba đường thẳng đó là S, rõ ràng S
nằm trong (O). Do T1, T2, T3 nằm trên (O) nên theo tính chất phương tích:
SA.ST1 = SB.ST 2 = SC.ST3 ⇒ PS /(O1 ) = PS /(O2 ) = PS /(O3 )
.
Tương tự câu 1/, ta có: PO /( O1 ) = PO /( O2 ) = PO /( O3 ) , tức là OS là trục đẳng phương chung của ba
đường tròn O1), (O2), (O3).
Mặt khác, S nằm trong cả ba đường tròn, O nằm ngoài cả ba đường tròn nên đường thẳng
OS cắt cả ba đường tròn tại hai điểm, tức là (O1), (O2), (O3) có đúng hai điểm chung.
Vậy trong trường hợp k = 1, ba đường tròn (O1), (O2), (O3) cũng có đúng hai điểm chung.
Điều kiện đủ của khẳng định trên được chứng minh.

O3

T3
A
Y
T2
S
O

B C O1
T1

O2

55
*Điều kiện cần:
Với một giá trị k > 0, k ≠ 1 , gọi O1( k ) , O2( k ) , O3( k ) lần lượt là tâm của các đường tròn đi qua
L, tiếp xúc với (O) tại A; đi qua M, tiếp xúc với (O) tại B, đi qua N, tiếp xúc với (O) tại N.
Giả sử các đường tròn
Z O3 ( O ) , ( O ) , ( O ) nói trên có
1( k ) 2( k ) 3( k )

đúng hai điểm chung, tức là ba


tâm của chúng là O1( k ) , O2( k ) , O3( k )
O3(k)
thẳng hàng. (1)
Gọi d1, d2, d3 là tiếp tuyến
của đường tròn (O) lần lượt tại A,
A B, C. Gọi X, Y, Z theo thứ tự là
Y
S
giao điểm của d2, d3; d3, d1; d1, d2.
O1(k)
O Chứng minh tương tự như trên,
B C
AX, BY, CZ đồng quy. (2)
O1
Đặt O1, O2, O3 là giao
điểm của BC với YZ, CA với ZX,
AB với XY. Dễ thấy rằng:
X
O1( k ) , O2( k ) , O3( k ) lần lượt thuộc các
đoạn thẳng AO1 , BO2 , CO3
AO1( k ) AL BO2( k ) BM
và = , = ,
AO1 AD BO2 BE
O2(k)
CO3( k ) CN
= .
CO3 CF
O2
Suy ra:
AO1( k ) BO2( k ) CO3( k )
= = =k. (3)
AO1 BO2 CO3
1
Từ (1), (2), (3), áp dụng bổ đề 3, ta có k = 1 hoặc k = .
2
1
Do đó, nếu các đường tròn (O1), (O2), (O3) có đúng hai điểm chung thì k = 1 hoặc k = .
2
Điều kiện cần của khẳng định được chứng minh.
1
Vậy tất cả các giá trị k cần tìm là k = 1 và k = .
2
Bài toán được giải quyết hoàn toàn.

56
Bài 6. Kí hiệu M là tập hợp gồm 2008 số nguyên dương đầu tiên. Tô tất cả các số
thuộc M bởi ba màu xanh, vàng, đỏ sao cho mỗi số được tô bởi một màu và mỗi màu đều
được dùng để tô ít nhất một số. Xét các tập hợp sau:
S1 = {( x, y , z ) ∈ M 3 , trong đó x, y, z có cùng màu và ( x + y + z ) ≡ 0 (mod 2008)} ;
S1 = {( x, y , z ) ∈ M 3 , trong đó x, y, z đôi một khác màu và ( x + y + z ) ≡ 0 (mod 2008)} .
Chứng minh rằng 2 S1 > S2 . (Kí hiệu M 3 là tích Đề các M × M × M ) .

*Trước hết ta sẽ chứng minh bổ đề sau:


“Với n là số nguyên dương, xét tập hợp M = {1, 2, 3, …, n}. Tô màu các phần tử của S bởi
màu xanh hoặc đỏ. Xét các tập hợp sau:
S1 = {( x, y , z ) ∈ M 3 , trong đó x, y, z cùng màu và ( x + y + z ) ≡ 0 (mod n)} ;
S 2 = {( x, y , z ) ∈ M 3 , trong đó x, y, z khác màu và ( x + y + z ) ≡ 0 (mod n)} .
Giả sử rằng trong M có a số được tô màu đỏ và b số được tô màu xanh (với a + b = n) thì
| S1 |= a 2 − ab + b 2 , | S 2 |= 3ab .”

Thật vậy:
Ta chọn một số x được tô màu đỏ, một số y được tô màu xanh, x ≠ y . Giả sử z là một số
thuộc S mà n | ( x + y + z ) , rõ ràng z tồn tại và duy nhất (z có thể trùng với x hoặ y).
Không mất tính tổng quát, giả sử z được tô màu đỏ, cùng màu với x.

Xét hai trường hợp:


- Nếu z khác cả x và y. Khi đó cả ba số x, y, z là phân biệt. Khi đó, ta có tất cả 6 bộ ba
thuộc tập S2 chứa cả x và y là: ( x, y , z ); ( x, z , y ); ( y , x, z ); ( y , z , x); ( z , x, y ); ( z , y , x ) .
Nếu không tính đến thứ tự của x và y thì chỉ có 3 bộ trong các bộ trên thuộc S2. Thật vậy: khi xét
x đứng trước z trong các bộ trên, ta có 3 bộ ba là: ( x, y , z ); ( x, z , y );( y , x, z ) .
Tương tự, lại xét các bộ không tính đến thứ tự của z và y (chú ý rằng ở trêm, ta giả sử x
và x cùng màu), xét x đứng sau z trong các bộ này, ta cũng có 3 bộ ba nữa là:
( z , x, y ); ( z , y , x );( y , z , x ) . Do đó, trong trường hợp này, ta có tất cả 3 bộ thuộc S2.

- Nếu z bằng x hoặc z bằng y. Không mất tính tổng quát, giả sử z = x (trường hợp z = y
hoàn toàn tương tự, không quan tâm đến màu của chúng nữa). Khi đó, ta cũng có các bộ 3 thuộc
tập S2 chứa cả x và y là: ( x, x, y ); ( x, y , x );( y , x, x ) (chỉ xét các bộ không tính đến thứ tự của x, y).
Do đó, trong cả hai trường hợp, mỗi bộ không tính đến thứ tự (x, y) với x, y khác màu
nhau cho ta đúng 3 phần tử trong tập T2 và mỗi phần tử như vậy xuất hiện đúng một lần. Suy ra
giá trị của |S2| bằng 3 lần số bộ không tính đến thứ tự (x, y) đã nêu.
Mặt khác: có a số được tô màu đỏ, b số được tô màu xanh nên số bộ (x, y) nói trên chính là
ab, từ đó ta được: | S 2 |= 3ab . Với x, y cho trước thì số z thỏa mãn n | ( x + y + z ) là duy nhất.
Cả x và y được chọn đúng n lần nên | S1 ∪ S 2 |= n 2 = ( a + b) 2 , hơn nữa: S1 ∩ S 2 = ∅ nên
| S1 | + | S 2 |= (a + b) 2 ⇒| S1 |= ( a + b) 2 − 3ab = a 2 − ab + b 2 .
Bổ đề được chứng minh.

57
*Trở lại bài toán:
Giả sử số các số được tô màu xanh, vàng, đỏ lần lượt là a, b, c thì : a + b + c = 2008; a, b, c ∈ ℕ * .
- Xét tập hợp:
A = {( x, y , z ) ∈ M 3 | x, y, z được tô cùng màu xanh và x + y + z ≡ 0 (mod 2008)} .
Các tập B, C định nghĩa tương tự, ứng với các màu vàng và đỏ.
- Xét tập hợp:
AB = {( x, y , z ) ∈ M 3 | x, y, z được tô bởi hai màu xanh, vàng và x + y + z ≡ 0 (mod 2008)}
Các tập BC, CA được định nghĩa tương tự, ứng với các cặp màu vàng, đỏ và đỏ, xanh.
- Xét tập hợp :
ABC = {( x, y , z ) ∈ M 3 |x, y, z được tô bởi cả ba màu xanh, vàng, đỏ và x + y + z ≡ 0 (mod 2008)}
Tiếp theo, ta sẽ dùng bổ đề trên đánh giá số phần tử của các tập hợp trên:
Gọi c là màu đại diện cho hai màu xanh và vàng, khi đó: số bộ ba được tô cùng màu chính là:
A ∪ B ∪ C ∪ AB và số bộ ba tô khác màu chính là: ABC ∪ BC ∪ CA .
Ta có: | A | + | B | + | C | + | AB |= ( a + b) 2 − c( a + b) + c 2 ,| ABC | + | BC | + | CA |= 3c( a + b) .
Hoàn toàn tương tự, ta có:
| A | + | B | + | C | + | BC |= (b + c) 2 − a (b + c) + a 2 ,| ABC | + | CA | + | AB |= 3( a (b + c) .
| A | + | B | + | C | + | CA |= (c + a ) 2 − b(c + a ) + b 2 ,| ABC | + | AB | + | BC |= 3b(c + a ) .
Theo cách xác định như trên thì:
S1 = A ∪ B ∪ C ⇒| S1 |=| A | + | B | + | C | , S 2 = ABC ⇒| S 2 |=| ABC |
Cộng từng vế tương ứng của nhóm thứ nhất rồi nhân với hai, ta được:
6(| A | + | B | + | C |) + 2(| AB | + | BC | + | CA |) = 3( a 2 + b 2 + c 2 ) .
Cộng từng vế tương ứng của nhóm thứ hai, ta được:
3 | ABC | +2(| AB | + | BC | + | CA |) = 6( ab + bc + ca ) .
Suy ra: 6(| A | + | B | + | C |) − 3 | ABC |= 3 ( a − b) 2 + (b − c) 2 + (c − a ) 2  ≥ 0 .
Do đó: 2 | S1 | − | S 2 |≥ 0 .
Đẳng thức không xảy ra vì không tồn tại a = b = c nguyên dương và a + b + c = 2008 .
Vậy bất đẳng thức ở trên là thực sự, tức là 2 | S1 | − | S 2 |> 0 ⇔ 2 | S1 |>| S 2 | .
Đây chính là đpcm.

58
LỜI GIẢI ĐỀ THI CHỌN ĐỘI TUYỂN QUỐC GIA

DỰ THI IMO 2009

Bài 1. Cho tam giác nhọn ABC nội tiếp đường tròn (O). Gọi A1 , B1 , C1 và A2 , B2 , C2
lần lượt là các chân đường cao của tam giác ABC hạ từ các đỉnh A, B, C và các điểm đối
xứng với A1 , B1 , C1 qua trung điểm của các cạnh BC , CA, AB . Gọi A3 , B3 , C3 lần lượt là các
giao điểm của đường tròn ngoại tiếp các tam giác AB2C2 , BC2 A2 , CA2 B2 với (O).
Chứng minh rằng: A1 A3 , B1 B3 , C1C3 đồng quy.

A
Ta sẽ chứng minh các đường thẳng
A
3 A1 A3 , B1 B3 , C1C3 cùng đi qua trọng tâm của tam giác
ABC. Thật vậy:
C B
1 1 Gọi M là trung trực của BC, A’ là điểm đối xứng với A
B
2 qua trung trực của BC. Ta sẽ chứng minh rằng A’ trùng
C
2
với A3 hay đường tròn (AB2C2) cắt (O) tại A’.
O

A A C A A'
B
2 1
B
3
C
3
C B
2 1

Ta có: A, A’ đối xứng nhau qua trung trực của BC C


1
nên: AB = A’C , AC = A’B . Do A, B và C1, C2 G
B
cùng đối xứng với nhau qua trung điểm của AB nên 2
BC2 = AC1 . Tương tự: CB2 = AB1 . Suy ra:
B A M C
BC2 AC1 AC A ' B 1
= = = .
CB2 AB1 AB A ' C
 
Kết hợp với C3 BA ' = B3CA ' ( cùng chắn cung AA’)


ta được: ∆C2 BA ' ∼ ∆B2CA ' (c.g .c) ⇒ BC   
2 A ' = CB2 A ' ⇒ AC2 A ' = AB2 A ' .

Do đó, tứ giác AC2B2A’ là tứ giác nội tiếp hay A’ trùng với A3. Gọi G là giao điểm của
AG AA3
trung tuyến AM với A1A3. Do AA3 // A1M nên: = = 2 ⇒ G là trọng tâm của tam giác
GM A1M
ABC hay đường thẳng A1A3 đi qua trọng tâm G của tam giác ABC.
Tương tự: B1 B3 , C1C3 cũng đi qua G.
Vậy các đường thẳng A1 A3 , B1 B3 , C1C3 đồng quy. Ta có điều phải chứng minh.

59
Bài 2.
Cho đa thức P ( x ) = rx 3 + qx 2 + px + 1 trong đó p, q, r là các số thực và r > 0 .
Xét dãy số sau:
 a1 = 1, a2 = − p, a3 = p − q
2


 an +3 = − p.an + 2 − q.an +1 − r.an , n ≥ 0
Chứng minh rằng: nếu đa thức P ( x ) có một nghiệm thực duy nhất và không có
nghiệm bội thì dãy số ( an ) có vô số số âm.

* Giả sử k là một nghiệm (thực hoặc phức) của đa thức:


Q ( x) = x3 + px 2 + qx + r , do r > 0 nên k ≠ 0 ⇒ k 3 + pk 2 + qk + r = 0 (*)
Theo giả thiết, đa thức P ( x) = rx 3 + qx 2 + px + 1 có đúng một nghiệm thực nên nó còn có thêm
1
hai nghiệm phức liên hợp nữa, đồng thời chính là nghiệm của P(x) do:
k
3 2
1 1 1 1 r + qk + pk 2 + k 3
P( ) = r   + q   + p   + 1 = =0.
k k k k k3

Xét dãy số (un) xác định bởi công thức:


r
un +1 = an +3 + ( p + k ) an + 2 − an +1 (**)
k
Mặt khác, theo giả thiết: an +3 = − pan + 2 − qan +1 − ran , n = 0,1, 2,...
Ta có:
r kq + r
un +1 = − pan + 2 − qan +1 − ran + ( p + k )an + 2 − an +1 = kan + 2 − .an +1 − ran
k k
kq + r r
= k (an + 2 − 2 .an +1 − an )
k k
kq + r
Từ (*) ⇒ −( kq + r ) = pk 2 + k 3 ⇒ − 2 = p + k , do đó:
k
r
un +1 = k ( an + 2 + ( p + k ) an +1 − an ) = kun , n = 0,1, 2,...
k
Trong (**), cho n = -1, ta có:
r r pk 2 + qk + r k 3
u0 = a2 + ( p + k )a1 − a0 = p 2 − q − ( p + k ) p − = − = = k2
k k k k
r
Suy ra: un = k n + 2 ⇒ an + 2 + ( p + k ) an +1 − an = k n + 2 , n = 0,1, 2,... (***)
k

60
Giả sử z là nghiệm phức của phương trình P(x) = 0 và ρ , θ lần lượt là modun và argument
của z trong đó: ρ , θ ∈ ℝ, ρ > 0 .
Ta có: z = ρ eiθ = ρ (cos θ + i sin θ ) và P ( x ) ∈ ℝ[ x ] nên: P ( z ) = 0 ⇒ P ( z ) = 0 ⇒ P ( z ) = 0 do đó:
z = ρ e − iθ = ρ (cos θ − i sin θ ) cũng là nghiệm của P(x).
r r
Từ (***), ta được: an + 2 + ( p + z )an +1 − an = ( z ) n + 2 , an + 2 + ( p + z ) an +1 − an = ( z ) n + 2 .
z z
Theo công thức Moavre, ta có: z = ρ (cos θ + i sin θ ) ⇒ z = ρ (cos nθ + i sin nθ ) nên:
n n

()
n+2
z n+ 2 − z = ρ n + 2 [ cos(n + 2)θ + i sin(n + 2)θ ] − ρ n + 2 [ cos(n + 2)θ − i sin(n + 2)θ ] =

()
n+ 2
z n+2 − z 2i.ρ n + 2 sin [ (n + 2)θ ] sin [ (n + 2)θ ]
= 2i.ρ n + 2 sin [ (n + 2)θ ] ⇒ = = ρ n +1.
z−z 2i.ρ sin θ sin θ
1 1
Trừ từng vế hai đẳng thức: ( z − z ) an +1 − r ( − ) an = ( z ) n + 2 − ( z ) n + 2
z z
z−z r z n+2 − ( z)n+2
⇔ ( z − z )an +1 − r ( )an = z n + 2 − ( z ) n + 2 ⇔ an +1 + 2 an =
z.z ρ (z − z)
r sin [ (n + 2)θ ]
⇔ an +1 + an = ρ n +1.
ρ 2
sin θ

Do ρ > 0 nên xét n0 là một giá trị nguyên dương sao cho:
sin [ (n0 + 2)θ ] sin [ (n0 + 2)θ ] r
< 0 ⇒ ρ n0 +1. < 0 ⇒ an0 +1 + an0 < 0 .
sin θ sin θ ρ2
r
Vì > 0 nên an0 +1 , an0 trái dấu với nhau. Do đó, trong hai giá trị này có một số âm.
ρ2
sin [ (n0 + 2)θ ]
Ta thấy khi n tiến tới vô cực, tồn tại vô số giá trị n0 sao cho < 0 , mà ứng với
sin θ
mỗi giá trị n0 như thế ta lại tìm được một số hạng âm của dãy đã cho, tức là dãy (an) có vô số số
âm. Đây chính là điều phải chứng minh.

61
Bài 3.
Cho các số nguyên dương a, b sao cho a, b và a.b đều không là số chính phương.
Chứng minh rằng trong hai phương trình sau:
ax 2 − by 2 = 1
ax 2 − by 2 = −1
có ít nhất một phương trình không có nghiệm nguyên dương.

* Trước hết ta sẽ chứng minh bổ đề sau:


Cho A, B là các số nguyên dương và A, B, AB đều không là các số chính phương.
Khi đó: nếu gọi (a, b) là nghiệm nguyên dương nhỏ nhất của phương trình Pell x 2 − ABy 2 = 1
(do AB không là số chính phương nên phương trình Pell này luôn có nghiệm nguyên dương,
nghĩa là (a, b) tồn tại) và (x0, y0) là nghiệm nguyên dương nhỏ nhất của phương trình
a = Ax02 + By02
Ax 2 − By 2 = 1 thì ta luôn có hệ thức liên hệ sau: 
b = 2 x0 y0
*Chứng minh:
Do (x0,y0) là nghiệm của Ax 2 − By 2 = 1 nên Ax02 − By02 = 1 .
Đặt u = Ax02 + By02 , v = 2 x0 y0 ; khi đó, ta có:

u 2 − ABv 2 = ( Ax02 + By02 ) − AB. ( 2 x0 y0 ) = ( Ax02 − By02 ) = 1 .


2 2 2

Do đó, (u; v) là một nghiệm của x 2 − ABy 2 = 1 .


Mà (a, b) là nghiệm nguyên dương nhỏ nhất của phương trình Pell x 2 − ABy 2 = 1 nên u ≥ a, v ≥ b .
Ta sẽ chứng minh rằng u = a, v = b.
Thật vậy, giả sử ngược lại, u > a, v > b. Ta có:
( )( )
a − b AB < a − b AB a + b AB = a 2 − ABb 2 = 1

(
⇒ a − b AB )( )
Ax0 + B y0 < Ax0 + B y0

⇔ ( ax0 − Bby0 ) A + (ay0 − Abx0 ) B < Ax0 + B y0

( )
2
Mặt khác: a + b AB < u + v AB = Ax02 + By02 + 2 x0 y0 AB = Ax0 + B y0
Do đó:
( ax0 − Bby0 ) (
A − ( ay0 − Abx0 ) B = a + b AB )( x 0 )
A − y0 B <

(x )( x ) = ( Ax − By ) ( x ) ( )
2
0 A − y0 B 0 A + y0 B 2
0
2
0 0 A + y0 B = x0 A + y0 B

 s A + t B < x0 A + y0 B (1)
Đặt s = ax0 − Bby0 , t = ay0 − Abx0 , ta được: 
 s A − t B < x0 A + y0 B (2)
62
2 2
(
Hơn nữa: As 2 − Bt 2 = A ( ax0 − Bby0 ) − B ( ay0 − Abx0 ) = a 2 − ABb 2 )( Ax
2
0 − By02 ) = 1.1 = 1
Ta thấy s > 0 vì: s > 0 ⇔ ax0 − Bby0 > 0 ⇔ ax0 > Bby0 ⇔ a 2 x02 > B 2b 2 y02 ⇔ a 2 x02 > Bb 2 ( Ax02 − 1)
⇔ ( a 2 − ABb 2 ) x02 > − Bb 2 ⇔ x02 > − Bb 2 , đúng do B > 0.
Ta cũng thấy t ≠ 0 bởi vì nếu t = 0 thì:
ay0 − Abx0 = 0 ⇔ ay0 = Abx0 ⇔ a 2 y02 = A2b 2 x02 ⇔ y02 ( ABb 2 + 1) = Ab 2 ( By02 + 1) ⇔ y02 = Ab 2
Điều này mâu thuẫn do A không là số chính phương.
-Nếu t > 0 thì (s; t) là một nghiệm dương của phương trình Ax 2 − By 2 = 1 , từ đó:
s ≥ a, t ≥ b , suy ra: s A + t B > x0 A + y0 B , điều này mâu thuẫn với (1).
-Tương tự, nếu t < 0 thì (s; -t) là một nghiệm dương của phương trình Ax 2 − By 2 = 1 , từ đó:
s ≥ a, −t ≥ b , suy ra: s A − t B > x0 A + y0 B , điều này mâu thuẫn với (2).
Do đó, điều giả sử là sai, nghĩa là u = a, v = b. Bổ đề được chứng minh.

* Trở lại bài toán:


Giả sử ngược lại, cả hai phương trình:
ax 2 − by 2 = 1 (*)
ax − by = −1
2 2
(**)
đều có nghiệm nguyên dương. Gọi (m, n) là nghiệm nguyên dương nhỏ nhất của phương trình
x 2 − aby 2 = 1 ; (x1; y1) là nghiệm nguyên dương nhỏ nhất của (*); (x2; y2) là nghiệm nguyên
dương nhỏ nhất của (**). Theo bổ đề trên, ta có các hệ thức sau:
m = ax12 + by12 m = bx22 + ay22
 và 
n = 2 x1 y 1 n = 2 x2 y2
Từ (*) ⇒ ax12 = by12 + 1 , từ (**) ⇒ ay22 = bx22 − 1 , so sánh các đẳng thức ở trên, ta cũng có:
ax12 + by12 = bx22 + ay22 ⇒ 2by12 + 1 = 2bx22 − 1 ⇔ b ( y12 − x22 ) = 1 .
Nhưng do b là số nguyên dương, không phải là số chính phương nên b > 1, nghĩa là đẳng thức
trên không thể xảy ra. Suy ra điều giả sử ở trên là sai.
Vậy trong hai phương trình (*) và (**) đã cho có ít nhất một phương trình không có nghiệm
nguyên dương. Đây chính là điều phải chứng minh.

63
Bài 4.
Tìm tất cả các số thực r sao cho bất đẳng thức sau đúng với mọi số a, b, c dương:
3
 a  b  c   1
r + r +  r +  ≥ r + 
 b + c  c + a  a+b  2

Ta sẽ xét điều kiện cần và đủ để tìm các giá trị r thỏa đề bài.
c
* Điều kiện cần: Xét trường hợp a = b > 0 . Đặt t = > 0 . Ta có:
a
3
 a  b  c   1
r + r +  r +  ≥ r + 
 b + c  c + a  a+b  2
2
 
 a  
2
c   1  3
1   c   1
3

⇔ r +  . r +  ≥  r +  ⇔  r + . r +  ≥ r + 
 a+c  2a   2 c  2a   2
 1+ 
 a
 1  
2
t  1
3
 2 2r  1  2   t  1
3

⇔ r + . r +  ≥  r +  ⇔ r + +   . r +  ≥  r + 
 1+ t   2  2  t + 1  t + 1    2  2
 2 t 3  1 2t 3   t 1
⇔ + −  .r 2 +  + −  .r +  − ≥0
 t +1 2 2   (t + 1) t + 1 4   t +1 8 
2

 5 −1
r ≥
Cho t → 0 , từ bất đẳng thức trên, suy ra: 4r 2 + 2r − 1 ≥ 0 ⇔  4 (*)
 − 5 −1
r ≤
 4
* Điều kiện đủ:
Ta sẽ chứng minh rằng với giá trị r thỏa mãn (*) thì bất đẳng thức đã cho đúng với mọi số dương
a b c
a, b, c. Thật vậy: Đặt: x = ,y= ,z = , x, y , z > 0 . Ta có:
b+c c+a a+b
a b b c c a a b c
xy + yz + zx + 2 xyz = . + . + . + . . =
b+c c+a c+a a+b a+b b+c b+c c+a a+b
ab( a + b) + bc(b + c) + ca (c + a ) + 2abc ( a + b)(b + c)(c + a )
= =1
( a + b)(b + c)(c + a ) ( a + b)(b + c)(c + a )
Ta sẽ chứng minh các bất đẳng thức sau:
(1) x + y + z ≥ 2( xy + yz + zx)
a b c  ab bc ca 
⇔ + + ≥ 2 + + 
b+c c+a a+b  (a + c)(b + c) (b + a)(c + a ) (c + b)(a + b) 
a(a + b)(a + c) + b(b + c)(b + a ) + c(c + a)(c + b) 2ab(a + b) + 2bc(b + c) + 2ca(c + a)
⇔ ≥
(a + b)(b + c)(c + a) (a + b)(b + c)(c + a)

64
⇔ a 3 + b 3 + c3 + ab(a + b) + bc(b + c) + ca (c + a ) + 3abc ≥ 2ab( a + b) + 2bc(b + c) + 2ca (c + a )
⇔ a 3 + b 3 + c3 + 3abc ≥ ab(a + b) + bc(b + c) + ca (c + a )
⇔ a (a 2 + bc − ab − ac) + b(b 2 + ca − ba − bc) + c(c 2 + ab − ca − cb) ≥ 0
⇔ a (a − b)(a − c) + b(b − a )(b − c) + c(c − a )(c − b) ≥ 0
Bất đẳng thức này chính là bất đẳng thức Schur với các số dương a, b, c.
3
(2) xy + yz + zx ≥
4
ab bc ca 3
⇔ + + ≥
(a + c)(b + c) (b + a )(c + a ) (c + b)( a + b) 4
3
⇔ ab(a + b) + bc(b + c) + ca (c + a ) ≥ ( a + b)(b + c)(c + a )
4
⇔ 4 [ ab(a + b) + bc(b + c) + ca (c + a )] ≥ 3 [ ab( a + b) + bc(b + c) + ca (c + a )] + 6abc
⇔ ab(a + b) + bc(b + c) + ca (c + a ) ≥ 6abc
⇔ a (b − c) 2 + b(c − a ) 2 + c(a − b)2 ≥ 0
Bất đẳng thức cuối đúng nên (2) đúng với mọi số dương a, b, c.
3
Đặt t = xy + yz + zx ⇒ t ≥ . Bất đẳng thức đã cho chính là:
4
1
(r + x)(r + y )(r + z ) ≥ (r + )3
2
3 3 1
⇔ r 3 + ( x + y + z )r 2 + ( xy + yz + zx)r + xyz ≥ r 3 + r 2 + r +
2 4 8
3 2 3 1
⇔ ( x + y + z − )r + ( xy + yz + zx − )r + xyz − ≥ 0
2 4 8
1
Do x + y + z ≥ 2( xy + yz + zx ) và xy + yz + zx + 2 xyz = 1 ⇔ xyz = (1 − xy − yz − zx ) nên để chứng
2
minh bất đẳng thức trên, ta chứng minh bất đẳng thức sau:
 3 3 1 1
⇔  2( xy + yz + zx) −  r 2 + ( xy + yz + zx − )r + (1 − xy − yz − zx) − ≥ 0
 2 4 2 8
3 3 1 1
⇔ (2t − )r 2 + (t − )r + (1 − t ) − ≥ 0
2 4 2 8
1 3 3 3
⇔ t (2r 2 + r − ) ≥ r 2 + r −
2 2 4 8
4r + 2r − 1 3(4r + 2r − 1)
2 2
 3
⇔ t. ≥ ⇔ (4r 2 + 2r − 1)  t −  ≥ 0
2 8  4
Bất đẳng thức cuối đúng do (*) và (2) nên bất đẳng thức đã cho là đúng.
5 −1 − 5 −1
Vậy điều kiện của r cần tìm là r ≥ hoặc r ≤ .
4 4
65
Bài 5.
Cho đường tròn (O) có đường kính AB và M là một điểm bất kì nằm trong (O), M
không nằm trên AB. Gọi N là giao điểm của phân giác trong góc M của tam giác AMB với
đường tròn (O). Đường phân giác ngoài góc AMB cắt các đường thẳng NA, NB lần lượt tại
P, Q. Đường thẳng MA cắt đường tròn đường kính NQ tại R, đường thẳng MB cắt đường
tròn đường kính NP tại S và R, S khác M.
Chứng minh rằng: đường trung tuyến ứng với đỉnh N của tam giác NRS luôn đi qua
một điểm cố định khi M di động phía trong đường tròn.

R
I
D
C Q
S

P M
A
O B

* Qua R kẻ đường thẳng song song với PQ cắt NA tại C, qua S kẻ đường thẳng song song với
PQ cắt NB tại D. Gọi I là trung điểm của CD .
Ta sẽ chứng minh rằng CD // AB.
Thật vậy, do N nằm trên đường tròn đường kính AB nên: ANB = 900 ⇒ AN ⊥ BN , suy
ra BN là tiếp tuyến của đường tròn đường kính PN.
Do đó: ∆BMN ∼ ∆BNS ( g .g )
=
Vì PQ là đường phân giác góc ngoài của AMN nên: SMP  = BMQ
AMP = QMR .
 = SNP
Mặt khác: SMP  (góc nội tiếp cùng chắn cung PS của đường tròn đường kính PN),
 = QNR
QMR  ( góc nội tiếp cùng chắn cung QR của đường tròn đường kính QN).
 = QNR
Do đó: SNP  ⇒ SNP
 + SNR
 = QNR
 + SNR
 ⇒ CNR
 = SNB
.
 = SNB
Xét hai tam giác ∆ BNS và ∆ RNC có: CNR  và RCN
 = MPN
 = NSM
 = NSB

nên: ∆BNS ∼ ∆RNC ( g .g ) .
Suy ra các tam giác đồng dạng: ∆BMN ∼ ∆BNS ∼ ∆RNC .
66
Tương tự, ta cũng có: ∆DSN ∼ ∆RAN ∼ ∆NAM .
NB NS
* Ta thấy, từ: ∆BNS ∼ ∆RNC ⇒ = ⇒ NB.NC = NR.NS
NR NC
NS ND
∆DSN ∼ ∆RAN ⇒ = ⇒ NA.ND = NR.NS .
NA NR
NA NC
Suy ra: NA.ND = NB.NC ⇒ = ⇒ AB // CD
NB ND
⇒ Trung điểm của AB, trung điểm của CD và N là ba điểm thẳng hàng.
Tức là N, O, I thẳng hàng. (1)
MN BN NB.NC
* Hơn nữa: ∆BMN ∼ ∆RNC ⇒ = ⇒ RC = .
NC RC MN
DN DS NA.ND
∆DSN ∼ ∆NAM ⇒ = ⇒ DS = .
MN NA MN
Kết hợp các điều trên, ta được: RC = DS, mà RC // DS (cùng song song với PQ) nên tứ giác
RCSD là hình bình hành.
Do đó, hai đường chéo CD và RS của tứ giác cắt nhau tại trung điểm của mỗi đường.
Suy ra I là trung điểm của CD cũng là trung điểm của RS.
Khi đó: NI chính là đường trung tuyến của tam giác NRS. (2)
Từ (1) và (2), suy ra: trung tuyến NI của tam giác NRS luôn đi qua O.
Vậy trung tuyến ứng với đỉnh N của tam giác NRS luôn đi qua I là điểm cố định khi M di động
khắp phía trong đường tròn (O).
Đây chính là điều phải chứng minh.

67
Bài 6.
Một hội nghị toán học có tất cả 6n + 4 nhà toán học phải họp với nhau đúng 2n + 1
lần ( n ≥ 1) . Mỗi lần họp, họ ngồi quanh một cái bàn 4 chỗ và n cái bàn 6 chỗ, các vị trí ngồi
chia đều khắp mỗi cái bàn. Biết rằng hai nhà toán học đã ngồi cạnh hoặc đối diện nhau ở
một cuộc họp này thì sẽ không được ngồi cạnh hoặc đối diện nhau ở một cuộc họp khác.
a/ Chứng minh rằng Ban tổ chức có thể xếp được chỗ ngồi nếu n = 1 .
b/ Hỏi rằng Ban tổ chức có thể sắp xếp được chỗ ngồi được hay không với mọi n > 1 ?

a. Với n = 1 , ta có bài toán như sau: một hội nghị toán học có 10 nhà toán học, họ phải họp
với nhau đúng 3 lần và trong mỗi lần họp như thế, họ phải ngồi quanh một cái bàn 4 chỗ và một
cái bàn 6 chỗ, các vị trí ngồi chia đều khắp bàn; đồng thời, hai người đã ngồi kề nhau hoặc đối
diện nhau trong cuộc họp này thì không được ngồi kề nhau hoặc đối diện nhau trong một cuộc
họp khác.
Đánh số thứ tự cho 10 nhà toán học đang xét là (1), (2), (3), …(10).
Ta sẽ chỉ ra một cách sắp xếp thỏa mãn đề bài trong trường hợp này. Ta có các sơ đồ sau:
-Buổi họp thứ 1:
5

1 6 10

2 4

7 9
3
8

-Buổi họp thứ 2:


2

1 6 10

9
5

3 4
7
8

-Buổi họp thứ 3:


5

1 2 4

10
6

7 9
8
3

68
b. Ta sẽ chứng minh rằng trong trường hợp tổng quát, Ban tổ chức luôn có thể xếp chỗ ngồi
cho các nhà toán học trong các cuộc họp.
Ta chia 6n + 4 nhà toán học thành 2n + 2 “nhóm”. Một “nhóm” chỉ gồm 1 người luôn
ngồi ở một vị trí cố định tại bàn 4 chỗ, đặt người này là X0; 2n + 1 “nhóm” còn lại chia ra từ
6n + 3 nhà toán học, mỗi “nhóm” có 3 nhà toán học.
Đặt các “nhóm” đó là X 1 , X 2 , X 3 ,..., X 2 n +1 . Các “nhóm” này sẽ lần lượt ngồi vào các vị trí còn lại
của bàn 4 chỗ cùng với X0, mỗi “nhóm” ngồi đúng một lần.
* Với các bàn 6 chỗ, ta có cách sắp xếp như sau:
Ở bước thứ k , 1 ≤ k ≤ 2n + 1 , với hai “nhóm” bất kì X i , X j trong đó: i + j ≡ k (mod 2n + 1),
1 ≤ i, j ≤ 2n + 1, i ≠ j thì các nhà toán học thuộc hai “nhóm” Xi, Xj sẽ ngồi vào họp cùng nhau ở
một bàn 6 chỗ nào đó; đồng thời, những nhà toán học thuộc cùng một “nhóm” thì ngồi ở các vị
trí tạo thành một tam giác đều trên các bàn 6 chỗ, nghĩa là họ sẽ không rơi vào trường hợp ngồi
đối diện nhau hoặc ngồi cạnh nhau.

X
j

X
i

* Với bàn 4 chỗ, ta có cách sắp xếp như sau:


k
- Nếu k là số chẵn thì ở bước này, chỉ có một “nhóm” có chỉ số là không được ngồi
2
chung bàn 6 chỗ với nhóm nào, như thế “nhóm” này sẽ ngồi vào bàn 4 chỗ cùng với X0.
k + 2n + 1
- Nếu k là số lẻ, ở bước này; tương tự trên, chỉ có một “nhóm” có chỉ số là
2
không được chung bàn 6 chỗ với “nhóm” nào, “nhóm” này sẽ ngồi vào bàn 4 chỗ cùng với X0.
Dễ dàng thấy rằng cách sắp xếp như thế thỏa mãn mọi yêu cầu của bài toán.
Vậy Ban tổ chức có thể sắp xếp được chỗ ngồi mọi n > 1 .

69
LỜI GIẢI ĐỀ THI CHỌN ĐỘI TUYỂN QUỐC GIA

DỰ THI IMO 2010


Bài 1.
Cho tam giác ABC không vuông tại A có đường trung tuyến AM. Gọi D là một điểm di
động trên đường thẳng AM. Gọi ( O1 ) , ( O2 ) là các đường tròn đi qua D, tiếp xúc với BC lần
lượt tại B và C. Gọi P, Q lần lượt là giao điểm của đường thẳng AB với đường tròn (O1),
đường thẳng AC với đường tròn (O2). Chứng minh rằng:
1. Tiếp tuyến tại P của (O1 ) và tiếp tuyến tại Q của (O2 ) phải cắt nhau tại một điểm.
Gọi giao điểm đó là S.
2. Điểm S luôn di chuyển trên một đường thẳng cố định khi D di động trên AM.

A
Q O2

P
S
D

x
O1

B M C
1. Vì M là trung điểm của BC nên MA2 = MB 2 , suy ra M có cùng phương tích đến hai
đường tròn ( O1 ) , ( O2 ) hay M thuộc trục đẳng phương của .
Hơn nữa, hai đường tròn cắt nhau tại D nên D cũng nằm trên trục đẳng phương của chúng.
Từ đó, suy ra DM chính là trục đẳng phương của ( O1 ) , ( O2 ) , mà A thuộc đường thẳng
DM nên A có cùng phương tích đến hai đường tròn ( O1 ) , ( O2 ) .
Suy ra: AP. AB = AQ. AC hay tứ giác BPQC nội tiếp.
Từ hệ thức trên ta cũng thấy rằng nếu P trùng với A thì Q cũng trùng với A, nếu P thuộc đoạn
AB thì Q cũng thuộc đoạn AC và ngược lại.

70
Không mất tính tổng quát, giả sử góc 
ABC nhọn, gọi Px là tia tiếp tuyến của đường tròn
 nhọn.
(O1) sao cho góc xPB
 = PBC
Theo tính chất tiếp tuyến, ta có: xPB =
 , mà PBC AQP nên xPB= AQB .
⇒ Px cũng là tiếp tuyến của đường tròn (APQ).
Do đó: (O1) tiếp xúc với đường tròn (APQ).
Hoàn toàn tương tự: ta cũng có (O2) cũng tiếp xúc với đường tròn (APQ).
Suy ra: tiếp tuyến tại P của (O1 ) và tiếp tuyến tại Q của (O2 ) cũng chính là hai tiếp tuyến của
(APQ) tại các điểm P, Q.
 ≠ 900 nên hai tiếp tuyến đó không song song và do đó
Hơn nữa, theo giả thiết: PAQ
chúng phải cắt nhau (đpcm).
2. Theo chứng minh ở trên, ta có: S thuộc tiếp tuyến của (O1) và (O2), SP = SQ nên S có
cùng phương tích đến hai đường tròn ( O1 ) , ( O2 ) nên S thuộc trục đẳng phương của hai đường
tròn này, nghĩa là S nằm trên AM.
Vậy khi D thay đổi trên AM thì S cũng di chuyển trên AM là đường thẳng cố định.
Ta có đpcm.

71
Bài 2.
Với mỗi số n nguyên dương, xét tập hợp sau :
Tn = {11( k + h) + 10( n k + n h ) |1 ≤ k , h ≤ 10} .
Tìm tất cả giá trị của n sao cho không tồn tại a, b ∈ Tn ; a ≠ b sao cho ( a − b ) chia hết cho 110.
Đặt f (k , h, n) = 11(k + h) + 10(n k + n h ), k , h, n ∈ ℕ .

Ta có: f (k , h, n) = f (h, k , n) nên không mất tính tổng quát, ta giả sử h ≥ k .

Nếu m ≡ n (mod11) thì :

f (k , h, m) − f (k , h, n) = 11(k + h) + 10(m k + m h )  − 11(k + h) + 10(n k + n h )  =


= 10 (m k − n k ) + (m h − n h )  ⋮110 ⇒ f (k , h, m) ≡ f (k , h, n) (mod110)

Từ đó, ta chỉ cần xét các giá trị n thỏa 1 ≤ n ≤ 11 .

Xét hiệu: f (6, 6, n) − f (1,1, n) = 110 + 20n.(n5 − 1) . Nếu 20n.(n5 − 1) chia hết cho 110 thì
giá trị n tương ứng sẽ không thỏa. Từ đó, ta loại đi các giá trị n = 1, 3, 4,5,9,11 .

Ta cũng có f (8, 2, n) − f (6, 4, n) = 10(n8 + n 2 − n 6 − n 4 ) = 10n 2 (n 2 − 1)2 (n 2 + 1) , với n = 10


thì 10n 2 (n 2 − 1) 2 (n 2 + 1)⋮110 nên giá trị này cũng không thỏa.

Ta sẽ chứng minh rằng các giá trị n = 2, 6, 7,8 thỏa mãn. Thật vậy:

Tính toán trực tiếp, ta thấy rằng với n = 2, 6, 7,8 thì n k ≠ n h (mod11) với k ≠ h (mod11) .

Giả sử ngược lại, với các giá trị n nêu trên, tồn tại hai bộ (k , h) ≠ (k ', h ') (giả sử k > k ' ) sao cho:
f (k , h, n) ≠ f (k ', h ', n) . Khi đó: 11(k + h − k '− h ') + 10(n k + n h − n k ' − n h ' ) ≡ 0 (mod110) .

Suy ra:
k − k ' ≡ h − h ' (mod10), n k + n h ≡ n k ' + n h ' (mod11) ⇔ n k ' (n k − k ' − 1) ≡ n h (n h ' − h − 1) (mod11) .

Do 11 là số nguyên tố nên theo định lí Fermat nhỏ thì: n k − k ' ≡ n h '− h (mod11) .

Dễ thấy: n k − k ' − 1 ≡ n h '− h − 1 ≠ 0 (mod11) nên từ đẳng thức trên, suy ra: n k ' ≡ n h (mod11) ⇒ k ' = h .

Do đó: k = h ' hay (k , h) ≡ (k ', h ') , mâu thuẫn.

Suy ra các giá trị n = 2, 6, 7,8 đều thỏa mãn yêu cầu của đề bài.

Vậy tất cả các giá trị n cần tìm là: n ≡ 2, 6, 7,8 (mod11) .

72
Bài 3.
Gọi một hình chữ nhật có kích thước 1× 2 là hình chữ nhật đơn và một hình chữ nhật
có kích thước 2 × 3 , bỏ đi 2 ô ở góc chéo nhau (tức có có 4 ô vuông con) là hình chữ nhật kép.
Người ta ghép khít các hình chữ nhật đơn và hình chữ nhật kép này lại với nhau được một
bảng hình chữ nhật có kích thước là 2008 × 2010 .
Tìm số nhỏ nhất các hình chữ nhật đơn có thể dùng để ghép.

Ta sẽ chứng minh rằng số hình chữ nhật đơn nhỏ nhất thỏa mãn đề bài là 1006. Thật vậy:
*Điều kiện cần:
Ta xét một cách phủ hình chữ nhật 2008 × 2010 thỏa mãn đề bài (chú ý rằng 2008 chỉ số
hàng và 2010 chỉ số cột). Gọi x, y, z , t là số các hình chữ nhật 1× 2, 2 ×1, 2 × 3, 3 × 2 trong cách
phủ đó (ở đây thực ra các hình chữ nhật 1× 2, 2 × 1 đều là các hình chữ nhật đơn của đề bài, chỉ
phân biệt ở cách phủ dọc hay ngang, trong đó 1× 2 được đặt ngang, 2 × 1 được đặt dọc; tương tự
với cách phân biệt 2 × 3, 3 × 2 ).
Tô màu trắng cho các hàng lẻ, tô màu đen cho các hàng chẵn. Ở tất cả các ô của hàng thứ
i,1 ≤ i ≤ 2010 , ta đánh số tương ứng các số tự nhiên i.
Ta sẽ chứng minh các nhận xét sau:
- Nhận xét 1: ta luôn có đẳng thức: 2( x + y ) + 4( z + t ) = 2008.2010 .
Mỗi hình chữ nhật 1× 2 hoặc 2 × 1 có chứa hai ô vuông, mỗi hình chữ nhật 2 × 3 hoặc
3 × 2 có chứa bốn ô vuông. Tổng các ô vuông này bằng số ô vuông của cả hình chữ nhật lớn nên
2( x + y ) + 4( z + t ) = 2008.2010 .
- Nhận xét 2: Giá trị x, y là chẵn.
Ta thấy trên toàn bảng, các hình chữ nhật 2 × 3, 3 × 2 đều có số ô trắng bằng số ô đen; các
hình chữ nhật 2 × 1 được đặt dọc nên cũng có số ô trắng bằng số ô đen. Suy ra, số hình chữ nhật
1× 2 ở các hàng được tô màu trắng bằng số hình chữ nhật ở các hàng được tô đen. Hơn nữa, tổng
số các hàng là 2010 là chẵn nên giá trị x phải là chẵn. Từ nhận xét 1, ta thấy y cũng phải chẵn.
Trở lại bài toán, ta xét tương ứng Φ đi từ tập hợp các hình chữ nhật đang xét đến các số
nguyên là hiệu giữa tổng các số ở ô vuông được tô đen với tổng các số ở ô vuông được tô trắng
ghi trên nó.
Dễ dàng thấy rằng: Φ (3 × 2) = 0; Φ (2 × 3) = ±2; Φ (2 × 1) = ±1 .
Từ đó suy ra: ∑ Φ(3 × 2) = 0; ∑ Φ(2 × 3) ≤ 2 z; ∑ Φ(2 ×1) ≤ y .
(kí hiệu ∑ Φ(3 × 2) là tổng tính trên tất cả các hình chữ nhật 3 × 2 được dùng, định nghĩa tương
tự với các hình chữ nhật khác).
Ta cũng thấy rằng, tổng các số ghi trên x hình chữ nhật 1× 2 là một số chẵn thuộc [ 2; 2.2008] ,
x
mà x là số chẵn (nhận xét 2), ta có đánh giá sau: ∑ Φ(1× 2) ≤ 2 ( 2.2008 − 2 ) .

73
1004 1004
Ta có: Φ(2008 × 2010) = ∑ 2010.[ 2i − (2i − 1) ] = ∑ 2010.i = 2010.1004 .
i =1 i =1

Mặt khác: Φ(2008 × 2010) = ∑ Φ (2 × 1) + ∑ Φ (1× 2) + ∑ Φ(3 × 2) + ∑ Φ(2 × 3) .


x
Từ các điều trên, suy ra: 2010.1004 ≤ .(2.2008 − 2) + y + 2 z ⇔ 2010.1004 ≤ 2007 x + y + 2 z .
2
Tiếp theo, ta xét hình chữ nhật 2010 × 2008 (tương tự như trên nhưng có 2010 hàng và
2008 cột), bắt đầu lại các lập luận về số các hình chữ nhật 1× 2, 2 ×1, 2 × 3, 3 × 2 được dùng.
Ta xây dựng được bất đẳng thức sau: 2008.1005 ≤ 2009 y + x + 2t .
Cộng hai bất đẳng thức này lại, ta có:
2008.1005 + 2010.1004 ≤ (2009 y + x + 2t ) + (2007 x + y + 2 z ) = 2008 x + 2010 y + 2( z + t ) .
Hơn nữa, theo nhận xét 1 thì: 2010.1004 = ( x + y ) + 2( z + t ) .
Từ đó ta được: 2008.1005 ≤ 2007 x + 2009 y ≤ 2009( x + y ) .
2008
Suy ra: x + y ≥ 1005. > 1004 , mà x + y là số chẵn nên x + y ≥ 1006 .
2009
Do đó, tổng các hình chữ nhật đơn cần dùng ít nhất là 1006. Điều kiện cần được chứng minh.

*Điều kiện đủ:


Ta sẽ chỉ ra một cách ghép hình chữ nhật dùng đúng 1006 hình chữ nhật đơn.

Khối 1 Khối 2 Khối 3 Khối 4

74
Hình trên mô tả cách ghép một hình chữ nhật 10 × 16 , trong đó: các hình chữ nhật khuyết được tô
bằng 5 màu khác nhau (đỏ, hồng, xanh lam, xanh lá cây, xanh đậm) để dễ dàng phân biệt; trên
hình các khối được tô màu xanh lá mạ là các hình chữ nhật đơn chắc chắn phải dùng, các khối
màu vàng thì tùy trường hợp, có thể là hình chữ nhật đơn mà cũng có thể là hình chữ nhật khuyết.

* Hình chữ nhật 2010 × 2008 có thể được tạo thành từ hình trên bằng quy tắc sau:
- Thêm các dòng bằng cách chèn vào giữa mỗi khối ở trên các hình có dạng:

Mỗi lần ghép như thế thì ta có thêm được hai hàng mới, do 2010 chia hết cho 2 nên khi thực hiện
việc này liên tiếp một cách thích hợp thì khối này sẽ tăng về chiều dài, tạo thành các khối mới có
kích thước 2010 × 4 và ở mỗi khối như vậy, ta chỉ dùng đúng 2 hình chữ nhật màu xanh lá mạ.
- Thêm cột bằng cách lặp lại các khối 1, 2 , 3, 4 ở trên hình (chú ý tính tuần hoàn giữa các
khối: (1) tương ứng với (3), (2) tương ứng với (4)).
Như thế thì ta cần phải có tất cả 502 khối dành cho 2008 cột. Đồng thời, ở khối đầu tiên và khối
cuối cùng, ta cần dùng thêm một hình chữ nhật đơn màu vàng, các khối ở giữa thì dùng các hình
chữ nhật khuyết màu vàng. Tức là: ở hai khối đầu tiên và cuối cùng, ta cần dùng 3 hình chữ nhật
đơn, các khối ở giữa chỉ cần dùng 2 hình chữ nhật đơn thôi.
Khi đó, tổng số hình chữ nhật đơn cần dùng là: 500.2 + 2.3 = 1006 .
Xoay hình chữ nhật 2010 × 2008 lại, ta được hình chữ nhật 2008 × 2010 cần phải ghép, hình chữ
nhật đó có đúng 1006 hình chữ nhật đơn thỏa mãn đề bài.
Do đó, điều kiện đủ được chứng minh.

Vậy giá trị nhỏ nhất các hình chữ nhật đơn cần dùng là 1006.
Bài toán được giải quyết hoàn toàn.

75
Bài 4.
1 1 1
Cho a, b, c là các số thực dương thỏa mãn điều kiện: 16( a + b + c) ≥ + + .
a b c
Chứng minh rằng:
1 1 1 8
+ + ≤ .
(a + b + 2(a + c)) (b + c + 2(b + a )) (c + a + 2(c + b))
3 3 3
9
Hỏi đẳng thức xảy ra khi nào?

Theo bất đẳng thức Cauchy, ta có:


2
a+c a+c  a+c  (a + b)(a + c)
a + b + 2(a + c) = (a + b) + + ≥ 3 3 (a + b).   = 3 3
2 2  2  2
1 2
⇒ ≤ .
( ) 27(a + b)(a + c)
3
a + b + 2(a + c)
Tương tự với hai biểu thức còn lại. Do đó:
1 2 4(a + b + c)
∑ ≤∑ = .
( ) sym 27( a + b )( a + c ) 27(a + b)(b + c)(c + a )
3
cyc a + b + 2( a + c )

Hơn nữa, ta thấy với mọi a, b, c dương:


9(a + b)(b + c)(c + a ) − 8(a + b + c)(ab + bc + ca ) = ∑ a (b − c)2 ≥ 0
sym

8
⇒ (a + b)(b + c)(c + a ) ≥ (a + b + c)(ab + bc + ca )
9
1 1
Do đó: ∑ ≤ . (1).
( ) 6(ab + bc + ca )
3
cyc a + b + 2( a + c )

Mặt khác, ta cũng có: (ab + ca + ca ) 2 ≥ 3abc(a + b + c) nên theo giả thiết:
1 1 1 ab + bc + ca 3(a + b + c) 3
16( a + b + c) ≥ + + = ≥ ⇒ ab + bc + ca ≥ . (2)
a b c abc ab + bc + ca 16
1 1 1 8
Từ (1) và (2), suy ra: + + ≤ .
(a + b + 2(a + c)) (b + c + 2(b + a )) (c + a + 2(c + b))
3 3 3
9
Ta có đpcm.
Đẳng thức xảy ra khi dấu bằng ở tất cả các bất đẳng thức trên xảy ra hay:

 a , b, c > 0
 1
a = b = c ⇒a=b=c= .
 4
1 1 1
16(a + b + c) = + +
 a b c

76
Bài 5.
Trong một hội nghị có n nước tham gia, mỗi nước có k đại diện ( n > k > 1 ). Người ta
chia n.k người này thành n nhóm, mỗi nhóm có k người sao cho không có hai người nào
cùng nhóm đến từ cùng một nước.
Chứng minh rằng có thể chọn ra một nhóm gồm n người sao cho họ thuộc các nhóm khác
nhau và đến từ các nước khác nhau.

Ta gọi một nước X và một nhóm Y nào đó có liên hệ với nhau nếu trong nhóm Y có
người của nước X. Khi đó, một nước X bất kì có k người đại diện nên có liên hệ với k nước và
một nhóm Y bất kì có chứa k người đại diện khác nhau của các nước khác nhau nên có liên hệ
với đúng k nước. Do đó, một tập hợp bất kì m nước nào đó trong n nước đã cho sẽ có liên hệ với
m.k
ít nhất = m nhóm khác nhau.
k
Gọi ai , i = 1, n là n nhóm đã cho và X i ,1, n là tập hợp các nhóm có liên hệ với nước thứ i.
Theo điều vừa chứng minh ở trên, ta thấy với mọi:
k
i1 , i2 , i3 ,..., ik ∈ {1, 2,3,..., n} , 1 ≤ k ≤ n thì: ∪X ij ≥k. (*)
j =1

Ta sẽ chứng minh rằng tồn tại (a1 , a2 , a3 ,..., an ) ∈ X 1 × X 2 × X 3 × ... × X n và ai ≠ a j , i ≠ j .


Thật vậy, ta bắt đầu bỏ đi phần tử thuộc mỗi tập Xi sao cho (*) vẫn được thỏa mãn. Cuối
cùng thu được các tập hợp mới X '1 , X '2 , X '3 ,..., X 'n (với X 'i ⊂ X i ) vẫn thỏa điều kiện (*) có số
phần tử nhỏ nhất mà nếu bỏ đi thêm bất cứ phần tử thuộc tập hợp X 'i ,1, n thì điều kiện (*) sẽ
không còn được thỏa.
Ta chứng minh rằng X 'i = 1, i = 1, n .
Thật vậy, không mất tính tổng quát giả sử X '1 có chứa phần tử khác nhau là α , β . Do nếu bỏ
thêm một trong hai phần tử α hoặc β thì điều kiện (*) không còn thỏa mãn nên sẽ có hai tập chỉ
số P, Q sao cho:
Với M = ( X '1 \ {α }) ∪ ∪ X 'i , N = ( X '1 \{β }) ∪ ∪ X 'i không thỏa mãn điều kiện (*), tức là:
i∈P i∈Q

M < P + 1, N < Q + 1 ⇒ M ≤ P , N ≤ Q .

Ta có:

M ∪ N = (( X '1 \ {α }) ∪ ( X '1 \ {β })) ∪ (∪ X 'i ∪ ∪ X 'i ) = X '1 ∪ ∪ X 'i


i∈P i∈Q i∈P ∪Q

∪ X 'i ⊆ M ∩ N
i∈P ∩Q

77
Từ hai điều này suy ra: M ∪ N ≥ P ∪ Q + 1 , M ∩ N ≥ P ∩ Q

Theo nguyên lí bù trừ, ta có:

P + Q ≥ M + N = M ∪ N + M ∩ N ≥ P ∩ Q + P ∪ Q +1 = P + Q +1

Điều vô lí này dẫn đến khẳng định X 'i = 1, i = 1, n .

Rõ ràng các tập hợp này không giao nhau vì nếu tồn tại
i ≠ j , ( X 'i ∩ X ' j ) ≠ ∅ ⇒ X 'i ∩ X ' j = 1 < 2 , mâu thuẫn với điều kiện (*).

Giả sử mỗi phần tử của X 'i chính là a 'i thì tập hợp sau: (a '1 , a '2 , a '3 ,..., a 'n ) thỏa mãn
a 'i ∈ X 'i ⊂ X i và ai ≠ a j , i ≠ j .
Vậy ta đã chỉ ra rằng tồn tại n nhóm khác nhau tương ứng liên hệ với n nước khác nhau,
mỗi nhóm đó liên hệ với đúng một nước và mỗi nước liên hệ đúng một nhóm nên n người đại
diện mà mỗi nước liên hệ với mỗi nhóm tương ứng rõ ràng thỏa mãn đề bài.
Ta có đpcm.

78
Bài 6.
Gọi Sn là tổng bình phương các hệ số trong khai triển của nhị thức (1 + x)n , trong đó
n là số nguyên dương; x là số thực bất kì.
Chứng minh rằng: S2 n + 1 không chia hết cho 3 với mọi n.

Ta sẽ chứng minh bổ đề sau (định lí Lucas):

“Cho m, n là hai số tự nhiên và p là một số nguyên tố. Giả sử:

m = mk . p k + mk −1. p k −1 + ... + m2 . p 2 + m1 p + m0

n = nk . p k + nk −1. p k −1 + ... + n2 p 2 + n1. p + n0

k
Khi đó: Cnm ≡ ∏ Cnmi i (mod p ) (quy ước rằng Cba = 0, a > b ).
i =0

*Chứng minh:

Không mất tính tổng quát, giả sử m > n (nếu m = n thì bổ đề hiển nhiên đúng).

p! ( p − 1)!
Trước hết, ta thấy rằng: ( p, i ) = 1, i = 1, p − 1 nên C pk = = p. ⋮ p , tức là:
k !( p − k )! k !( p − k )!

C pk ≡ 0 (mod p ), k = 1, p − 1 .

p −1
Ta có: ( x + 1) p = x p + 1 + ∑ C ip .x p −i ≡ x p + 1(mod p ) . (*)
i =1

j j
Ta sẽ chứng minh nhận xét: ( x + 1) p ≡ x p + 1(mod p ), ∀j ∈ ℕ* bằng quy nạp. Thật vậy:

- Với j = 1 , nhận xét đúng theo (*).

- Giả sử nhận xét này đúng với j = h ≥ 1 . Ta sẽ chứng minh rằng nó cũng đúng với j = h + 1 .
h h
Ta có: ( x + 1) p ≡ x p + 1 (mod p ) .

( ) ≡ (x )
h h h h+1 h+1
ph
Suy ra: ( x + 1) p + 1 (mod p ) ⇒ ( x + 1) p ≡ x p + 1 (mod p ) .

Do đó nhận xét đúng với j = h + 1 . Theo nguyên lí quy nạp, nhận xét được chứng minh.

Ta xét khai triển sau:


79
k

∑ mi . pi k k mi
≡ ∏ (1 + x p ) mi ≡ ∏ ∑ Cmj i x j . p (mod p ) .
i i
(1 + x) m = (1 + x) i=0
i=0 i =0 j = 0

Hệ số của x n ở vế (1 + x)m là Cmn ; do biểu diễn n = nk . p k + nk −1. p k −1 + ... + n2 p 2 + n1. p + n0 là duy


k mi k

∏ ∑ Cmji x j. p là ∏C
i
mi
nhất nên hệ số của x n ở vế ni .
i =0 j =0 i =0

k
Từ đó ta được: Cnm ≡ ∏ Cnmi i (mod p ) .Bổ đề được chứng minh.
i =0

2 nn
 n  n 
*Trở lại bài toán: Ta có: (1 + x) 2 n = (1 + x)n .(n + 1) n ⇔ ∑ C2i n .x 2 n −i =  ∑ Cni .x n −i  .  ∑ Cnn −i .xi  .
i =1  i =0   i=0 
n n
Đồng nhất hệ số của x 2n ở hai vế, ta có: C2nn = ∑ Cni .Cnn −i = ∑ (Cni ) 2 .
i=0 i =0

Do đó, với mọi n tự nhiên thì S n = C2nn .

Như thế ta cần chứng minh rằng: C42nn + 1 không chia hết cho 3 với mọi n.

k
Giả sử: 2n = ∑ ai .3i , ai ∈ ℕ, i = 1, k . Xét hai trường hợp:
i =0

- Nếu ai ∈ {0;1}, ∀i = 1, k thì 2ai ∈ {0; 2}, ∀i = 1, k và tổng ai ∈ {0;1}, ∀i = 1, k là số chẵn, đặt
k
k ∑ ai k

∑ ai = 2t , t ∈ ℕ ⇒ 2 i=0 = 22t = 4t ≡ 1(mod 3) ; ta cũng có: 4n = ∑ 2ai .3i , ai ∈ ℕ, i = 1, k .


i =0 i =0

k
k k ∑ ai
Theo bổ đề trên thì C42nn + 1 ≡ ∏ C2aai i + 1 ≡ ∏ 2ai +1 ≡ 2 i=0 + 1 ≡ 2(mod 3) .
i=0 i =0

- Nếu tồn tại một giá trị a j = 2 ; không mất tính tổng quát, giả sử đây là số nhỏ nhất trong
tập hợp ai , i = 0, k . Khi đó: hệ số tương ứng tại vị trí j ở khai triển theo lũy thừa 3 của 4n là 1.
k
Mà C12 = 0 nên C42nn ≡ ∏ C2aai i ≡ 0 (mod 3) ⇒ C42nn + 1 ≡ 1 (mod 3) .
i =0

Vậy trong mọi trường hợp, ta đều có S2 n + 1 không chia hết cho 3. Đây chính là đpcm.

80
PHẦN 3
*****

HÌNH ẢNH CỦA


ĐỘI TUYỂN
QUA CÁC NĂM
81
*Năm 2005.

*Đội tuyển Việt Nam thi IMO 2005:

1.Trần Trọng Đan

2.Phạm Kim Hùng

3.Nguyễn Nguyên Hùng

4.Đỗ Quốc Khánh

5.Trần Chiêu Minh

6.Nguyễn Trường Thọ

82
*Năm 2006.

*Đội tuyển Việt Nam dự thi

IMO 2006:

1. Đặng Bảo Đức

2. Hoàng Mạnh Hùng

3. Nguyễn Duy Mạnh

4. Lê Hồng Quý

5. Nguyễn Xuân Thọ

6. Lê Nam Trường

83
*Năm 2007:

*Đội tuyển Việt Nam dự thi


IMO 2007:

1. Đỗ Xuân Bách

2. Nguyễn Xuân Chương

3. Lê Ngọc Sơn

4. Phạm Thành Thái

5. Đỗ Ngọc Thanh

6. Phạm Duy Tùng

84
*Năm 2008:

*Đội tuyển Việt Nam dự thi


IMO 2008:

1. Lê Ngọc Anh

2. Nguyễn Phạm Đạt

3. Dương Trọng Hoàng

4. Đỗ Thị Thu Thảo

5. Đặng Trần Tiến Vinh

6. Hoàng Đức Ý

85
*Năm 2009:

*Đội tuyển Việt Nam thi


IMO 2009:

1. Hà Khương Duy

2. Nguyễn Xuân Cương

3. Nguyễn Hoàng Hải

4. Phạm Hy Hiếu

5. Phạm Đức Hùng

6. Tạ Đức Thành

86
*Năm 2010:

*Đội tuyển Việt Nam thi


IMO 2010:

1. Phạm Việt Cường

2. Nguyễn Kiều Hiếu

3. Nguyễn Minh Hiếu

4. Trần Thái Hưng

5. Vũ Đình Long

6. Nguyễn Ngọc Trung

87

You might also like